TEST 2 SKILLS

Réussis tes devoirs et examens dès maintenant avec Quizwiz!

9. What is the removal of devitalized tissue from a wound called? 1. Debridement 2. Pressure distribution 3. Negative-pressure wound therapy 4. Sanitization

1

A nurse is teaching a group of mothers about first aid. Should poison come in contact with their childs clothing and skin, which action should the nurse instruct the mothers to take first? 1) Remove the contaminated clothing immediately. 2) Flood the contaminated area with lukewarm water. 3) Wash the contaminated area with soap and water and rinse. 4) Call the nearest poison control center immediately.

1

Physiological changes associated with aging place the older adult especially at risk for which nursing diagnosis? 1) Risk for Falls 2) Risk for Ineffective Airway Clearance (choking) 3) Risk for Poisoning 4) Risk for Suffocation (drowning)

1

The Joint Commissions national Speak Up campaign encourages patients to become active and informed participants on the healthcare team. The goal is to: 1) prevent healthcare errors. 2) help control the cost of healthcare. 3) reduce the number of automobile accidents. 4) provide a forum for people without health insurance.

1

The nurse suspects a 3-year-old child who is coughing vigorously has aspirated a small object. Which action should the nurse take first? 1) Encourage the child to continue coughing. 2) Deliver upward abdominal thrusts with a fisted hand. 3) Deliver five rapid back blows between the shoulder blades. 4) Perform a blind finger sweep of the childs mouth.

1

____ 10. Which of the following describes the difference between dehiscence and evisceration? 1) With dehiscence, there is a separation of one or more layers of wound tissue; evisceration involves the protrusion of internal viscera from the incision site. 2) Dehiscence is an urgent complication that requires surgery as soon as possible; evisceration is not as urgent. 3) Dehiscence involves the protrusion of internal viscera from the incision site; with evisceration, there is a separation of one or more layers of wound tissue. 4) Dehiscence involves rupture of subcutaneous tissue; evisceration involves damage to dermal tissue.

1

____ 12. Pressure ulcers are directly caused by which of the following conditions at the site? 1) Compromised blood flow 2) Edema 3) Shearing forces 4) Inadequate venous return

1

____ 17. Why is the information obtained from a swab culture of a wound limited? 1) A positive culture does not necessarily indicate infection, because chronic wounds are often colonized with bacteria. 2) A negative culture may not indicate infection, because chronic wounds are often colonized with bacteria. 3) Most wound infections are viral, so the swab culture would not be indicative of a wound infection. 4) A swab culture result does not include bacterial sensitivity information necessary to provide treatment.

1

____ 23. Your patient has a deep wound on the right hip, with tunneling at the 8 oclock position extending 5 cm. The wound is draining large amounts of serosanguineous fluid and contains 100% red beefy tissue in the wound bed. Of the following, which would be an appropriate dressing choice? 1) Alginate dressing 2) Dry gauze dressing 3) Hydrogel 4) Hydrocolloid dressing

1

____ 28. When applying heat or cold therapy to a wound, what should the nurse do? 1) Leave the therapy on each area no longer than 15 minutes. 2) Leave the therapy on each area no longer than 30 minutes. 3) When using heat, ensure the temperature is at least 135F (57.2C) before applying it. 4) When using cold, ensure the temperature is less than 32F (0C) before applying it.

1

____ 4. A patient with quadriplegia presents to the outpatient clinic with an ischial wound that extends through the epidermis into the dermis. When documenting the depth of the wound, how would the nurse classify it? 1) Partial-thickness wound 2) Penetrating wound 3) Superficial wound 4) Full-thickness wound

1

____ 7. What is the primary goal that the nurse should establish for a patient with an open wound? 1) The wound will remain free of infection throughout the healing process. 2) Client completes antibiotic treatment as ordered. 3) The wound will remain free of scar tissue at healing. 4) Client increases caloric intake throughout the healing process.

1

____ 9. Three days after a patient had abdominal surgery, the nurse notes a 4-cm periwound erythema and swelling at the distal end of the incision. The area is tender and warm to the touch. Staples are intact along the incision, and there is no obvious drainage. Heart rate is 96 beats/min and oral temperature is 100.8F (38.2C). The nurse would suspect that the patient has what kind of complication? 1) Infection at the incisional site 2) Dehiscence of the wound 3) Hematoma under the skin 4) Formation of granulation tissue

1

Before transferring a patient from the bed to a stretcher, which assessment data does the nurse need to gather? (Select all that apply.) 1. Patient's weight 2. Patient's activity tolerance 3. Patient's level of mobility 818 4. Recent laboratory values 5. Nutritional intake

1, 2, 3

The nurse must take a verbal order during an emergency on the unit. Which of the following guidelines can be used for taking verbal or telephone orders? (Select all that apply). 1. Only authorized staff may receive and record verbal or telephone orders. The health care agency identifies in writing the staff who are authorized. 2. Clearly identify patient's name, room number, and diagnosis. 3. Read back all orders to health care provider. 4. Use clarification questions to avoid misunderstandings. 5. Write "VO" (verbal order) or "TO" (telephone order), including date and time, name of patient, and complete order; sign the name of the health care provider and nurse.

1, 2, 3, 4, 5

The effects of immobility on the cardiac system include which of the following? (Select all that apply.) 1. Thrombus formation 2. Increased cardiac workload 3. Weak peripheral pulses 4. Irregular heartbeat 5. Orthostatic hypotension

1, 2, 5

8. When is the application of a warm compress to an ankle muscle sprain indicated? (Select all that apply.) 1. To relieve edema 2. To reduce shivering 3. To improve blood flow to an injured part 4. To protect bony prominences from pressure injuries 5. To immobilize area

1, 3

After receiving an intramuscular (IM) injection in the deltoid, a patient states, "My arm really hurts. It's burning and tingling where I got my injection." What should the nurse do next? (Select all that apply.) 1. Assess the injection site. 2. Administer an oral medication for pain. 3. Notify the patient's health care provider of assessment findings. 4. Document assessment findings and related interventions in the patient's medical record. 5. This is a normal finding, so nothing needs to be done. 6. Apply ice to the site for relief of burning pain.

1, 3 ,4

A 51-year-old adult comes to a medical clinic for an annual physical exam. The patient is found to be slightly overweight and reports being inactive, walking only 2 to 3 times a week with his wife after work. He has good muscle strength and coordination of lower extremities. Which of the following recommendations from the Physical Activity Guidelines for Americans should the nurse suggest? Choose all that apply 1. Move more and sit less throughout the day. 2. Participate in at least 90 minutes a week of moderate-intensity aerobic physical activity. 3. Perform muscle-strengthening activities using light weights on 2 or more days a week. 4. Walk at a vigorous pace with wife at least 150 minutes over five days a week. 5. Focus on balance training.

1, 3, 4

A nurse working on a medicine unit in the hospital hears the fire alarm go off. As the nurse walks down the hallway, there is smoke coming from the family waiting area. Which of the following steps should the nurse take? (Select all that apply.) 1. Immediately phone in to the hospital alert system the exact location of the fire. 2. Direct the nurse technician to place empty stretchers behind the fire doors. 3. Go to each patient room, and direct ambulatory patients to walk themselves to a safe area. 4. Work with the nurse technician to help move patients requiring wheelchairs from their rooms. 5. Close the room doors of patients who cannot get out of bed, and keep them in their rooms

1, 3, 4

A patient is receiving 40 mg of enoxaparin subcutaneously every 12 hours while on prolonged bed rest to prevent thrombophlebitis. Because bleeding is a potential side effect of this medication, the nurse should continually assess the patient for what signs of bleeding? (Select all that apply.) 1. Bruising 2. Pale yellow urine 3. Bleeding gums 4. Coffee ground-like vomitus 5. Light brown stool

1, 3, 4

____ 3. Which of the following are examples of nonselective mechanical dbridement methods? Choose all that apply. 1) Wet-to-dry dressings 2) Sharp dbridement 3) Whirlpool 4) Pulsed lavage

1, 3, 4 pg. 1248

10. Which of the following are measures to reduce tissue damage from shear? (Select all that apply.) 1. Use a transfer device (e.g., transfer board) 2. Have head of bed elevated when transferring patient 3. Have head of bed flat when repositioning patient 4. Raise head of bed 60 degrees when patient positioned supine 5. Raise head of bed 30 degrees when patient positioned supine

1, 3, 5

A middle-aged adult patient has limited mobility following a total knee arthroplasty. During assessment, the nurse notes that the patient is having difficulty breathing while lying supine. Which assessment data support a pulmonary issue related to immobility? (Select all that apply.) 1. Oxygen saturation of 89% 2. Irregular radial pulse 3. Diminished breath sounds bilateral bases on auscultation 4. BP: 132/84 5. Pain reported at 3 on scale of 0 to 10 following medication 6. Respiratory rate of 26

1, 3, 6

3. After surgery the patient with a closed abdominal wound reports a sudden "pop" after coughing. When the nurse examines the surgical wound site, the sutures are open, and pieces of small bowel are noted at the bottom of the now-opened wound. Which are the priority nursing interventions? (Select all that apply.) 1. Notify the health care provider. 2. Allow the area to be exposed to air until all drainage has stopped. 3. Place several cold packs over the area, protecting the skin around the wound. 4. Cover the area with sterile, saline-soaked towels immediately. 5. Cover the area with sterile gauze and apply an abdominal binder.

1, 4

____ 4. Why is an accurate description of the location of a wound important? Choose all that apply. 1) Influences the rate of healing 2) Determines the appropriate treatment choice 3) Will affect the frequency of dressing changes 4) Affects patient movement and mobility

1, 4

5. Which skin-care measures are used to manage a patient who is experiencing fecal and/or urinary incontinence? (Select all that apply.) 1. Frequent position changes 2. Keeping the buttocks exposed to air at all times 3. Using a large absorbent diaper, changing when saturated 4. Using an incontinence cleaner 5. Applying a moisture barrier ointment

1, 4, 5

Family members have asked for a meeting with the nursing staff of an assisted-living residential center to discuss the feasibility of their mother using a walker. The family is worried that her health is declining; they wonder whether she can use the walker safely. Which of the following instructions should the nurse give the family after assessing that it is safe for the woman to use a walker? (Select all that apply.) 1. A walker is useful for patients who have impaired balance. 2. The patient uses a walker by pushing the device forward. 3. Leaning over the walker improves the patient's balance. 4. Walkers should not be used on stairs. 5. If the patient has difficulty advancing the walker, a walker with wheels is an option.

1, 4, 5

An older adult states that she cannot see her medication bottles clearly to determine when to take her prescription. What should the nurse do? (Select all that apply.) 1. Provide a dispensing system for each day of the week. 2. Provide larger, easier-to-read labels. 3. Tell the patient what is in each container. 4. Have a family caregiver administer the medication. 5. Use teach-back to ensure that the patient knows what medication to take and when.

1,2,5

A patient has been on bed rest for over 5 days. Which of these findings during the nurse's assessment may indicate a complication of immobility? 1. Decreased peristalsis 2. Decreased heart rate 3. Increased blood pressure 4. Increased urinary output

1. Decreased peristalsis

The nurse is caring for an older adult in a long-term care setting. The nurse reviews the medical record to find that the patient has progressive loss of total bone mass. The patient's history and tendency to take smaller steps with feet kept closer together will most likely result in which of the following? 1. Increase the patient's risk for falls and injuries 2. Result in less stress on the patient's joints 3. Decrease the amount of work required for patient movement 4. Allow for mobility in spite of the aging effects on the patient's joints

1. Increase the patient's risk for falls and injuries

A nurse is helping a patient perform active assisted range of motion in the right elbow. Which statement describes the correct technique? 1. Support elbow by holding distal part of extremity. 2. Grasp joint with fingers to provide support. 3. Have patient move joint independently. 4. Move the joint past the point of resistance. 5. Perform the exercise a few times only, and gradually build up to more.

1. Support elbow by holding distal part of extremity.

The nurse is administering 250 mg of a medication elixir to the patient. The medication comes in a dose of 1000 mg/5 mL. How many milliliters should the nurse administer? Record your answer using two decimal places. ____ mL

1.25

The patient is to receive amoxicillin 500 mg q8h; the medication is dispensed at 250 mg/5mL. How many milliliters will the nurse administer for one dose? ___ mL

10

Match the fall prevention intervention on the left with the scientific rationale on the right. 1. Prioritize nurse call system responses to patients at high risk. 2. Place patient in a wheelchair with wedge cushion. 3. Establish elimination schedule with bedside commode. 4. Use a low bed for patient. 5. Provide a hip protector. 6. Place nonskid floor mat on floor next to bed. A. Maintains comfort and makes exit difficult B. Makes it difficult for patients with lower extremity weakness to stand C. Reduces slipping when walking D. Reduces fall impact E. Ensures rapid response for help F. Reduces chance of patient trying to get out of bed on own

1E, 2A, 3F, 4B, 5D, 6C.

. A 78-year-old patient is being seen in the emergency department. The nurse observes his gait and balance appear to be slightly unsteady. What assessment should the nurse perform next? 1) Perform the Get Up and Go Test. 2) Ask the patient if he has fallen in the past year. 3) Refer the patient for a comprehensive fall evaluation. 4) Administer the Timed Up and Go Test.

2

1. It is important to take precautions to prevent medication errors. A nurse is administering an oral tablet to a patient. Which of the following steps is the second check for accuracy in determining the patient is receiving the right medication? 1. Logging on to automated dispensing system (ADS) or unlocking medicine drawer or cart. 2. Before going to patient's room, comparing patient's name and name of medication on label of prepared drugs with MAR. 3. Selecting correct medication from ADS, unit-dose drawer, or stock supply and comparing name of medication on label with MAR or computer printout. 4. Comparing MAR or computer printout with names of medications on medication labels and patient name at patient's bedside.

2

4. A patient with a history of falling continually attempts to get out of bed unassisted despite frequent reminders to call for help first. Which action should the nurse take first? 1) Apply a cloth vest restraint. 2) Encourage a family member to stay with the patient. 3) Administer lorazepam (an antianxiety medication). 4) Keep the patients bed side rails up.

2

A patient has received a radiation implant. The patient is weak and needs help even to turn in bed. Which action should the nurse take when caring for this patient? 1) Avoid giving the patient a complete bed bath. 2) Limit the amount of time spent with the patient. 3) Allow extra time for the patient to express feelings. 4) Do not allow anyone to visit the patient.

2

____ 19. A man was involved in a motor vehicle accident yesterday. He is to be sedated for over 2 weeks while breathing with the assistance of a mechanical ventilator. Which of the following would be an appropriate nursing diagnosis for him at this time? 1) Risk for Infection related to subcutaneous injuries 2) Risk for Impaired Skin Integrity related to immobility 3) Impaired Tissue Integrity related to ventilator dependency 4) Impaired Skin Integrity related to ventilator dependency

2

____ 2. Skin integrity and wound healing are compromised in the client who takes blood pressure medications because antihypertensives: 1) Can cause cellular toxicity. 2) Increase the risk of ischemia. 3) Delay wound healing. 4) Predispose to hematoma formation.

2

____ 24. Of the following, which is the best choice for performing wound irrigation? 1) Water jet irrigation 2) 35-cc syringe with a 19-gauge angiocatheter 3) 5-cc syringe with a 23-gauge needle 4) Bulb syringe

2

____ 26. A patient had abdominal surgery. The incision has been closed by primary intention, and the staples are intact. To provide more support to the incision site and decrease the risk of dehiscence, it would be appropriate to apply which of the following? 1) Steri-Strips 2) Abdominal binder 3) T-binder 4) Paper tape

2

____ 30. What is a common characteristic of aging skin? 1) Increased permeability to moisture 2) Diminished sweat gland activity 3) Reduced oxygen-free radicals 4) Overproduction of elastin

2

____ 5. A patient underwent abdominal surgery for a ruptured appendix. The surgeon did not surgically close the wound. The wound healing process described in this situation is: 1) Primary intention healing. 2) Secondary intention healing. 3) Tertiary intention healing. 4) Approximation healing.

2

____ 8. While assessing a new wound, the nurse notes red, watery drainage. What type of drainage will the nurse document this as? 1) Sanguineous 2) Serosanguineous 3) Serous 4) Purosanguineous

2

A nurse is administering ophthalmic ointment to a patient. Place the following steps in correct order for the administration of the ointment. 1. Clean eye, washing from inner to outer canthus. 2. Assess patient's level of consciousness and ability to follow instructions. 3. Apply thin ribbon of ointment evenly along inner edge of lower eyelid on conjunctiva. 4. Have patient close eye and rub lightly in a circular motion with a cotton ball. 5. Ask patient to look at ceiling, and explain the steps to patient.

2, 1, 5, 3, 4

A 46-year-old patient is admitted to the emergency department following an automobile accident. The patient has a pelvic fracture and is ordered on bed rest and placed in an immobilization device to limit further injury until the fracture can safely be repaired. Which measures would be appropriate for this patient to prevent complications of bed rest? (Select all that apply.) 1. Administer intravenous analgesic as ordered. 2. Have patient perform incentive spirometry. 3. Support patient in active assistive ROM exercises of upper extremities. 4. Provide patient a low-calorie diet. 5. Apply sequential compression devices to legs.

2, 3, 5

A nurse working the night shift is assigned a patient who has a history of having fallen in the hospital during a previous admission. The nurse wants to review the admission assessment completed by the nurse on the day shift. Which of the following sections in the assessment are most likely to provide information about the patient's current fall risks? (Select all that apply.) 1. Allergy history 2. Medication history 3. Patient age 4. Patient's occupation 5. Physical exam of neuromuscular function

2, 3, 5

A nurse is administering a metered-dose inhaler (MDI) with a spacer to a patient with chronic obstructive pulmonary disease. Place the steps of the procedure in the correct order. 1. Insert MDI into end of spacer. 2. Perform a respiratory assessment. 3. Remove mouthpiece from MDI and spacer device. 4. Place the spacer mouthpiece into patient's mouth, and instruct patient to close lips around the mouthpiece. 5. Depress medication canister, spraying 1 puff into spacer device. 6. Shake inhaler for 2-5 seconds. 7. Instruct patient to hold breath for 10 seconds. 8. Instruct patient to breathe in slowly through mouth for 3 to 5 seconds.

2, 3, 6, 1, 4, 5, 8, 7

7. Which of the following is an indication for a binder to be placed around a surgical patient with a new abdominal wound? (Select all that apply.) 1. Collection of wound drainage 2. Providing support to abdominal tissues when coughing or walking 3. Reduction of abdominal swelling 4. Reduction of stress on the abdominal incision 5. Stimulation of peristalsis (return of bowel function) from direct pressure

2, 4

____ 1. Select the process(es) that occur(s) during the inflammatory phase of wound healing. Choose all that apply. 1) Granulation 2) Hemostasis 3) Epithelialization 4) Inflammation

2, 4

A patient has an order for application of compression stockings. Place the following steps for application of the stockings in the correct order: 1. Place patient's toes into foot of stocking up to the heel; keep smooth. 2. Use tape measure to measure patient's leg for proper stocking size. 3. Slide stocking up over patient's calf until sock is completely extended. 4. Turn elastic stocking inside out, keeping hand inside holding heel. Take other hand and pull stocking inside out until reaching the heel. 5. Slide remaining portion of stocking over patient's foot, covering toes. Be sure foot fits into toe and heel of stocking.

2, 4, 1, 5, 3

Which of the following are safe practices to follow in the safe preparation and storage of food? (Select all that apply.) 1. Always use a single cutting board to prepare foods for cooking. 2. Refrigerate leftovers as soon as possible. 3. Always buy vegetables in packages marked "prewashed." 4. Cook meats to the proper temperature. 5. Wash hands thoroughly before food preparation.

2, 4, 5

Which of the following motivates a patient to participate in an exercise program? (Select all that apply.) 1. Providing a patient with a pamphlet on exercise 2. Providing information to the patient when he or she is ready to change behavior 3. Explaining the importance of exercise at the time of diagnosis of a chronic disease 4. Having a structured daily plan that incorporates physical activity 5. Having support from significant other to engage in exercise

2, 4, 5

The nurse is administering an IV push medication to a patient who has a compatible IV fluid running through intravenous tubing. Place the following steps in the appropriate order. 1. Release tubing and inject medication within amount of time recommended by agency policy, pharmacist, or medication reference manual. Use watch to time administration. 2. Select injection port of IV tubing closest to patient. Whenever possible, injection port should accept a needleless syringe. Use IV filter if required by medication reference or agency policy. 3. After injecting medication, release tubing, withdraw syringe, and recheck fluid infusion rate. 4. Connect syringe to port of IV line. Insert needleless tip or small-gauge needle of syringe containing prepared drug through center of injection port 5. Clean injection port with antiseptic swab. Allow to dry. 6. Occlude IV line by pinching tubing just above injection port. Pull back gently on syringe plunger to aspirate blood return.

2, 5, 4, 6, 1, 3

Which point(s) should the nurse include when teaching safety precautions to a mother of a toddler? Select all that apply. 1) Make sure the child sleeps on his back at night. 2) Keep the telephone number of the poison control center accessible. 3) Use a front-facing car seat placed in the back seat of the car. 4) Keep syrup of ipecac on hand in case of accidental poisoning.

2,3

A patient is experiencing some problems with joint stability in the right leg. The doctor has prescribed crutches for the patient to use while being allowed to bear weight only on the left leg. Which of the following gaits should the patient be taught to use? 1. Four-point 2. Three-point 3. Two-point 4. Swing-through

2. Three-point

____ 1. What is the function of the stratum corneum? 1) Provides insulation for temperature regulation 2) Provides strength and elasticity to the skin 3) Protects the body against the entry of pathogens 4) Continually produces new skin cells

3

____ 18. For the client with a stage IV pressure ulcer, what would an applicable patient goal/outcome be? 1) Client will maintain intact skin throughout hospitalization. 2) Client will limit pressure to wound site throughout treatment course. 3) Wound will close with no evidence of infection within 6 weeks. 4) Wound will improve prior to discharge as evidenced by a decrease in drainage.

3

____ 20. What intervention would be most appropriate for a wound with a beefy red wound bed? 1) Mechanical dbridement 2) Autolytic dbridement 3) Dressing to keep the wound moist and clean 4) Removal of devitalized tissue and a sterile dressing

3

____ 21. A patient has a stage II pressure ulcer on her right buttock. The ulcer is covered with dry, yellow slough that tightly adheres to the wound. What is the best treatment the nurse could recommend for treating this wound? 1) Dry gauze dressing changed twice daily 2) Nonadherent dressing with daily wound care 3) Hydrocolloid dressing changed as needed 4) Wet-to-dry dressings changed three times a day

3

____ 27. A patient has an area of nonblanchable erythema on his coccyx. The nurse has determined this to be a stage I pressure ulcer. What would be the most important treatment for this patient? 1) Transparent film dressing 2) Sheet hydrogel 3) Frequent turn schedule 4) Enzymatic dbridement

3

____ 31. Which client does the nurse recognize as being at greatest risk for pressure ulcers? 1) Infant with skin excoriations in the diaper region 2) Young adult with diabetes in skeletal traction 3) Middle-aged adult with quadriplegia 4) Older adult requiring use of assistive device for ambulation

3

____ 6. When teaching a patient about the healing process of an open wound after surgery, which of the following points would the nurse make? 1) The patient will need to take antibiotics until the wound is completely healed. 2) Because the patients wound was left open, the wound will likely become infected. 3) The patient will have more scar tissue formation than for a wound closed at surgery. 4) The patient should expect to remain hospitalized until complete wound healing occurs.

3

9. Place the following steps for applying a wrist restraint in the correct order: 1. Pad the skin overlying the wrist. 2. Insert two fingers under the secured restraint to be sure that it is not too tight. 3. Be sure that the patient is comfortable and in correct anatomical alignment. 4. Secure restraint straps to bedframe with quick-release buckle. 5. Wrap limb restraint around wrist or ankle with soft part toward skin and secure snugly.

3, 1, 5, 2, 4

An older-adult patient is admitted following a hip fracture and surgical repair. Before ambulating the patient postoperatively on the evening of surgery, which of the following would be most important to assess? (Select all that apply.) 1. Patient's usual dietary intake 2. Time and date of the patient's last bowel movement 3. Preadmission activity tolerance 4. Baseline heart rate 5. Patient's home living situation

3, 4

A nurse enters the hospital room of a patient who had a total knee replacement the day before. Which of the following pose potential safety risks? (Select all that apply.) 1. A current safety inspection sticker is on the IV fluids pump. 2. A walker is positioned near the patient's bedside. 3. The hospital bed is in the high position. 4. There is no gait belt at the bedside. 5. The overbed table with the patient's glasses is positioned against the wall opposite the end of the bed.

3, 4, 5

A patient is to receive medications through a small-bore nasogastric feeding. Which nursing actions are appropriate? (Select all that apply.) 1. Verifying tube placement after medications are given 2. Mixing all medications together to give all at once 3. Using an enteral tube syringe to administer medications 4. Flushing tube with 30 to 60 mL of water after the last dose of medication 5. Checking for gastric residual before giving the medications 6. Keeping the head of the bed elevated 30 to 60 minutes after the medications are given

3, 4, 5, 6

The nurse finds a 68-year-old woman wandering in the hallway and exhibiting confusion. The patients says she is looking for the bathroom. Which interventions are appropriate for this patient? (Select all that apply.) 1. Ask the health care provider to order a restraint. 2. Recommend insertion of a urinary catheter. 3. Provide scheduled toileting rounds every 2 to 3 hours. 4. Institute a routine exercise program for the patient. 5. Keep the bed in high position with side rails down. 6. Keep the pathway from the bed to the bathroom clear.

3, 4, 6

A nurse is instructing a patient who has decreased leg strength on the left side on how to use a cane. Which actions indicate proper cane use by the patient? (Select all that apply.) 1. The patient keeps the cane on the left side of the body. 2. The patient slightly leans to one side while walking. 3. The patient keeps two points of support on the floor at all times. 4. After the patient places the cane forward, he or she then moves the right leg forward to the cane. 5. The patient places the cane forward 15 to 25 cm (6 to 10 inches) with each step.

3, 5

Which of the following indicates that additional assistance is needed to transfer a patient from the bed to the stretcher? (Select all that apply.) 1. The patient is 5 feet, 6 inches and weighs 120 lb. 2. The patient speaks and understands English. 3. The patient is returning to unit from recovery room after a procedure requiring conscious sedation. 4. The patient has a history of being able to stand independently. 5. The patient received analgesia for pain 30 minutes ago.

3, 5

Place the following steps in the correct order for positioning a patient in the 30-degree lateral side-lying position. 1. Raise side rail and go to opposite side of bed. 2. Lower side rail and flex patient's knee that will not be next to mattress. Keep foot on mattress and place one hand on patient's upper bent leg near hip and other hand on shoulder. 3. Lower head of bed flat if patient can tolerate it. 4. Roll patient onto side toward you. 5. Lower side rail and position patient on side of bed opposite the direction toward which patient is to be turned. 6. Place hands under patient's dependent shoulder and bring shoulder blade forward. 7. Place hands under patient's dependent hip and bring hip slightly forward so that angle from hip to mattress is approximately 30 degrees.

3, 5, 1, 2, 4, 6, 7

____ 2. What are two risk assessment tools used in the United States to evaluate a patients risk for pressure ulcers? Choose all that apply. 1) Pressure ulcer healing chart 2) PUSH tool 3) Braden scale 4) Norton scale

3,4

A patient is taking 1 tablet of hydrocodone bitartrate 5 mg and acetaminophen 500 mg every 4 hours. The patient is also taking 2 tablets of acetaminophen 325 mg every 12 hours. How many grams of acetaminophen is the patient taking daily? . ______ g

3.3

medical errors are the ________ leading cause of death in the US

3rd

6. Which of the following describes a hydrocolloid dressing? 1. A seaweed derivative that is highly absorptive 2. Premoistened gauze placed over a granulating wound 3. A debriding enzyme that is used to remove necrotic tissue 4. A dressing that forms a gel that interacts with the wound surface

4

A patient in the emergency department is angry, yelling, cursing, and waving his arms when the nurse comes to the treatment cubicle. Which action(s) by the nurse is(are) advisable? 1) Reassure the patient by entering the room alone. 2) Ask the patient if he is carrying any weapons. 3) Stay between the patient and the door; keep the door open. 4) Make eye contact while stating firmly I will not tolerate cursing and threats.

4

Despite less-restrictive interventions, a patients behavior escalates, requiring emergency application of restraints. Which of the following must the nurse do in this situation? 1) Obtain a physicians order before applying restraints. 2) Monitor the patients status every 4 hours while restrained. 3) Release the restraints and check circulation every hour. 4) Continually reevaluate the patients need for restraint.

4

Which aspect of restraint use can the nurse delegate to the nursing assistive personnel? 1) Assessing the patients status 2) Determining the need for restraint 3) Evaluating the patients response to restraints 4) Applying and removing the restraints

4

Which of the following instructions is most important for the nurse to include when teaching a mother of a 3-year-old about protecting her child against accidental poisoning? 1) Store medications on countertops out of the childs reach. 2) Purchase medication in child-resistant containers 3) Take medications in front of the child, and explain that they are for adults only. 4) Never leave the child unattended around medications or cleaning solutions.

4

____ 11. The nurse will know that the plan of care for the diabetic client with severe peripheral neuropathy is effective if the client: 1) begins an aggressive exercise program. 2) follows a diet plan of 1,200 calories per day. 3) is fitted for deep-depth diabetic footwear. 4) remains free of foot wounds.

4

____ 13. A patient hospitalized in a long-term rehabilitation facility is immobile and requires mechanical ventilation with a tracheostomy. She has a pressure area on her coccyx measuring 5 cm by 3 cm. The area is covered with 100% eschar. What would the nurse identify this as? 1) Stage II pressure ulcer 2) Stage III pressure ulcer 3) Stage IV pressure ulcer 4) Unstageable pressure ulcer

4

____ 14. A client developed a stage IV pressure ulcer to his sacrum 6 weeks ago, and now the ulcer appears to be a shallow crater involving only partial skin loss. What would the nurse now classify the pressure ulcer as? 1) Stage I pressure ulcer, healing 2) Stage II pressure ulcer, healing 3) Stage III pressure ulcer, healing 4) Stage IV pressure ulcer, healing

4

____ 15. A patient has underlying cardiac disease and requires careful monitoring of his fluid balance. He also has a draining wound. Which of the following methods for evaluating his wound drainage would be most appropriate for assessing fluid loss? 1) Draw a circle around the area of drainage on a dressing. 2) Classify drainage as less or more than the previous drainage. 3) Weigh the patient at the same time each day on the same scale. 4) Weigh dressings before they are applied and after they are removed.

4

____ 16. A patient had a CVA (stroke) 2 days ago, resulting in decreased mobility to her left side. During the assessment, the nurse discovers a stage I pressure area on the patients left heel. What is the initial treatment for this pressure ulcer? 1) Antibiotic therapy for 2 weeks 2) Normal saline irrigation of the ulcer daily 3) Dbridement to the left heel 4) Elevation of the left heel off the bed

4

____ 22. The nurse would know care for a stage II pressure ulcer is achieving the desired goal when: 1) The ulcer is completely healed with minimal scarring. 2) The patient reports no pain at the site. 3) A minimal amount of drainage is noted. 4) The wound bed contains 100% granulated tissue.

4

____ 25. Your patient has multiple open wounds that require treatment. When performing dressing changes, you should: 1) Remove all of the soiled dressings before beginning wound treatment. 2) Cleanse wounds from most contaminated to least contaminated. 3) Treat wounds on the patients side first, then the front and back of the patient. 4) Irrigate wounds from least contaminated to most contaminated.

4

____ 29. A patient has a contaminated right hip wound that requires dressing changes twice daily. The surgeon informs the nurse that when the wound heals a little more he will suture it closed. The nurse recognizes that the surgeon is using which form of wound healing? 1) Primary intention 2) Regenerative healing 3) Secondary intention 4) Tertiary intention

4

____ 3. What is the primary difference between acute and chronic wounds? Chronic wounds: 1) Are full-thickness wounds, but acute wounds are superficial. 2) Result from pressure, but acute wounds result from surgery. 3) Are usually infected, whereas acute wounds are contaminated. 4) Exceed the typical healing time, but acute wounds heal readily.

4

____ 32. The nurse working in the emergency department is preparing heat therapy for one of the patients in the unit. Which one is it most likely to be? Choose all that apply. 1) Is actively bleeding 2) Has swollen, tender insect bite 3) Has just sprained her ankle 4) Has lower back pain

4

4. What is the correct sequence of steps when performing wound irrigation to a large open wound? 1. Use slow, continuous pressure to irrigate wound. 2. Attach 19-gauge angiocatheter to syringe. 3. Fill syringe with irrigation fluid. 4. Place biohazard bag near bed. 5. Position angiocatheter over wound.

4, 3, 2, 5, 1

When repositioning an immobile patient, the nurse notices redness over the hip bone. What is indicated when a reddened area blanches on fingertip touch? 1. A local skin infection requiring antibiotics 2. Sensitive skin that requires special bed linen 3. A stage 3 pressure injury needing the appropriate dressing 4. Blanching hyperemia, indicating the attempt by the body to overcome the ischemic episode

4. Blanching hyperemia, indicating the attempt by the body to overcome the ischemic episode

An older-adult patient has been bedridden for 2 weeks. Which of these complaints by the patient indicates to the nurse that he or she is developing a complication of immobility? 1. Increase of appetite 2. Gum soreness 3. Difficulty in swallowing 4. Left ankle joint stiffness

4. Left ankle joint stiffness

A nurse working on a surgery floor is assigned four patients. The nurse assesses each patient, noting behaviors and physical signs and symptoms. Which of the following patients is more likely to be violent toward the nurse? 1. The first patient maintains eye contact with the nurse, is calm during the nurse's assessment, and asks questions frequently. 2. The second patient is very drowsy, loses attention span when the nurse asks questions, and mumbles when speaking. 3. The third patient moves nervously in bed, swears and grimaces when trying to cough, and speaks in a low volume. 4. The fourth patient speaks in a loud voice and becomes irritable when the nurse arrives to help walk the patient.

4. The fourth patient speaks in a loud voice and becomes irritable when the nurse arrives to help walk the patient.

Place in the correct order the steps needed (below) to transfer a patient with sufficient lower body strength to a chair. 1. On count of three, instruct patient to stand while straightening hips and legs and keeping knees slightly flexed. 2. Assist patient to assume proper alignment in sitting position. 3. Help patient apply stable, nonskid shoes/socks. 4. Spread your feet apart. Flex hips and knees, aligning knees with patient's knees. 5. Apply gait/transfer belt. 6. Maintain patient's balance as you pivot foot farthest from chair and then help patient ease into chair. 7. Grasp transfer belt along patient's sides.

5, 3, 4, 7, 1, 6, 2

5. a nurse is caring for an older adult client who is at risk for developing pressure ulcers. Which of the following interventions should the nurse use to help maintain the integrity of the client's skin? (select all that apply.) a. Keep the head of the bed elevated 30°. B. Massage the client's bony prominences frequently. c. apply cornstarch liberally to the skin after bathing. d. Have the client sit on a gel cushion when in a chair. e. reposition the client at least every 3 hr while in bed.

5. a. CORRECT: the nurse should slightly elevate the client head of bed reduce shearing forces that could tear sensitive skin on the sacrum, buttocks, and heels. B. the nurse can traumatize deep tissues if she massages the skin over bony prominences. c. the nurse can abrade the client's sensitive skin and increase the risk for aspiration if cornstarch or powders are used. d. CORRECT: the nurse should have the client sit on a gel, air, or foam cushion to redistribute weight away from ischial areas. e. the nurse should reposition the client at least every 2 hr. Frequent position changes are important for preventing skin breakdown, but every 3 hr is not frequent enough.

Place the steps of administering an intradermal injection in the correct order. 1. Inject medication slowly. 2. Note the presence of a bleb. 3. Advance needle through epidermis to 3 mm. 4. Using nondominant hand, stretch skin over site with forefinger. 5. Insert needle at a 5- to 15-degree angle into the skin until resistance is felt. 6. Cleanse site with antiseptic swab.

6, 4, 5, 3, 1, 2

For clients at risk for falls, repeat the assessment every ___ hours?

8

43. A patient who is receiving IV fluids notifies the nurse that his arm feels tight. Upon assessment, the nurse notes that the arm is swollen and cool to the touch. What should the nurse's first action be? a. Discontinue the IV site, and apply a warm compress. b. Attached a syringe, and pull back on the plunger to aspirate the IV fluid. c. Start a new IV site distal from the site. d. Stop the IV fluids, and notify the physician immediately.

A An IV site that is puffy, swollen, and cool to the touch indicates infiltration. The IV site should be discontinued immediately because it is no longer a viable access point. Pulling back on the syringe will not result in fluid return because there is no longer venous access. A new IV should be started in the opposite arm after the old IV has been removed. The IV should be removed; it is not sufficient to only stop the fluids.

23. The nurse knows that patient education about a buccal medication has been effective when the patient states a. "I should let the medication dissolve completely." b. "I can only drink water, not juice, with this medication." c. "For faster distribution, I should chew my medication first." d. "I should place the medication in the same location."

A "I should let the medication dissolve completely." Buccal medications should be placed in the side of the cheek and allowed to dissolve completely. Buccal medications act with the patient's saliva and mucosa. The patient should not chew or swallow the medication. Gastric secretions may destroy some medications. The patient should rotate sides of the check to avoid irritating the mucosal lining.

12. The nurse is planning to administer a tuberculin test with a 27-gauge, 3/8-inch needle. The nurse should insert the needle at an angle of _____ degrees. a. 15 b. 45 c. 90 d. 180

A 15 A 27-gauge, 3/8-inch needle is used for intradermal injections such as a tuberculin test, which should be inserted at a 15-degree angle, just under the dermis of the skin. Placing the needle at 45 degrees, 90 degrees, or 180 degrees will place the medication too deep.

30. The nurse realizes which patient is at greatest risk for an unintended synergistic effect? a. 72-year-old who is seeing four different specialists b. 4-year-old who has mistakenly taken the entire packet of his mother's birth control pills c. 50-year-old who was prescribed a second blood pressure medication d. 35-year-old drug addict who has ingested meth mixed with several household chemicals

A 72-year-old who is seeing four different specialist A synergistic effect occurs when two medications potentiate each other, creating a greater effect than a single medication on its own. The 72-year-old seeing four different providers is likely to experience polypharmacy. Polypharmacy places the patient at risk for unintended mixing of medications that potentiate each other. The child taking too much of a medication by mistake could experience overdose or toxicity. The 50-year-old is prescribed two different blood pressure medications for their synergistic effect, but this is a desired event. A drug addict mixing chemicals can be toxic.

12. The nurse is attempting to start an exercise program in a local community as a health promotion project. In explaining the purpose of the project, the nurse explains to community leaders that a. A sedentary lifestyle contributes to the development of health-related problems. b. The recommended frequency of workouts should be twice a day. c. An exercise prescription should incorporate aerobic exercise only. d. The purpose of weight training is to bulk up muscles.

A A sedentary lifestyle contributes to the development of health-related problems. A sedentary lifestyle contributes to the development of health-related problems. A holistic approach is taken to develop overall fitness and includes warm-ups, aerobic exercise, resistance training, weight training, and so forth. The recommended frequency of aerobic exercise is 3 to 5 times per week or every other day for approximately 30 minutes. Cross-training is recommended for the patient who prefers to exercise every day. Some patients use weight training to bulk up their muscles. However, the purposes of weight training from a health perspective are to develop tone and strength and to simulate and maintain healthy bone.

20. The patient weighs 450 lbs (204.5 kg) and complains of shortness of breath with any exertion. His health care provider has recommended that he begin an exercise program. He states that he can hardly get out of bed and just cannot do anything around the house. To focus on the cause of the patient's complaints, the nurse devises which of the following nursing diagnoses? a. Activity intolerance related to excessive weight b. Activity intolerance related to bed rest c. Impaired gas exchange related to shortness of breath d. Imbalanced nutrition: less than body requirements

A Activity intolerance related to excessive weight The diagnostic label directs nursing interventions. This requires the correct selection of related factors. For example, Activity intolerance related to excess weight gain requires very different interventions than if the related factor is prolonged bed rest. In this case, the intolerance is related to the patient's excessive weight. He is not on bed rest, although he claims that it is difficult for him to get out of bed. Shortness of breath is a symptom, not a cause, of Impaired gas exchange, making this nursing diagnosis ineffective. The patient certainly has an imbalance of nutrition, but it is more than body requirements.

24. What is the nurse's priority action to protect a patient from medication error? a. Administering as many of the medications as possible at one time b. Asking anxious family members to leave the room before giving a medication c. Checking the patient's room number against the medication administration record d. Administering as many of the medications as possible at one time

A Administering as many of the medications as possible at one time Verbal orders should be limited to urgent situations where written communication is unavailable. The nurse should explain the reasons and logistics of a procedure to calm anxious family members, and should ask family members not to distract medication administration for the patient's safety. After proper education, if the family members are creating an unsafe environment, the nurse may ask them to step out of the room. The medication administration record should be checked against the patient's hospital identification band; a room number is not an acceptable identifier. Medications should be given when scheduled, and medications with special assessment indications should be separated.

7. The nurse is preparing to reposition a patient. Before doing so, the nurse must a. Assess the weight to be lifted and the assistance needed. b. Attempt to manually lift the patient alone before asking for assistance. c. Attempt a manual lift only when lifting most or all of the patient's weight. d. Not use the agency lift team if a mechanical lift is available.

A Assess the weight to be lifted and the assistance needed. Before lifting, assess the weight to be lifted and determine the assistance needed and the resources available. Manual lifting is the last resort, and it is used when the task at hand does not involve lifting most or all of the patient's weight. Use safe patient handling equipment in conjunction with agency lift teams to reduce the risk of injury to the patient and members of the health care team.

3. The nurse is completing a skin assessment on a medical-surgical patient. Which nursing assessment questions should be included in a skin integrity assessment? (Select all that apply.) a. "Can you easily change your position?" b. "Do you have sensitivity to heat or cold?" c. "How often do you need to use the toilet?" d. "Is movement painful?" e. "What medications do you take?" f. "Have you ever fallen?"

A B C D

21. The patient is being admitted for elective knee surgery. While the nurse is admitting the patient, she will a. Begin to develop a discharge plan. b. Plan to wait until after the surgery to plan for discharge. c. Place a generalized discharge plan in the record for later use. d. Address immediate needs of the patient only and address other needs later.

A Begin to develop a discharge plan. The nurse needs to begin discharge planning when the patient enters the health care system. The nurse cannot wait until after surgery to begin to plan for discharge. In addition, the discharge plan is always individualized to the patient and directed at meeting the actual and/or potential needs of the patient.

8. Isotonic, isometric, and resistive isometric are three categories of exercise. They are classified according to the type of muscle contraction involved. Of the following exercises, which are considered isotonic? a. Bicycling, swimming, walking, jogging, dancing b. Tightening or tensing of muscles without moving body parts c. Push-ups, hip lifting, pushing feet against a footboard on the bed d. Quadriceps set exercises and contraction of the gluteal muscles

A Bicycling, swimming, walking, jogging, dancing Examples of isotonic exercises are walking, swimming, dance aerobics, jogging, bicycling, and moving arms and legs with light resistance. Isometric exercises involve tightening or tensing of muscles without moving body parts. Examples include quadriceps set exercises and contraction of the gluteal muscles. Examples of resistive isometric exercises are push-ups and hip lifting, as well as placing a footboard on the foot of the bed for patients to push against with their feet.

2. The nurse is caring for a patient with wound healing by tertiary intention. Which factors does the nurse recognize as influencing wound healing? (Select all that apply.) a. Nutrition b. Evisceration c. Tissue perfusion d. Infection e. Hemorrhage f. Age

A C D F

7. A patient who has been receiving intermittent chemotherapy through a peripheral IV site is ordered to receive a high dose of vancomycin through the same vein. Why does this concern the nurse? a. Chemotherapy is irritating to the vascular system and may cause the vein to infiltrate. b. Two medications should never be placed into the same IV site. c. Once chemotherapy is in a patient's system, any additional medicine given will cause a synergistic effect. d. Chemotherapy treatments require a special pump designed solely for chemotherapy.

A Chemotherapy is irritating to the vascular system and may cause the vein to infiltrate. Chemotherapy medications and vancomycin are irritating to the veins and introduce increased risk for infiltration. Infiltration of a chemotherapy medication can lead to extravasation, a serious IV complication. Medications can be placed into the same IV site if they are compatible. Chemotherapy does not cause a synergistic effect with antibiotics. Chemotherapy medications can be infused through the same device as maintenance IV fluids or even antibiotics.

14. Which of these findings if seen in a postoperative patient should the nurse associate with dehiscence? a. Complaint by patient that something has given way b. Protrusion of visceral organs through a wound opening c. Chronic drainage of fluid through the incision site d. Drainage that is odorous and purulent

A Complaint by patient that something has given way occurs is when a wound fails to heal properly and the layers of skin and tissue separate. It involves abdominal surgical wounds and occurs after a sudden strain such as coughing, vomiting, or sitting up in bed. Patients often report feeling as though something has given way. Evisceration is seen when vital organs protrude through a wound opening. A fistula is an abnormal passage between two organs or between an organ and the outside of the body that can be characterized by chronic drainage of fluid. Infection is characterized by drainage that is odorous and purulent.

17. The nurse is caring for a patient with a healing stage III pressure ulcer. Upon entering the room, the nurse notices an odor and observes a purulent discharge, along with increased redness at the wound site. What is the next best step for the nurse? a. Complete the head-to-toe assessment, and include current treatment, vital signs, and laboratory results. b. Notify the charge nurse about the change in status and the potential for infection. c. Notify the physician by utilizing Situation, Background, Assessment, and Recommendation (SBAR). d. Notify the wound care nurse about the change in status and the potential for infection.

A Complete the head-to-toe assessment, and include current treatment, vital signs, and laboratory results. The patient is showing signs and symptoms associated with infection in the wound. It is serious and needs treatment but is not a life-threatening emergency, where care is needed immediately or the patient will suffer long-term consequences. The nurse should complete the assessment; gather all data such as current treatment modalities, medications, vital signs including temperature, and laboratory results such as the most recent complete blood count or white cell count. The nurse can then notify the physician and receive treatment orders for the patient. It is important to notify the charge nurse and consult the wound nurse on the patient's status and on any new orders.

1. a nurse in a provider's of ce is preparing to assess a client's skin as part of a comprehensive physical examination. Which of the following ndings should the nurse expect? (select all that apply.) a. capillary re ll less than 2 seconds B. 1+ pitting edema in both feet c. Pale nail beds in both hands d. thick skin on the soles of the feet e. numerous light brown macules on the face

A D E

41. Which nursing action is the number one priority for ensuring that medication stays in the target therapeutic range? a. Drawing the peak and trough levels at the same time each day b. Administering a double dose after a dose was missed c. Delivering the same amount of the drug at the same time each day d. Increasing absorption by holding all other medications 1 hour before administration

A Drawing the peak and trough levels at the same time each day The quantity and distribution of a medication in different body compartments change constantly. Drawing peak and trough levels allows health care providers to see whether the current medication dosage is effective for the patient, or if it needs to be adjusted. Administering a double dose is dangerous and could cause the medication to cross the toxic threshold. Delivering the same amount each day may not be therapeutic or may be toxic for the patient. Holding all other medications should not affect the peak or half-life of the medications, assuming that they are compatible.

3. a nurse educator is reviewing the wound healing process with a group of nurses. the nurse educator should include in the information which of the following alterations for wound healing by secondary intention? (select all that apply.) a. stage III pressure ulcer B. sutured surgical incision c. casted bone fracture d. laceration sealed with adhesive e. open burn area

A E

21. A patient needs assistance excreting a gaseous medication. What is the correct nursing action? a. Encourage the patient to cough and deep-breathe. b. Suction the patient's respiratory secretions. c. Administer the antidote via inhalation. d. Administer 100% FiO2 via simple face mask.

A Encourage the patient to cough and deep-breathe Gaseous and volatile medications are excreted through gas exchange. Deep breathing and coughing will assist in clearing the medication more quickly.

10. The nurse knows that the purpose of aspiration on IM injections is to a. Ensure proper placement of the needle. b. Increase the force of the injection. c. Reduce the discomfort of the injection. d. Prolong the absorption time of the medication.

A Ensure proper placement of the needle. The purpose of aspiration is to ensure that the needle is in the belly of the muscle and not in the vascular system. Blood return upon aspiration indicates improper placement, and the injection should not be given. Increasing the force of the injection, reducing discomfort, and prolonging absorption time are not reasons for aspirating medications.

29. The nurse knows that a subcutaneous injection takes longer to absorb because a. Fewer blood vessels are found under the subcutaneous level. b. Adipose tissue takes longer to metabolize medication. c. Connective tissue holds medication in place longer. d. Some medication leaks out after instillation.

A Fewer blood vessels are found under the subcutaneous level. How quickly a medication is absorbed is dependent on blood flow to the site. Locations with less blood supply take longer to absorb. Absorption is not based on adipose tissue; however, excessive adipose tissue may cause the medication to take longer before reaching the blood supply. The connective tissue is not part of medication absorption. If a medication is properly administered, none of it should be wasted.

36. The nurse is caring for a patient who has suffered a stroke and has residual mobility problems. The patient is at risk for skin impairment. Which initial interventions should the nurse select to decrease this risk? a. Gentle cleaners and thorough drying of the skin b. Absorbent pads and garments c. Positioning with use of pillows d. Therapeutic beds and mattresses

A Gentle cleaners and thorough drying of the skin Assessment and skin hygiene are two initial defenses for preventing skin breakdown. Avoid soaps and hot water when cleansing the skin. Use gentle cleansers with nonionic surfactants. After bathing, make sure to dry the skin completely, and apply moisturizer to keep the epidermis well lubricated. Absorbent pads and garments are controversial and should be considered only when other alternatives have been exhausted. Positioning the patient reduces pressure and shearing force to the skin and is part of the plan of care but is not one of the initial components. Depending on the needs of the patient, a specialty bed may be needed, but again, this does not provide the initial defense for skin breakdown.

39. The nurse is staffing a medical-surgical unit that is assigned most of the patients with pressure ulcers. The nurse has become competent in the care of pressure wounds and recognizes that a staged pressure ulcer that does not require a dressing is stage a. I. b. II. c. III. d. IV.

A I. Stage I intact pressure ulcers that resolve slowly without epidermal loss over 7 to 14 days do not require a dressing. This allows visual inspection and monitoring. A transparent dressing could be used to protect the patient from shear but cannot be used in the presence of excessive moisture. A composite film, hydrocolloid, or hydrogel can be utilized on a clean stage II. A hydrocolloid, hydrogel covered with foam, calcium alginate, gauze, and growth factors can be utilized with a clean stage III. Hydrogel, calcium alginate, gauze, and growth factors can be utilized with a clean stage IV. An unstageable wound cover with eschar should utilize a dressing of adherent film or gauze with an ordered solution of enzymes. In rare cases when eschar is dry and intact, no dressing is used, but this is an unstaged ulcer.

38. A provider has ordered a STAT medication to be administered. The nurse knows that the best route of administration is a. IV. b. IM. c. SQ. d. PO.

A IV IV medications have the quickest effect because they receive the most blood flow. A STAT order is to be carried out as quickly as possible, so the effect should be as immediate as possible. Oral, subcutaneous (SQ), and intramuscular (IM) are other ways to deliver medication but with less blood flow.

30. The nurse has collected the following assessment data: right heel with reddened area that does not blanch. What nursing diagnosis would the nurse assign? a. Ineffective tissue perfusion b. Risk for infection c. Imbalanced nutrition: less than body requirements d. Acute pain

A Ineffective tissue perfusion The area on the heel has experienced a decreased supply of blood and oxygen (tissue perfusion), which has resulted in tissue damage. The most appropriate nursing diagnosis with this information is Ineffective tissue perfusion. Risk for infection, Acute pain, and Imbalanced nutrition may be part of this patient's nursing diagnosis, but the data provided do not support this nursing diagnosis.

20. A patient presents to the emergency department with a laceration of the right forearm caused by a fall. After determining that the patient is stable, the next best step is to a. Inspect the wound for bleeding. b. Inspect the wound for foreign bodies. c. Determine the size of the wound. d. Determine the need for a tetanus antitoxin injection.

A Inspect the wound for bleeding. After determining that a patient's condition is stable, inspect the wound for bleeding. An abrasion will have limited bleeding, a laceration can bleed more profusely, and a puncture wound bleeds in relation to the size and depth of the wound. Address any bleeding issues. Inspect the wound for foreign bodies; traumatic wounds are dirty and may need to be addressed. Determine the size of the wound. A large open wound may expose bone or tissue and be protected, or the wound may need suturing. When the wound is caused by a dirty penetrating object, determine the need for a tetanus vaccination.

18. A drug requires a low pH to be metabolized. Knowing this, the nurse anticipates that the medication will be administered by which route? a. Oral b. Parenteral c. Buccal d. Inhalation

A Oral An oral medication would pass through to the stomach—an area of low pH. The nurse would question an order for a medication that required an acidic environment to be metabolized. Buccal, inhalation, and parenteral routes provide neutral or alkaline environments.

26. The nurse is completing an assessment of the skin's integrity, which includes a. Pressure points. b. All pulses. c. Breath sounds. d. Bowel sounds.

A Pressure points. The nurse continually assesses the skin for signs of ulcer development. Assessment of tissue pressure damage includes visual and tactile inspection of the skin. Observe pressure points such as bony prominences and areas next to treatments such as a binasal cannula and the nares. Assessment of pulses, breath sounds, and bowel sounds is part of a head-to-toe assessment and could influence the function of the body and ultimately skin integrity; however, this assessment is not a specific part of a skin assessment.

2. The nurse is caring for a patient who was involved in an automobile accident 2 weeks ago. The patient sustained a head injury and is unconscious. The nurse is able to identify that the major element involved in the development of a decubitus ulcer is a. Pressure. b. Resistance. c. Stress. d. Weight.

A Pressure. Pressure is the main element that causes pressure ulcers. Three pressure-related factors contribute to pressure ulcer development: pressure intensity, pressure duration, and tissue tolerance. When the intensity of the pressure exerted on the capillary exceeds 12 to 32 mm Hg, this occludes the vessel, causing ischemic injury to the tissues it normally feeds. High pressure over a short time and low pressure over a long time cause skin breakdown. Resistance (the ability to remain unaltered by the damaging effect of something), stress (worry or anxiety), and weight (individuals of all sizes, shapes, and ages acquire skin breakdown) are not major causes of pressure ulcers.

4. The nurse is providing care to a patient who is bedridden. To prevent fatigue, the nurse raises the height of the bed. The nurse understands that balance is maintained by raising the bed to a. Prevent a shift in the nurse's base of support. b. Narrow the base of support. c. Allow the nurse to bring his or her feet close together. d. Shift the center of gravity further away from the base of support.

A Prevent a shift in the nurse's base of support. Raising the height of the bed when performing a procedure prevents bending too far at the waist and causing a shift in the base of support. Balance is maintained by maintaining proper body alignment and posture through two simple techniques. First, widen the base of support by separating the feet to a comfortable distance. Second, increase balance by bringing the center of gravity closer to the base of support.

3. A 2-year-old child is ordered to have ear irrigation performed daily. The nurse correctly performs the procedure by a. Pulling the auricle down and back to straighten the ear canal. b. Pulling the auricle upward and outward to straighten the ear canal. c. Instilling the irrigation solution by holding the syringe just inside the ear canal. d. Holding the fluid in the canal for 2 to 3 minutes with a cotton swab.

A Pulling the auricle down and back to straighten the ear canal. Children up to 3 years of age should have the auricle pulled down and back, children 4 years of age to adults should have the auricle pulled upward and outward. Irrigation solution should be instilled 1 cm (1/2) above the opening of the ear canal. Irrigation solution should be allowed to drain freely during instillation.

13. The nurse knows to assess for signs of medication toxicity within older adults because of which physiological change? a. Reduced glomerular filtration b. Delayed esophageal clearance c. Decreased gastric peristalsis d. Decreased cognitive function

A Reduced glomerular filtration All of the options are signs of aging; however, the glomerular filtration rate affects metabolism and medication clearance. The buildup of medication can cause toxicity in older patients.

22. A nurse has withdrawn a narcotic from the medication dispenser. Upon checking the drug against the medication administration record, the nurse notices that the narcotic order has expired. What should be the nurse's first action? a. Return the medication to the medication dispenser according to protocol. b. Exit the medication room to call the physician to request a reorder of the narcotic. c. Assess the patient to see if the narcotic is still needed; if so, administer the medication. d. Call the pharmacy and request that the narcotic be removed from the patient profile.

A Return the medication to the medication dispenser according to protocol. The nurse should follow Nurse Practice Acts and safe narcotic administration guidelines by safely returning the medication to the secure medication dispenser. This allows for an accurate narcotic count. The nurse should not leave the medication room with a nonprescribed medication; the physician can be contacted once the medication is replaced safely. The nurse cannot administer a medication when there is no current order; this is a violation of the Six Rights of Medication Administration. The nurse should notify the pharmacist after safely returning the narcotic to the medication dispenser; removing the medication from the patient profile will reduce further medication errors.

35. The nurse is preparing to administer medications to two patients with the same last name. After the administration, the nurse realizes that she did not check the identification of the patient before administering medication. Which of the following actions should the nurse complete first? a. Return to the room to check and assess the patient. b. Administer the antidote to the patient immediately. c. Alert the charge nurse that a medication error has occurred. d. Complete proper documentation of the medication error in the patient's chart.

A Return to the room to check and assess the patient. The nurse's first priority is to establish the safety of the patient by assessing the patient. Second, notify the charge nurse and the physician. Administer antidote if required. Finally, the nurse needs to complete proper documentation.

2. A structural curvature of the spine associated with vertebral rotation is known as a. Scoliosis. b. Osteogenesis. c. Osteomalacia. d. Arthritis.

A Scoliosis. Scoliosis is a structural curvature of the spine associated with vertebral rotation. Osteogenesis imperfecta is an inherited disorder that makes bones porous, short, bowed, and deformed. Osteomalacia is an uncommon metabolic disease characterized by inadequate and delayed mineralization, resulting in compact and spongy bone. Arthritis is an inflammatory joint disease characterized by inflammation or destruction of the synovial membrane and articular cartilage, and by systemic signs of inflammation.

15. A patient has developed a decubitus ulcer. What laboratory data would be important to gather? a. Serum albumin b. Creatine kinase c. Vitamin E d. Potassium

A Serum albumin Normal wound healing requires proper nutrition. Serum proteins are biochemical indicators of malnutrition, and serum albumin is probably the most frequently measured of these parameters. The best measurement of nutritional status is prealbumin because it reflects not only what the patient has ingested, but also what the body has absorbed, digested, and metabolized. Measurement of creatine kinase helps in the diagnosis of myocardial infarcts and has no known role in wound healing. Potassium is a major electrolyte that helps to regulate metabolic activities, cardiac muscle contraction, skeletal and smooth muscle contraction, and transmission and conduction of nerve impulses. Vitamin E is a fat-soluble vitamin that prevents the oxidation of unsaturated fatty acids. It is believed to reduce the risk of coronary artery disease and cancer. Vitamin E has no known role in wound healing.

1. A nurse is caring for an adult patient who has had a minor motor vehicle accident. The health history reveals that the patient is currently in the process of obtaining a divorce. Which of the following actions should the nurse take?(Select all that apply.) a. Agree upon and make time for the patient to talk. b. Use active listening skills and therapeutic touch as appropriate. c. Teach stress reduction strategies. d. Inform patient that stressed individuals are more likely to have accidents. e. Agree to witness telephone conversations with separated husband. f. Refer the patient to the nurse's church marriage counselor.

A, B, C, & D

7. The nurse is caring for a group of medical-surgical patients. The unit has been notified of a fire on an adjacent wing of the hospital. The nurse quickly formulates a plan to keep the patients safe. Which of the following should the nurse implement? (Select all that apply.) a. Close all doors. b. Note evacuation routes. c. Note oxygen shut-offs. d. Await direction from the fire department. e. Evacuate everyone from the building. f. Review "Stop, drop, and roll" with the nursing staff.

A, B, C, & D

4. The nurse suspects the possibility of a bioterrorist attack. Which of the following factors is most likely related to this possibility? (Select all that apply.) a. A rapid increase in patients presenting with fever or respiratory or gastrointestinal symptoms b. Lower rates of symptoms among patients who spend time primarily indoors c. Large number of rapidly fatal cases of patients with presenting symptoms d. Shortage of personal protective equipment available from central supply e. An increase in the number of staff calling in sick for their assigned shift f. Patients with symptoms all coming from one location in the area

A, B, C, & F

1. The nurse is caring for a patient with a stage II pressure ulcer and as the coordinator of care understands the need for a multidisciplinary approach. The nurse evaluates the need for several consults. Which of the following should always be included in the consults? (Select all that apply.) a. Registered dietitian b. Enterostomal and wound care nurse c. Physical therapist d. Case management personnel e. Chaplain f. Pharmacist

A, B, C, D

5. The nurse is caring for a patient who will have both a large abdominal bandage and an abdominal binder. The nurse's responsibilities and activities before applying the bandage and binder include which of the following? (Select all that apply.) a. Inspecting the skin for abrasions and edema b. Covering exposed wounds c. Assessing condition of current dressings d. Assessing the skin at underlying areas for circulatory impairment e. Marking the sites of all abrasions f. Cleansing the area with hydrogen peroxide

A, B, C, D

The nurse is caring for a patient with a wound healing by full-thickness repair. Which phases will the nurse monitor for in this patient? (Select all that apply.) a. Hemostasis b. Maturation c. Inflammatory d. Proliferative e. Reproduction f. Reestablishment of epidermal layers

A, B, C, D

2. When assessing the activity tolerance of a patient, the nurse would evaluate which of the following? (Select all that apply.) a. Skeletal abnormalities b. Emotional factors c. Age d. Pregnancy status e. Race

A, B, C, D Factors influencing activity tolerance include physiological factors such as skeletal abnormalities, emotional factors such as anxiety/depression, developmental factors such as age and gender, and pregnancy status. Race is not a factor because people of all races are faced with similar factors that affect their activity tolerance.

The nurse is completing a skin assessment on a medical-surgical patient. Which nursing assessment questions should be included in a skin integrity assessment? (Select all that apply.) a. "Can you easily change your position?" b. "Do you have sensitivity to heat or cold?" c. "How often do you need to use the toilet?" d. "What medications do you take?" e. "Is movement painful?" f. "Have you ever fallen?"

A, B, C, E

6. The home health nurse is caring for a patient in the home who is using an electrical infusion device. While visiting the patient, the nurse smells smoke and notices an electrical fire started by this device. The nurse uses the fire extinguisher and fights the fire when (Select all that apply.) a. All occupants have left the home. b. Fire department has been called. c. Fire is confined to one room. d. An exit route is available. e. The correct extinguisher is available. f. The nurse thinks she can use the fire extinguisher.

A, B, D, & E

A nurse writes the following outcomes for a patient who has chronic obstructive pulmonary disease to improve activity level: Diastolic blood pressure will remain below 70 mm Hg with systolic below 130 mm Hg. Resting heart rate will range between 65 and 75. The last goal is that the patient will exercise 3 times a week. Which evaluative findings indicate successful goal achievement? (Select all that apply.) a. Resting heart rate 70 b. Blood pressure 126/64 c. Blood pressure 140/90 d. Reports doing stretching and flexibility exercises 2 times this week e. Reports doing resistive training 1 time and aerobics 2 times this week

A, B, E

The nurse is caring for a patient who has had a recent stroke and is paralyzed on the left side. The patient has no respiratory or cardiac issues but cannot walk. The patient cannot button a shirt and cannot feed self due to being left-handed and becomes frustrated very easily. The patient has been eating very little and has lost 2 lbs. The patient asks the nurse, "How can I go home like this? I'm not getting better." Which health care team members will the nurse need to consult? (Select all that apply.) a. Dietitian b. Physical therapist c. Respiratory therapist d. Cardiac rehabilitation therapist e. Occupational therapist f. Psychologist

A, B, E, F

1. Bones perform five functions in the body: support, protection, movement, mineral storage, and hematopoiesis. In the discussion of body mechanics, which are the most important? (Select all that apply.) a. Support b. Protection c. Movement d. Mineral storage e. Hematopoiesis

A, C Bones perform five functions in the body: support, protection, movement, mineral storage, and hematopoiesis. In the discussion of body mechanics, two of these functions—support and movement—are most important. In support, bones serve as the framework and contribute to the shape, alignment, and positioning of body parts. In movement, bones together with their joints constitute levers for muscle attachment. As muscles contract and shorten, they pull on bones, producing joint movement. Protection involves encasing the soft tissue organs in a protective cage. Mineral storage helps to strengthen bones but also helps regulate blood levels of certain nutrients. Hematopoiesis is the formation of blood cells.

A home health nurse is assessing the home for fire safety. Which information from the family will cause the nurse to intervene? (Select all that apply.) a. Smoking in bed helps me relax and fall asleep. b. We never leave candles burning when we are gone. c. We use the same space heater my grandparents used. d. We use the RACE method when using the fire extinguisher. e. There is a fire extinguisher in the kitchen and garage workshop.

A, C, D

The nurse is caring for a patient with impaired physical mobility. Which potential complications will the nurse monitor for in this patient? (Select all that apply.) a. Footdrop b. Somnolence c. Hypostatic pneumonia d. Impaired skin integrity e. Increased socialization

A, C, D

2. The nurse is caring for an older adult who presents to the clinic after a fall. The nurse reviews fall prevention in the home. Which of the following should the patient avoid? (Select all that apply.) a. Watering outdoor plants with a nozzle and hose b. Purchasing light bulbs with strength greater than 60 watts c. Missing yearly eye examinations d. Using bathtubs without safety strips e. Unsecured rugs throughout the home f. Walking to the mailbox in the summer

A, C, D, & E

The nurse is caring for a patient who will have a large abdominal bandage secured with an abdominal binder. Which actions will the nurse take before applying the bandage and binder? (Select all that apply.) a. Cover exposed wounds. b. Mark the sites of all abrasions. c. Assess the condition of current dressings. d. Inspect the skin for abrasions and edema. e. Cleanse the area with hydrogen peroxide. f. Assess the skin at underlying areas for circulatory impairment.

A, C, D, F

The nurse is caring for a patient with a surgical incision that eviscerates. Which actions will the nurse take? (Select all that apply.) a. Place moist sterile gauze over the site. b. Gently place the organs back. c. Contact the surgical team. d. Offer a glass of water. e. Monitor for shock.

A, C, E

A nurse is working in a facility that uses no-lift policies. Which benefits will the nurse observe in the facility? (Select all that apply.) a. Reduced number of work-related injuries b. Increased musculoskeletal accidents c. Reduced safety of patients d. Improved health of nurses e. Increased indirect costs

A, D

Which methods will the nurse use to administer an intravenous (IV) medication that is incompatible with the patient's IV fluid? (Select all that apply.) a. Start another IV site. b. Administer slowly with the IV fluid. c. Do not give the medication and chart. d. Flush with 10 mL of sterile water before and after administration. e. Flush with 10 mL of normal saline before and after administration.

A, D, E

Upon assessment a nurse discovers that a patient has erythema. Which actions will the nurse take? (Select all that apply.) a. Consult a dietitian. b. Increase fiber in the diet. c. Place on chest physiotherapy. d. Increase frequency of turning. e. Place on pressure-relieving mattress.

A,D,E

Match the intervention for promoting child safety on the left with the correct developmental stage on the right. A. School-age child B. Preschooler 1. Teach children proper bicycle and skate board safety. 2. Teach children how to cross streets and walk in parking lot. 3. Teach children proper techniques for specific sports. 4. Teach children not to operate electric toothbrushes while unsupervised. 5. Teach children not to talk to or go with a stranger. 6. Teach children not to eat items found in the grass.

A: 1, 2, 3 B: 4, 5, 6

Match the threats to safety on the right to the category of risk factors on the left. A. Individual Risks B. Developmental Risks 1. An older adult has limited finances. 2. A young toddler likes to explore objects by placing them in his mouth. 3. A 55-year-old patient has a residual gait change due to a stroke. 4. A school-age child chooses to play ice hockey. 5. A patient newly diagnosed with diabetes has low health literacy.

A: 1,3,5 B: 2,4

A nurse is assessing activity tolerance of a patient. Which areas will the nurse assess? (Select all that apply.) a. Skeletal abnormalities b. Emotional factors c. Pregnancy status d. Race e. Age

ABCE

45. The physician orders 4 mg of oxycodone to be delivered every 6 hours. After 4 hours, the patient is complaining that she is in more pain. The nurse advises the physician to make which medication adjustment? a. Add an additional narcotic on top of the oxycodone. b. Divide the dose in half and administer 2 mg every 3 hours. c. Give another 4 mg of oxycodone after 4 hours. d. Change the medication being administered for pain relief.

B The patient's metabolism causes the peak effect to occur when the medicine is at its highest concentration. After reaching its peak, the serum concentration of the medication falls progressively. By spreading out the dose, the patient should receive constant pain relief. Changing the medication, increasing the dose, or adding another medication is not the best course of action.

39. A nurse is attempting to administer medication to a child, but the child refuses to take the medication. The nurse asks for the parent's cooperation by saying a. "Please hold your child's arms down at her sides, so I can get the full dose of medication into her mouth." b. "I will prepare the medication for you and observe if you would like to try to administer the medication." c. "Let's turn the lights off and give the child a moment to fall asleep before administering the medication." d. "Since your child loves applesauce, let's add the medication to it, so your child doesn't resist."

B "I will prepare the medication for you and observe if you would like to try to administer the medication." Children often have difficulties taking medication, but it is less traumatic for the child if the parent administers the medication. Holding down the child is not the best option because it may further upset the child. Never administer an oral medication to a sleeping child. Don't mix medications into the child's favorite foods, because the child might start to refuse the food.

37. A patients states that she would prefer not to take her daily allergy pill this morning because it makes her too drowsy throughout the day. The nurse responds therapeutically by saying, a. "The physician ordered it; therefore you must take your medication every morning at the same time whether you're drowsy or not." b. "Let's change the time you take your pill to 9 PM, so the drowsiness occurs when you would normally be sleeping." c. "You can skip this medication on days when you need to be awake and alert." d. "Try to get as much done as you can before you take your pill, so you can sleep in the afternoon."

B "Let's change the time you take your pill to 9 PM, so the drowsiness occurs when you would normally be sleeping." The nurse should use knowledge about the medication to educate the patient about potential response to medications. Then the medication schedule can be altered based on that knowledge, after the physician has been notified. It is the patient's right to refuse her medication; however, the nurse should educate the patient on the importance and effects of her medication. Asking a patient to change her entire life schedule around a medication is unreasonable and will decrease compliance. The nurse should be supportive and should offer solutions to manage medication effects.

2. Which statement by the patient is an indication to use the Z-track method? a. "I'm really afraid that a big needle will hurt." b. "The last shot like that turned my skin colors." c. "I am allergic to many medications." d. "My legs are too obese for the needle to go through."

B "The last shot like that turned my skin colors." The Z-track is indicated when the medication being administered has the potential to irritate sensitive tissues. The Z-track method is not meant to reduce discomfort from the procedure. If a patient is allergic to a medication, it should not be administered. If a patient has additional subcutaneous tissue to go through, a needle of a different size may be selected.

5. A patient has an order to receive 20 units of U-50 insulin. The nurse is using a U-100 syringe. How many units should the nurse draw up in the syringe and administer? a. 0.04 mL b. 0.4 mL c. 4 mL d. 10 mL

B 0.4 mL The nurse is careful to perform nursing calculations to ensure proper medication administration. U-50 insulin has 50 units of insulin in every milliliter, a U-100 syringe has 100 units in every milliliter. Conversion equals 20 units.

6. The patient is to receive phenytoin (Dilantin) at 0900. The nurse knows that the ideal time to draw a trough level is a. 0800. b. 0830. c. 0900. d. 0930.

B 0830 Trough levels are generally drawn 30 minutes before the drug is administered. If the medication is administered at 0900, the trough should be drawn at 0830.

31. Which patient using an inhaler would benefit most from using a spacer? a. 3-year-old with a cleft palate b. 25-year-old with multiple sclerosis c. 50-year-old with hearing impairment d. 72-year-old with left-sided hemiparesis`

B 25-year-old with multiple sclerosis A spacer is indicated for a patient who has limited coordination or function. Individuals with multiple sclerosis often lose motor control and function and have difficulty seeing. Children often have difficulty using a spacer, so a simple face mask is preferred for infants and children younger than 4. Hearing impairment may make teaching the patient to use the inhaler difficult, but it does not indicate the need for a spacer. A patient with one-sided weakness would have a difficult time assembling and administering an inhaler by using a spacer, but the patient could use the inhaler single-handedly.

1. The nurse is working on a medical-surgical unit that has been participating in a research project associated with pressure ulcers. The nurse recognizes that the risk factors that predispose a patient to pressure ulcer development include a. A diet low in calories and fat. b. Alteration in level of consciousness. c. Shortness of breath. d. Muscular pain.

B Alteration in level of consciousness. Patients who are confused or disoriented or who have changing levels of consciousness are unable to protect themselves. The patient may feel the pressure but may not understand what to do to relieve the discomfort or to communicate that he or she is feeling discomfort. Impaired sensory perception, impaired mobility, shear, friction, and moisture are other predisposing factors. Shortness of breath, muscular pain, and a diet low in calories and fat are not included among the predisposing factors.

2. a nurse is assessing an older adult client who has signi cant tenting of the skin over his forearm. Which of the following factors should the nurse consider as a cause for this nding? (select all that apply.) a. thin, parchment‐like skin B. loss of adipose tissue c. dehydration d. diminished skin elasticity e. excessive wrinkling

B C D

4. The nurse is caring for a patient with potential skin breakdown. Which components would the nurse include in the skin assessment? (Select all that apply.) a. Mobility b. Hyperemia c. Induration d. Blanching e. Temperature of skin f. Nutritional status

B C D E

3. a nurse is assessing postoperative circulation of the lower extremities for a client who had knee surgery. the nurse should include which of the following? (select all that apply.) a. range of motion B. skin color c. edema d. skin lesions e. skin temperature

B C E

22. The nurse is caring for a patient who has a wound drain with a collection device. The nurse notices that the collection device has a sudden decrease in drainage. What would be the nurse's next best step? a. Remove the drain; a drain is no longer needed. b. Call the physician; a blockage is present in the tubing. c. Call the charge nurse to look at the drain. d. As long as the evacuator is compressed, do nothing.

B Call the physician; a blockage is present in the tubing. Because a drainage system needs to be patent, look for drainage flow through the tubing, as well as around the tubing. A sudden decrease in drainage through the tubing may indicate a blocked drain, and you will need to notify the physician. The health care provider determines the need for drain removal and removes drains. Notifying the charge nurse, although important for communication, is not the next step in providing care for this patient. The evacuator may be compressed when a blockage is present.

25. The patient is in severe pain and is requesting a prn medication before the prn time interval has elapsed. The nurse's priority is to a. Give the medication early for any pain score greater than 8. b. Call the prescriber and request a stat order. c. Explain to the patient why he will have to wait for the medication. d. Document the patient's request and pain score.

B Call the prescriber and request a stat order. The nurse should utilize clinical judgment to advocate for the patient by requesting a stat order for the patient's breakthrough pain. The nurse cannot give a medication without an order because this violates the "Right Time" portion of the Six Rights of Medication Administration. If a nurse assesses that a patient is in severe pain, she must use clinical judgment to find that patient a means of pain relief. Although the nurse should document the patient's request and pain score, this is not the priority.

20. An order is written for (phenytoin) Dilantin 500 mg IM q3-4h prn for pain. The nurse recognizes that treatment of pain is not a standard therapeutic indication for this drug. The nurse believes that the prescriber meant to write for hydromorphone (Dilaudid). What should the nurse do? a. Give the patient Dilaudid, as it was meant to be written. b. Call the prescriber to clarify and justify the order. c. Administer the medication and monitor the patient frequently. d. Refuse to give the medication and notify the nurse supervisor.

B Call the prescriber to clarify and justify the order. If the nurse is apprehensive about the drug, dose, route, or reason for a medication, the nurse should first call the prescriber and clarify. The nurse should not change the order without the prescriber's consent. Ultimately, the nurse can be held responsible for administering an incorrect medication. If the prescriber is unwilling to change the order and does not justify the order in a reasonable and evidence-based manner, the nurse may refuse to give the medication and notify her supervisor

35. The home health nurse is caring for a patient with impaired skin integrity in the home. The nurse is reviewing dressing changes with the caregiver. Which intervention assists in managing the expenses associated with long-term wound care? a. Sterile technique b. Clean dressings and no touch technique c. Double bagging of contaminated dressings d. Ability of the caregiver

B Clean dressings and no touch technique Clean dressings as opposed to sterile dressings are recommended for home use. This recommendation is in keeping with principles regarding nosocomial infection, and it takes into account the expense of sterile dressings and the dexterity required for application. The caregiver can use the same no touch technique for dressing changes that is used for changing surface dressings without touching the wound or the surface that might come in contact with the wound. Double bagging is required for the disposal of contaminated dressings. The dressings go in a bag, which is fastened and then placed in the household trash. The ability of the caregiver certainly is a component of the success of home treatment, but it does not influence the cost of supplies.

6. The nurse is updating the plan of care for a patient with a stage III pressure ulcer and a nursing diagnosis of Impaired skin integrity. Which of the following outcomes when met indicate progression toward goals? (Select all that apply.) a. Ask whether patient's expectations are being met. b. Prevent injury to the skin and tissues. c. Obtain the patient's perception of interventions. d. Reduce injury to the skin. e. Reduce injury to the underlying tissues. f. Restore skin integrity.

B D E F

27. A confused patient refuses his medication. What is the nurse's first response? a. Agrees with the patient's decision and documents it in his chart b. Educates the patient about the importance of the medication c. Discreetly hides the medication in the patient's favorite Jell-O d. Informs the patient that he must take the medication whether he wants to or not

B Educates the patient about the importance of the medication Much of a patient's apprehension about medication comes from lack of understanding, and educating the patient may lead to better compliance. Ultimately, the patient does have the right to refuse the medication; however, the nurse should first try to educate the patient. Hiding, deceiving, or forcing a patient into taking a medication is unethical and violates his right to autonomy.

40. The nurse is caring for a patient with a wound. The patient appears anxious as the nurse is preparing to change the dressing. What should the nurse do to decrease the patient's anxiety? a. Tell the patient to close his eyes. b. Explain the procedure. c. Turn on the television. d. Ask the family to leave the room.

B Explain the procedure. Explaining the procedure educates the patient regarding the dressing change and involves him in his care, thereby allowing the patient some control in decreasing anxiety. Telling the patient to close his eyes and turning on the television are distractions that do not usually decrease a patient's anxiety. If the family is a support system, asking support systems to leave the room can actually increase a patient's anxiety.

19. The nurse knows that an idiosyncratic event with the stimulant pseudoephedrine (Sudafed) is occurring when the patient a. Experiences blurred vision while driving. b. Falls asleep during daily activities. c. Presents with a pruritus rash. d. Develops xerostomia.

B Falls asleep during daily activities. An idiosyncratic event is a reaction opposite to what the side effects of the medication normally are, or the patient overreacts or underreacts to the medication. Blurred vision is a toxic effect. A rash could indicate an allergic reaction. Dry mouth is a typical response to a stimulant.

24. The nurse is ambulating a patient in the hall when she notices that he is beginning to fall. The nurse should a. Grab the patient and hold him tight to prevent the fall. b. Gently lower the patient to the floor. c. Jump back and let the patient fall naturally. d. Push the patient against the wall and guide him to the floor.

B Gently lower the patient to the floor. If the patient has a fainting episode or begins to fall, assume a wide base of support with one foot in front of the other, thus supporting the patient's body weight. Then extend one leg and let the patient slide against the leg, and gently lower the patient to the floor, protecting the patient's head. Grabbing the patient will shift the nurse's center of gravity and may lead to a back injury. Allowing the patient to fall could lead to head injury for the patient. Pushing the patient against the wall could also cause the patient to hit his head and cause injury.

5. The nurse is admitting an older patient from a nursing home. During the assessment, the nurse notes a shallow open ulcer without slough on the right heel of the patient. This pressure ulcer would be staged as stage a. I. b. II. c. III. d. IV.

B II. This would be a stage II pressure ulcer because it presents as partial-thickness skin loss involving epidermis, dermis, or both. The ulcer is superficial and presents clinically as an abrasion, blister, or shallow crater. Stage I is intact skin with nonblanchable redness over a bony prominence. With a Stage III pressure ulcer, subcutaneous fat may be visible, but bone, tendon, and muscles are not exposed. Stage IV involves full-thickness tissue loss with exposed bone, tendon, or muscle.

26. The nurse is developing a plan of care for a patient diagnosed with activity intolerance. Of the following strategies, which has the best chance of maintaining patient compliance? a. Performing 20 minutes of aerobic exercise daily with 10 minute warm-up and cool-down periods b. Instructing the patient to use an exercise log to record day, time, duration, and responses to exercise activity c. Instructing the patient on the evils of not exercising, and getting her to take responsibility for her current health status d. Arranging for the patient to join a gym that she will have to pay, for so that she does not need to depend on insurance.

B Instructing the patient to use an exercise log to record day, time, duration, and responses to exercise activity Keeping a log may increase adherence to an exercise prescription. Cross-training (combination of exercise activities) provides variety to combat boredom and increases the potential for total body conditioning as opposed to daily aerobic exercise. "Blaming" a patient for his or her health status is usually counterproductive. Instead, the nurse should instruct the patient about the physiological benefits of a regular exercise program. Developing a plan of exercise that the patient may perform at home may improve compliance

18. The nurse is examining a patient who is admitted to the emergency department with severe elbow pain. Of the following situations, which would cause the nurse to suspect a ligament tear or joint fracture? a. Range of motion of the elbow is limited. b. Joint motion is greater than normal. c. The patient has arthritis. d. The elbow cannot be moved (frozen).

B Joint motion is greater than normal. Increased mobility (beyond normal) of a joint may indicate connective tissue disorders, ligament tears, or possible joint fractures. Limited range of motion often indicates inflammation such as arthritis, fluid in the joint, altered nerve supply, or contractures (frozen joints).

22. The patient is admitted with a stroke. The outcome of this disorder is uncertain, but the patient is unable to move his right arm and leg. The nurse understands that a. Active range of motion is the only thing that will prevent contractures from forming. b. Passive range of motion must be instituted to help prevent contracture formation. c. Range-of-motion exercises should be started 2 days after the patient is stable. d. Range-of-motion exercises should be done on major joints only.

B Passive range of motion must be instituted to help prevent contracture formation. When patients cannot participate in active range of motion, the nurse must institute passive range of motion to maintain joint mobility and prevent contractures. Passive range of motion can be substituted for active when needed. For the patient who does not have voluntary motor control, passive range-of-motion exercises are the exercises of choice. Unless contraindicated, the nursing care plan includes exercising each joint (not just major joints) through as nearly a full range of motion as possible. Initiate passive range-of-motion exercises as soon as the patient loses the ability to move the extremity or joint.

15. The nurse is developing an exercise plan for someone diagnosed with congestive heart failure and exercise intolerance. In doing so, the nurse should a. Plan for 20 minutes of continuous aerobic activity and increase as tolerated. b. Perform 6-minute walks at the patient's pace at least 2 times a day. c. Instruct the patient that he should not take his beta blocker medication on exercise days. d. Encourage a high-calorie diet to plan for extra calorie expenditure.

B Perform 6-minute walks at the patient's pace at least 2 times a day. For the diagnosis of exercise intolerance, the patient should begin by performing 6-minute walks at his own pace at least twice a day. The patient would not be able to tolerate 20 minutes of continuous aerobic activity. Patients should be instructed to take medications as ordered. Low-calorie, low-sodium, and high-protein diets are best for this type of patient.

13. The patient is eager to begin his exercise program with a 2-mile jog. The nurse instructs the patient to warm up with stretching exercises. The patient states that he is ready and does not want to waste time with a "warm-up." The nurse explains that the warm-up a. Allows the body to readjust gradually to baseline functioning. b. Prepares the body and decreases the potential for injury. c. Should not involve stretching exercises because they can lead to injury. d. Should be performed with high intensity to prepare for the coming challenge.

B Prepares the body and decreases the potential for injury. The warm-up activity prepares the body for activity and decreases the potential for injury. It usually lasts about 5 to 10 minutes and may include stretching, calisthenics, and/or aerobic activity performed at a lower intensity. The cool-down period allows the body to readjust gradually to baseline functioning and provides an opportunity to combine movement such as stretching with relaxation-enhancing mind-body awareness.

28. The nurse is caring for a medical-surgical patient. To decrease the risk of pressure ulcers and encourage the patient's willingness and ability to increase mobility, which intervention is most important for the nurse to complete? a. Encourage the patient to sit up in the chair. b. Provide analgesic medication as ordered. c. Explain the risks of immobility to the patient. d. Turn the patient every 3 hours while in bed.

B Provide analgesic medication as ordered. Maintaining adequate pain control and patient comfort increases the patient's willingness and ability to increase mobility, which in turn reduces pressure ulcer risks. It is good to encourage a patient to move about but even better if the patient actually sits up in the chair. Explaining the risk of immobility is important for the patient because it may impact the patient's willingness but not his or her ability. Turning the patient is important for decreasing pressure ulcers but needs to be done every 2 hours, and again does not influence the patient's ability to increase mobility.

21. The nurse is caring for a patient on the medical-surgical unit with a wound that has a drain and a dressing that needs changing. Which of these actions should the nurse take first? a. Don sterile gloves. b. Provide analgesic medications as ordered. c. Avoid accidentally removing the drain. d. Gather supplies.

B Provide analgesic medications as ordered. Because removal of dressings is painful, if often helps to give an analgesic at least 30 minutes before exposing a wound and changing the dressing. The next sequence of events includes gathering supplies for the dressing change, donning gloves, and avoiding the accidental removal of the drain during the procedure.

32. The medical-surgical acute care patient has received a nursing diagnosis of Impaired skin integrity. The nurse consults a a. Respiratory therapist. b. Registered dietitian. c. Chaplain. d. Case manager.

B Registered dietitian. Assessment and a plan for the patient to optimize the diet are essential. Adequate calories, protein, vitamins, and minerals promote wound healing. The nurse is the coordinator of care, and collaborating with the dietitian would result in planning the best meals for the patient. The respiratory therapist can be consulted when a patient has issues with the respiratory system. Case management can be consulted when the patient has a discharge need. A chaplain can be consulted when the patient has a spiritual need.

11. The nurse is caring for a patient in the burn unit. The nurse recalls that this type of wound heals by a. Tertiary intention. b. Secondary intention. c. Partial-thickness repair. d. Primary intention.

B Secondary intention. A wound involving loss of tissue such as a burn or a pressure ulcer or laceration heals by secondary intention. The wound is left open until it becomes filled with scar tissue. It takes longer for a wound to heal by secondary intention; thus the chance of infection is greater. A clean surgical incision is an example of a wound with little loss of tissue that heals by primary intention. The skin edges are approximated or closed, and the risk for infection is low. Partial- thickness repair are done on partial-thickness wounds that are shallow, involving loss of the epidermis and maybe partial loss of the dermis. These wounds heal by regeneration because the epidermis regenerates. Tertiary intention is seen when a wound is left open for several days, and then the wound edges are approximated. Wound closure is delayed until the risk of infection is resolved.

9. In planning a physical activity program for a patient, the nurse must understand that a. Isotonic exercises cause contraction without changing muscle length. b. The best program includes a combination of exercises. c. Isometric contraction involves the movement of body parts. d. Resistive isometric exercises can lead to bone wasting.

B The best program includes a combination of exercises. The best program of physical activity includes a combination of exercises that produce different physiological and psychological benefits. Isotonic exercises cause muscle contractions and changes in muscle length. Isometric exercises involve tightening or tensing of muscles without moving body parts. Resistive isometric exercises help promote muscle strength and provide sufficient stress against bone to promote osteoblastic activity.

3. Which nursing observation would indicate that the patient was at risk for pressure ulcer formation? a. The patient ate two thirds of breakfast. b. The patient has fecal incontinence. c. The patient has a raised red rash on the right shin. d. The patient's capillary refill is less than 2 seconds.

B The patient has fecal incontinence. The presence and duration of moisture on the skin increase the risk of ulcer formation by making it susceptible to injury. Moisture can originate from wound drainage, excessive perspiration, and fecal or urinary incontinence. Bacteria and enzymes in the stool can enhance the opportunity for skin breakdown because the skin is moistened and softened, causing maceration. Eating a balanced diet is important for nutrition, but eating just two thirds of the meal does not indicate that the individual is at risk. A raised red rash on the leg again is a concern and can affect the integrity of the skin, but it is located on the shin, which is not a high-risk area for skin breakdown. Pressure can influence capillary refill, leading to skin breakdown, but this capillary response is within normal limits.

17. The patient is brought to the emergency department with possible injury to his shoulder. To help determine the degree of injury, the nurse should evaluate a. The patient's gait. b. The patient's range of motion. c. Fine motor coordination. d. Activity tolerance.

B The patient's range of motion. Assessing range of motion is one assessment technique used to determine the degree of damage or injury to a joint. Gait is the manner or style of walking. It may have little bearing on the shoulder damage. Assessing fine motor coordination would be beneficial in helping to assess the patient's ability to perform tasks but would not help in evaluating the shoulder. Activity tolerance refers to the type and amount of exercise or activity a person is able to perform. Damage to the shoulder would affect this, but this would not have a direct bearing on the amount of damage done to the shoulder.

11. The nurse is giving an IM injection. Upon aspiration, the nurse notices blood return in the syringe. What should the nurse do? a. Administer the injection at a slower rate. b. Withdraw the needle and prepare the injection again. c. Pull the needle back slightly and inject the medication. d. Give the injection and hold pressure over the site for 3 minutes.

B Withdraw the needle and prepare the injection again. Blood return upon aspiration indicates improper placement, and the injection should not be given. Instead withdraw the needle, dispose of the syringe and needle properly, and prepare the medication again. Administering the medication into a blood vessel could have dangerous adverse effects, and the medication will be absorbed faster than intended owing to increased blood flow. Holding pressure is not an appropriate intervention. Pulling back the needle slightly does not guarantee proper placement of the needle and medication administration.

2. What methods are used to properly discard narcotics? (Select all that apply.) a. Placing the syringe of narcotics in the sharps container b. Washing liquids down the sink c. Flushing tablets down the toilet d. Returning the open tablet to the medication dispenser e. Locking the narcotic in a secure cabinet f. Throwing tablets into the trash

B, C

5. The nurse is completing an admission history on a new home health patient. The patient has been experiencing seizures as the result of a recent brain injury. The nurse diagnoses risk for injury with a goal of keeping the patient safe in the event of a seizure. Which interventions should the nurse utilize for this patient? (Select all that apply.) a. Teach the family how to insert an oral airway during the seizure. b. Assess the home for items that could harm the patient during a seizure. c. Provide information on how to obtain a Medical Alert bracelet. d. Teach the patient to communicate to the caregiver plans for bathing. e. Discuss with family steps to take if the seizure does not discontinue. f. Demonstrate how to restrain the patient in the event of a seizure.

B, C, D, & E

3. In developing a nursing care plan for increasing activity tolerance in a patient, the nurse should (Select all that apply.) a. Use generalized therapies because they work for everyone. b. Consult with members of the health care team. c. Avoid goals published by the American College of Sports Medicine. d. Involve the patient and the patient's family in designing an exercise plan. e. Consider the patient's ability to increase activity level.

B, D, E

3. Which of the following concepts are important to utilize when evaluating orders for restraints? (Select all that apply.) a. Behaviors that necessitate the use of restraint are part of the nursing plan of care. b. A physician's order is required for restraint and includes a face-to-face evaluation. c. The physician's preference for the format of the order can override agency policy. d. Orders are time limited. Restraints are not ordered prn (as needed). e. It should be specified that restraints are to be removed periodically. f. Restraint orders are time dated and signed by the physician.

B, D, E, & F

1. Which of the following are methods to reduce the risk of needlestick injury? (Select all that apply.) a. Recap the needle after giving an injection. b. Have sharps boxes emptied when three-quarters full. c. Use two hands to dispose of sharps into the disposal. d. Never force a needle into the sharps disposal. e. Clearly mark sharps disposal containers. f. Use needleless devices whenever possible.

B, D, E, F

A nurse is preparing to move a patient who is able to assist. Which principles will the nurse consider when planning for safe patient handling? (Select all that apply.) a. Keep the body's center of gravity high. b. Face the direction of the movement. c. Keep the base of support narrow. d. Use the under-axilla technique. e. Use proper body mechanics. f. Use arms and legs.

B, E, F

1. a nurse is caring for an adolescent client who is 2 days postoperative following an appendectomy and has type I diabetes mellitus. the client is tolerating a regular diet. He has ambulated successfully around the unit with assistance. He requests pain medication every 6 to 8 hr while reporting pain at a 2 on a scale of 0 to 10 after receiving the medication. His incision is approximated and free of redness, with scant serous drainage on the dressing. the nurse should recognize that the client has which of the following risk factors for impaired wound healing? (select all that apply.) a. extremes in age B. Impaired circulation c. Impaired/suppressed immune system d. Malnutrition e. Poor wound care

B. Impaired circulation c. Impaired/suppressed immune system

5. a nurse is performing an integumentary assessment for a group of clients. Which of the following ndings should the nurse recognize as requiring immediate intervention? a. Pallor B. cyanosis c. Jaundice d. erythema

B. cyanosis

19. The nurse is completing an assessment on an individual who has a stage IV pressure ulcer. The wound is odorous, and a drain is currently in place. The nurse determines that the patient is experiencing issues with self-concept when the patient states which of the following? a. "I think I will be ready to go home early next week." b. "I am so weak and tired, I want to feel better." c. "I am ready for my bath and linen change as soon as possible." d. "I am hoping there will be something good for dinner tonight."

C "I am ready for my bath and linen change as soon as possible." The patient's psychological response to any wound is part of the nurse's assessment. Body image changes can influence self-concept. Factors that affect the patient's perception of the wound include the presence of scars, drains, odor from drainage, and temporary or permanent prosthetic devices. The wound is odorous, and a drain is in place. The patient who is asking for a bath and change in linens gives you a clue that he or she may be concerned about the smell in the room. The patient stating that he or she wants to feel better, talking about going home, and caring about what is for dinner could be interpreted as positive statements that indicate progress along the health journey.

9. The nurse is preparing to administer a 0.5-mL rabies vaccine into the deltoid muscle of a patient. Which needle size is best for the procedure? a. 20 gauge ´ 1 1/2 inch b. 23 gauge ´ 1/2 inch c. 25 gauge ´ 5/8 inch d. 27 gauge ´ 3/8 inch

C 25 gauge ´ 5/8 inch For an intramuscular injection into the deltoid, a 25-gauge, 5/8-inch needle is recommended. Needles that are 20 gauge ´ 11/2 inch, 23 gauge ´ 1/2 inch, and 27 gauge ´ 3/8 inch are not appropriately sized for this type of injection.

5. Approximately what percentage of all back pain is associated with manual lifting tasks? a. 10% b. 25% c. 50% d. 75%

C 50% Half of all back pain is associated with manual lifting tasks.

28. A patient who is being discharged today is going home with an inhaler. The patient is to administer 2 puffs of his inhaler twice daily. The inhaler contains 200 puffs. When should the nurse appropriately advise the patient to refill his medication? a. As soon as he leaves the hospital b. When the inhaler is half empty c. 6 weeks from the start of using the inhaler d. 50 days after discharge

C 6 weeks from the start of using the inhaler The inhaler should last the patient 50 days; the nurse should advise the patient to refill the prescription when he has 7 to 10 days of medication remaining. Refilling it as soon as he leaves the hospital or when the inhaler is half empty is too early. If the patient waits 50 days, the patient will run out of medication before it can be refilled.

33. The nurse is administering an intravenous medication that is to be administered over 10 minutes. Which method should the nurse choose to efficiently administer the medication? a. Place the medication in a large-volume cath-tipped syringe. b. Mix the medication into the patient's maintenance fluids. c. Attach separate tubing and set the medication syringe in a mini-infusion pump. d. Stand at the patient's bedside and carefully watch the clock while pushing the medication.

C Attach separate tubing and set the medication syringe in a mini-infusion pump. To administer this medication efficiently, the nurse should use an infusion pump to run the medication in over a prolonged time. This method is more accurate and is more time efficient than other methods because the nurse can leave the room. The nurse should not mix medication into the maintenance bag without pharmacist and physician approval. Pushing the medication is not a time-efficient method for the nurse. A cath-tipped syringe is an inappropriate device for administration of a medication.

14. Many patients find it difficult to incorporate an exercise program into their daily lives because of time constraints. For these patients, it is beneficial to reinforce that many ADLs are used to accumulate the recommended 30 minutes or more per day of moderate-intensity physical activity. When instructing these patients, the nurse explains that a. Housework is not considered an aerobic exercise. b. To strengthen back muscles, the patient should bend using back muscles. c. Daily chores should begin with gentle stretches. d. The patient should stick to one chore until it is done before beginning a new one.

C Daily chores should begin with gentle stretches. Daily chores should begin with gentle stretches. Housework is considered aerobic exercise. To make it more aerobic, work faster and scrub harder. Bend your legs rather than your back to prevent back injury. Alternate cleaning activities to prevent overworking the same muscle groups.

43. The nurse is caring for a postoperative medial meniscus repair of the right knee. To assist with pain management following the procedure, which intervention should the nurse implement? a. Monitor vital signs every 15 minutes. b. Apply brace to right knee. c. Elevate right knee and apply ice. d. Check pulses in right foot.

C Elevate right knee and apply ice. Ice assists in preventing edema formation, controlling bleeding, and anesthetizing the body part. Elevation assists in preventing edema, which in turn can cause pain. Monitoring vital signs every 15 minutes is routine postoperative care and includes a pain assessment but in itself is not an intervention that decreases pain. Applying a brace provides support and decreases the opportunity for additional trauma, which in turn assists in the healing process. Checking the pulses is important to monitor the circulation of the extremity but in itself is not a pain management intervention.

7. The nurse is caring for a patient with a stage IV pressure ulcer. The nurse recalls that a pressure ulcer takes time to heal and is an example of a. Primary intention. b. Partial-thickness wound repair. c. Full-thickness wound repair. d. Tertiary intention.

C Full-thickness wound repair. Pressure ulcers are full-thickness wounds that extend into the dermis and heal by scar formation because the deeper structures do not regenerate, hence the need for full-thickness repair. The full-thickness repair has three phases: inflammatory, proliferative, and remodeling. A wound heals by primary intention when wounds such as surgical wounds have little tissue loss; the skin edges are approximated or closed, and the risk for infection is low. Partial-thickness repairs are done on partial-thickness wounds that are shallow, involving loss of the epidermis and maybe partial loss of the dermis. These wounds heal by regeneration because the epidermis regenerates. Tertiary intention is seen when a wound is left open for several days, and then the wound edges approximated. Wound closure is delayed until risk of infection is resolved.

4. The wound care nurse visits a patient in the long-term care unit. The nurse is monitoring a patient with a stage III pressure ulcer. The wound seems to be healing, and healthy tissue is observed. How would the nurse stage this ulcer? a. Stage I pressure ulcer b. Healing stage II pressure ulcer c. Healing stage III pressure ulcer d. Stage III pressure ulcer

C Healing stage III pressure ulcer When a pressure ulcer has been staged and is beginning to heal, the ulcer keeps the same stage and is labeled with the words "healing stage." Once an ulcer has been staged, the stage endures even as the ulcer heals. This ulcer was labeled a stage III, it cannot return to a previous stage such as stage I or II. This ulcer is healing, so it is no longer labeled a stage III.

29. The nurse is caring for a patient with a stage IV pressure ulcer. The nurse assigns which of the following nursing diagnoses? a. Readiness for enhanced nutrition b. Impaired physical mobility c. Impaired skin integrity d. Chronic pain

C Impaired skin integrity After the assessment is completed and the information that the patient has a stage IV pressure ulcer is gathered, a diagnosis of Impaired skin integrity is selected. Readiness for enhanced nutrition would be selected for an individual with an adequate diet that could be improved. Impaired physical mobility and Chronic pain, as well as the nutrition nursing diagnosis, could well be the nursing diagnoses selected for this patient, but current data in the question strongly support Impaired skin integrity.

3. Unlike arthritis, joint degeneration a. Occurs only from noninflammatory disease. b. Occurs only from inflammatory disease. c. Involves overgrowth of bone at the articular ends. d. Affects mostly non-weight-bearing joints

C Involves overgrowth of bone at the articular ends. Joint degeneration, which can occur with inflammatory and noninflammatory disease, is marked by changes in articular cartilage combined with overgrowth of bone at the articular ends. Degenerative changes commonly affect weight-bearing joints.

25. The nurse is caring for a patient with a healing stage III pressure ulcer. The wound is clean and granulating. Which of the following orders would the nurse question? a. Use a low-air-loss therapy unit. b. Consult a dietitian. c. Irrigate with hydrogen peroxide. d. Utilize hydrogel dressing.

C Irrigate with hydrogen peroxide. Clean pressure ulcers with noncytotoxic cleansers such as normal saline, which will not kill fibroblasts and healing tissue. Cytotoxic cleansers such as Dakin's solution, acetic acid, povidone-iodine, and hydrogen peroxide can hinder the healing process and should not be utilized on clean granulating wounds. Consulting a dietitian for the nutritional needs of the patient, utilizing a low-air-loss therapy unit to decrease pressure, and applying hydrogel dressings to provide a moist environment for healing are all orders that would be appropriate.

37. The nurse is caring for a patient who is at risk for skin impairment. The patient is able to sit up in a chair. The nurse includes this intervention in the plan of care. How long should the nurse schedule the patient to sit in the chair? a. At least 3 hours b. Not longer than 30 minutes c. Less than 2 hours d. As long as the patient remains comfortable

C Less than 2 hours When patients are able to sit up in a chair, make sure to limit the amount of time to 2 hours or less. The chair sitting time should be individualized. In the sitting position, pressure on the ischial tuberosities is greater than in a supine position. Utilize foam, gel, or an air cushion to distribute weight. Longer than 2 hours can increase the chance of ischemia.

32. The prescriber wrote for a 40-kg child to receive 25 mg of medication 4 times a day. The therapeutic range is 5 to 10 mg/kg/day. What is the nurse's priority? a. Administer the medication because it is within the therapeutic range. b. Notify the physician that the prescribed dose is in the toxic range. c. Notify the physician that the prescribed dose is below the therapeutic range. d. Change the dose to one that is within range.

C Notify the physician that the prescribed dose is below the therapeutic range. The dosage range is 200 to 400 mg a day. The prescribed dose is 100 mg/day, which is below therapeutic range. The nurse should notify the physician first and ask for clarification on the order. The dose is not above the therapeutic range and is not at a toxic level. The nurse should never alter an order without the prescriber's approval and consent.

14. A registered nurse interprets that a scribbled medication order reads 25 mg. The nurse administers 25 mg of the medication to a patient, and then discovers that the dose was incorrectly interpreted and should have been 15 mg. Who is ultimately responsible for the error? a. Physician b. Pharmacist c. Nurse d. No fault

C Nurse Ultimately, the person administering the medication is responsible for ensuring that it is correct. The nurse administered the medication, so in this case it is the nurse. This is the importance of verifying the Six Rights of Medication Administration.

42. Which of the following demonstrates proper oral medication administration? a. Removing the medication from the wrapper and placing it in a cup labeled with the patient's information b. Using the edge of the medicine cup to fill with 0.5 mL of liquid medication c. Placing all of the patient's medications in the same cup, except medications with assessments d. Combining liquid medications from 2 single dose cups into 1 medicine cup

C Placing all of the patient's medications in the same cup, except medications with assessments Placing medications that require preadministration assessment in a separate cup serves as a reminder to check before the medication is given, making it easier for the nurse to withhold medication if necessary. Medications should not be removed from their package until they are in the patient's room because this makes identification of the pill easier and reduces contamination. When measuring a liquid, the nurse should use the meniscus level to measure, not the edge. In addition, liquid medications measuring less than 10 mL should be drawn up in a needleless syringe. Single-dose medications should not be transferred to medicine cups to reduce unnecessary manipulation of the dose.

18. The nurse is collaborating with the dietitian about a patient with a stage III pressure ulcer. After the collaboration, the nurse orders a meal plan that includes increased a. Fat. b. Carbohydrates. c. Protein. d. Vitamin E.

C Protein. Protein needs are especially increased in supporting the activity of wound healing. The physiological processes of wound healing depend on the availability of protein, vitamins (especially A and C), and the trace minerals of zinc and copper. A balanced diet of fat and carbohydrates, along with protein, vitamins, and minerals, is needed in any diet. Wound healing does not require increased amounts of fats or carbohydrates. Vitamin E has no known role in wound healing.

16. Which of the following would be the most important piece of assessment data to gather with regard to wound healing? a. Muscular strength assessment b. Sleep assessment c. Pulse oximetry assessment d. Sensation assessment

C Pulse oximetry assessment Oxygen fuels the cellular functions essential to the healing process; the ability to perfuse tissues with adequate amounts of oxygenated blood is critical in wound healing. Blood flow through the pulmonary capillaries provides red blood cells for oxygen attachment. Oxygen diffuses from the alveoli into the pulmonary blood; most of the oxygen attaches to hemoglobin molecules within the red blood cells. Red blood cells carry oxygenated hemoglobin molecules through the left side of the heart and out to the peripheral capillaries, where the oxygen detaches, depending on the needs of the tissues. Pulse oximetry measures the oxygen saturation of blood. Assessment of muscular strength and sensation, although useful for fitness and mobility testing, does not provide any data with regard to wound healing. Sleep, although important for rest and for integration of learning and restoration of cognitive function, does not provide any data with regard to wound healing.

33. The nurse is caring for a patient with a stage II pressure ulcer and has assigned a nursing diagnosis of Risk for infection. The patient is unconscious and bedridden. The nurse is completing the plan of care and is writing goals for the patient. What is the best goal for this patient? a. The patient's family will demonstrate specific care of the wound site. b. The patient will state what to look for with regard to an infection. c. The patient will remain free of an increase in temperature and of odorous or purulent drainage from the wound. d. The patient's family members will wash their hands when visiting the patient.

C The patient will remain free of an increase in temperature and of odorous or purulent drainage from the wound. Because the patient has an open wound and the skin is no longer intact to protect the tissue, the patient is at increased risk for infection. The nurse will be assessing the patient for signs and symptoms of infection, including an increase in temperature, an increase in white count, and odorous and purulent drainage from the wound. The patient is unconscious and is unable to communicate the signs and symptoms of infection; also, this is an intervention, not a goal for this diagnosis. It is important for the patient's family to be able to demonstrate how to care for the wound and wash their hands, but these statements are interventions, not goals or outcomes for this nursing diagnosis.

1. The coordinated efforts of the musculoskeletal and nervous system maintain balance, posture, and body alignment. Body alignment refers to a. A low center of gravity balanced over a wide base of support. b. The result of weight, center of gravity, and balance. c. The relationship of one body part to another. d. The force that occurs in a direction to oppose movement.

C The relationship of one body part to another. Body alignment refers to the relationship of one body part to another body part along a horizontal or vertical line. Body balance occurs when a relatively low center of gravity is balanced over a wide, stable base of support. Coordinated body movement is a result of weight, center of gravity, and balance. Friction is a force that occurs in a direction to oppose movement.

34. The nurse is caring for a postpartum patient. The patient has an episiotomy after experiencing birth. The physician has ordered heat to treat this condition, and the nurse is providing this treatment. This patient is at risk for a. Infection. b. Impaired skin integrity. c. Trauma. d. Imbalanced nutrition.

C Trauma. Heat causes vasodilatation and is used to improve blood flow to an injured body part. The application of heat incorrectly when the treatment is too hot, or is applied too long or to the wrong place, can result in a burn for the patient and risk for additional trauma. The skin already has impaired integrity owing to the surgical procedure, and because of this has been at risk for infection since the surgical procedure was performed. This patient is of childbearing age and has had a child. Additional needs for nutrition are present during pregnancy and breastfeeding, but this is an established nursing diagnosis. Data are insufficient to support the nursing diagnosis ofImbalanced nutrition.

38. The nurse is caring for a patient who is immobile and is at risk for skin impairment. The plan of care includes turning the patient. What is the best method for repositioning the patient? a. Obtain assistance and use the drawsheet to place the patient into the new position. b. Place the patient in a 30-degree supine position. c. Utilize a transfer sliding board and assistance to slide the patient into the new position. d. Elevate the head of the bed 45 degrees.

C Utilize a transfer sliding board and assistance to slide the patient into the new position. When repositioning the patient, obtain assistance and utilize a transfer sliding board under the patient's body to prevent dragging the patient on bed sheets and placing the patient at high risk for shearing and friction injuries. The patient should be placed in a 30-degree lateral position, not supine position. The head of the bed should be elevated less than 30 degrees to prevent pressure ulcer development from shearing forces.

A nurse is following safety principles to reduce the risk of needlestick injury. Which actions will the nurse take? (Select all that apply.) a. Recap the needle after giving an injection. b. Remove needle and dispose in sharps box. c. Never force needles into the sharps disposal. d. Use clearly marked sharps disposal containers. e. Use needleless devices whenever possible.

C, D, E

1. A nurse knows that patient education has been effective when the patient states a. "I must take my parenteral medication with food." b. "If I am 30 minutes late taking my medication, I should skip that dose." c. "I will rotate the location where I give myself injections." d. "Once I start feeling better, I will stop taking my medication."

C. "I will rotate the location where I give myself injections."

44. A patient informs the nurse that his urine is starting to look discolored. How should the nurse respond? a. "Don't worry, that is a normal side effect of your medication." b. "That is an unusual side effect. I'll notify your provider immediately." c. "You need to drink more fluids to flush the medication from your system." d. "Other than the discoloration, has anything changed with your urination?"

D The nurse wants to gather additional assessment information about the change in urine color. Information is insufficient to recommend drinking more fluids, or to know whether this is a normal or abnormal side effect. The other options do not provide a focused assessment and are not therapeutic responses.

8. A physician orders 1000 mL of normal saline to infuse at a rate of 50 mL/hr. The nurse plans on hanging a new bag at what time? a. 2 hours b. 5 hours c. 10 hours d. 20 hours

D 20 hours It will take 20 hours for a liter (1000 mL) of fluid to infuse at a rate of 50 mL/hr. After 2 hours, only 100 mL would have infused. After 5 hours, only 250 mL would have infused. At 10 hours, 500 mL would have infused.

44. The patient has been provided a nursing diagnosis of Risk for skin impairment and has a 15 on the Braden scale upon admission. The nurse has implemented interventions for this nursing diagnosis. Upon reassessment, which Braden score would be the best sign that the risk for skin breakdown is decreasing? a. 12 b. 13 c. 20 d. 23

D 23 The Braden scale is composed of six subscales: sensory perception, moisture, activity, mobility, nutrition, and friction and shear. The total score ranges from 6 to 23, and a lower total score indicates a higher risk for pressure ulcer development. The cutoff score for onset of pressure ulcer risk with the Braden scale in the general adult population is 18. The best sign is a perfect score of 23.

19. The patient has been bedridden for several months owing to severe congestive heart disease. In determining a plan of care for this patient that will address his activity level, the nurse formulates which of the following nursing diagnoses? a. Fatigue related to poor physical condition b. Impaired gas exchange related to decreased cardiac output c. Decreased cardiac output related to decreased myocardial contractility d. Activity intolerance related to physical deconditioning

D Activity intolerance related to physical deconditioning When activity and exercise are problems for a patient, nursing diagnoses often focus on the individual's ability to move. The diagnostic label directs nursing interventions. In this case, physical deconditioning must be addressed relative to activity level, perhaps leading to 6-minute walks twice a day. Physical deconditioning is the cause of fatigue as well, so it would take priority over that diagnosis. Decreased cardiac output and myocardial contractility are serious concerns that must be addressed before activity intolerance to keep the patient safe and to help determine the level of exercise that the patient can tolerate, but reconditioning of the patient's body will help improve contractility and cardiac output.

11. The nurse is developing an exercise program for elderly patients living in a nursing home. To develop a beneficial health promotion program, the nurse needs to understand that when dealing with the elderly a. Exercise is of very little benefit because the patients are old. b. It is important to disregard their current interests in favor of exercise. c. No physical benefit can be gained without a formal exercise program. d. Adjustments to exercise programs may have to be made to prevent problems.

D Adjustments to exercise programs may have to be made to prevent problems. Exercise is extremely beneficial for older adults, but adjustments to an exercise program may have to be made for those of advanced age to prevent problems. When developing an exercise program for any older adult, consider not only the person's current activity level, range of motion, muscle strength and tone, and response to physical activity, but also the person's interests, capacities, and limitations. Older adults who are unable to participate in a formal exercise program are able to achieve the benefits of improved joint mobility and enhanced circulation by simply stretching and exaggerating movements during performance of routine activities of daily living.

34. A nurse is caring for a patient who is in hypertensive crisis. When the nurse is flushing the patient's peripheral IV, the patient complains of pain. Upon assessment, the nurse notices a red streak that is warm to the touch. What is the nurse's initial action? a. Notify the physician. b. Administer pain medication. c. Discontinue the IV. d. Apply a cool compress to the site.

D Apply a cool compress to the site. The patient has phlebitis; the initial nursing action would be to apply a cool compress. The nurse should start a new IV before discontinuing the old one because it is important to always have an IV access site in case of emergency. Then the physician can be notified. Pain medication may need to be administered.

6. The nurse is preparing to position an immobile patient. Before doing so, the nurse must understand that a. Manual lifting is the easier method and should be tried first. b. Following body mechanics principles alone will prevent back injury. c. Body mechanics can be ignored when patient handling equipment is used. d. Body mechanics alone are not sufficient to prevent injuries.

D Body mechanics alone are not sufficient to prevent injuries. Body mechanics alone are not sufficient to prevent musculoskeletal injuries when positioning or transferring patients. The use of patient-handling equipment in combination with proper body mechanics is more effective than either one in isolation. Body mechanics cannot be ignored even when patient handling equipment is being used. Manual lifting is the last resort, and it is only used when it does not involve lifting most or all of the patient's weight.

15. A patient is to receive medication through a nasogastric tube. What is the most important nursing action to ensure effective absorption? a. Thoroughly shake the medication before administering. b. After all medications are administered, flush tube with 15 to 30 mL of water. c. Position patient in the supine position for 30 minutes. d. Clamp suction for 30 to 60 minutes after medication administration.

D Clamp suction for 30 to 60 minutes after medication administration. Absorption time for a medication administered through a nasogastric (NG) tube is the same as for an oral medication: 30 to 60 minutes. Therefore, the nurse would need to hold the suction for that amount of time to let the medication absorb. Thoroughly shaking the medication mixes the medication before administration but does not affect absorption. Flushing the medications ensures that all were administered. Patients with NG tubes should never be positioned supine but instead should be positioned at a 30- to 90-degree angle to prevent aspiration, provided no contraindication condition is known.

16. Which of the following exercise activities would most likely provide the opportunity for mind-body awareness? a. Warm-up activity b. Resistance training c. Aerobic exercise d. Cool-down activity

D Cool-down activity The cool-down period allows the body to readjust gradually to baseline functioning and provides an opportunity to combine movement such as stretching with relaxation-enhancing mind-body awareness. The warm-up activity prepares the body and decreases the potential for injury. Aerobic exercise includes running, bicycling, jumping rope, and so forth, and is the main portion of exercise activity; it precedes the cool-down period. Resistance training increases muscle strength and endurance and is associated with improved performance of daily activities but not with enhancing mind-body awareness.

23. The patient has been in bed for several days and needs to be ambulated. Before ambulation, the nurse a. Removes the gait belt to allow for unrestricted movement. b. Has the patient get up from bed before he has a chance to get dizzy. c. Has the patient look down to watch his feet to prevent tripping. d. Dangles the patient on the side of the bed.

D Dangles the patient on the side of the bed. Some patients experience orthostatic hypotension—a drop in blood pressure that occurs when the patient changes from a horizontal to a vertical position. Assist the patient to a position of sitting at the side of the bed, and dangle for 1 to 2 minutes before standing. The nurse needs to provide support at the waist so that the patient's center of gravity remains midline. This is achieved with the use of a gait belt. A gait belt encircles the patient's waist and may have handles attached for the nurse to hold while the patient ambulates. The patient should maintain as normal a walking posture as possible with the head erect.

27. The nurse is working with the patient in developing an exercise plan. The patient tells the nurse that she just will not participate in a formal exercise program. The nurse then suggests that exercise activities can be incorporated into activities of daily living. The patient seems to be agreeable to that concept. Of the following activities, which would be considered a moderate-intensity activity? a. Doing laundry b. Making the bed c. Ironing d. Folding clothes

D Folding clothes Low-intensity ADLs include doing the laundry, making the bed, ironing, and washing dishes. Moderate-intensity ADLs include sweeping the kitchen or sidewalk, washing windows, folding clothes, and vacuuming.

6. The nurse is completing a skin assessment on a patient with darkly pigmented skin. Which of the following would be used first to assist in staging an ulcer on this patient? a. Cotton-tipped applicator b. Disposable measuring tape c. Sterile gloves d. Halogen light

D Halogen light When assessing a patient with darkly pigmented skin, proper lighting is essential to accurately complete the first step in assessment—inspection—and the whole assessment process. Natural light or a halogen light is recommended. Fluorescent light sources can produce blue tones on darkly pigmented skin and can interfere with an accurate assessment. Other items that could possibly be used during the assessment include gloves for infection control, a disposable measuring device to measure the size of the wound, and a cotton-tipped applicator to measure the depth of the wound, but these items not the first item used.

26. A patient is at risk for aspiration. What nursing action is most appropriate? a. Hold the patient's cup for him so he can concentrate on taking pills. b. Thin out liquids so they are easier to swallow. c. Give the patient a straw to control the flow of liquids. d. Have the patient self-administer the medication.

D Have the patient self-administer the medication. Aspiration occurs when food, fluid, or medication intended for GI administration inadvertently enters the respiratory tract. To minimize aspiration risk, allow the patient, if capable, to self-administer medication. Patients should also hold their own cup to control how quickly they take in fluid. Liquids should be thickened to reduce the risk of aspiration. Patients at risk for aspiration should not be given straws because use of a straw decreases the control the patient has over volume intake.

17. A patient is in need of immediate pain relief for a severe headache. The nurse knows that which medication will be absorbed the quickest? a. Tylenol 650 mg PO b. Morphine 4 mg SQ c. Ketorolac (Toradol) 8 mg IM d. Hydromorphone (Dilaudid) 4 mg IV

D Hydromorphone (Dilaudid) 4 mg IV IV is the fastest route for absorption owing to the increase in blood flow. Oral, subcutaneous (SQ), and intramuscular (IM) are others ways to deliver medication but with less blood flow.

10. An active lifestyle is important for maintaining and promoting health. In developing an exercise program, the nurse understands that a. Physical exercise is contraindicated for patients with chronic illnesses. b. Regular physical activity is beneficial only for the body part that is exercised. c. Physical exercise has no effect on psychological well-being. d. Physical activity enhances functioning of all body systems.

D Physical activity enhances functioning of all body systems. Regular physical activity and exercise enhance the functioning of all body systems, including cardiopulmonary functioning, musculoskeletal fitness, weight control and maintenance, and psychological well-being. It is also essential in treatment for chronic illness.

10. The nurse is caring for a patient who has experienced a laparoscopic appendectomy. The nurse recalls that this type of wound heals by a. Tertiary intention. b. Secondary intention. c. Partial-thickness repair. d. Primary intention.

D Primary intention. A clean surgical incision is an example of a wound with little loss of tissue that heals with primary intention. The skin edges are approximated or closed, and the risk for infection is low. Partial-thickness repairs are done on partial-thickness wounds that are shallow, involving loss of the epidermis and maybe partial loss of the dermis. These wounds heal by regeneration because the epidermis regenerates. Tertiary intention is seen when a wound is left open for several days, and then the wound edges are approximated. Wound closure is delayed until the risk of infection is resolved. A wound involving loss of tissue such as a burn or a pressure ulcer or laceration heals by secondary intention. The wound is left open until it becomes filled with scar tissue. It takes longer for a wound to heal by secondary intention; thus the chance of infection is greater.

12. Which nursing observation would indicate that a wound healed by secondary intention? a. Minimal scar tissue b. Minimal loss of tissue function c. Permanent dark redness at site d. Scarring can be severe.

D Scarring can be severe. A wound healing by secondary intention takes longer than one healing by primary intention. The wound is left open until it becomes filled with scar tissue. If the scarring is severe, permanent loss of function often occurs. Wounds that heal by primary intention heal quickly with minimal scarring. Scar tissue contains few pigmented cells and has a lighter color than normal skin.

25. In assisting the patient to exercise, the nurse should a. Expect that pain will occur with exercise of unused muscle groups. b. Set the pace for the exercise class. c. Force muscles or joints to go just beyond resistance. d. Stop the exercise if pain is experienced.

D Stop the exercise if pain is experienced. Assess for pain, shortness of breath, or a change in vital signs. If present, stop exercise. Let each patient exercise at his or her own pace. Assess for joint limitations, and do not force a muscle or a joint during exercise.

13. The nurse is caring for a patient who has experienced a total hysterectomy. Which nursing observation would indicate that the patient was experiencing a complication of wound healing? a. The incision site has started to itch. b. The incision site is approximated. c. The patient has pain at the incision site. d. The incision has a mass, bluish in color.

D The incision has a mass, bluish in color. A hematoma is a localized collection of blood underneath the tissues. It appears as swelling, change in color, sensation, or warmth or a mass that often takes on a bluish discoloration. A hematoma near a major artery or vein is dangerous because it can put pressure on the vein or artery and obstruct blood flow. Itching of an incision site can be associated with clipping of hair, dressings, or possibly the healing process. Incisions should be approximated with edges together. After surgery, when nerves in the skin and tissues have been traumatized by the surgical procedure, it is expected that the patient would experience pain.

40. A 64-year-old quadriplegic patient needs an IM injection of antibiotic. What is the best site for the administration? a. Deltoid b. Dorsal gluteal c. Ventrogluteal d. Vastus lateralis

D Vastus lateralis Vastus lateralis is a large muscle that is easily accessible from the supine position. Because this patient no longer walks, the ventrogluteal muscle will begin to atrophy and is not the ideal location. The dorsal gluteal site is a location for a subcutaneous injection, and this patient requires an IM injection. The deltoid is easily accessible, but this muscle is not well developed in many adults.

A nurse is caring for an immobile patient. Which metabolic alteration will the nurse monitor for in this patient? a. Increased appetite b. Increased diarrhea c. Increased metabolic rate d. Altered nutrient metabolism

D. Altered nutrient metabolism **Increased pulse rate if patient is supine

A nurse is preparing a care plan for a patient who is immobile. Which psychosocial aspect will the nurse consider? a. Loss of bone mass b. Loss of strength c. Loss of weight d. Loss of hope

D. Loss of hope

What are never events?

Events that should NEVER happen: falls, foreign objects left in pts, surgical site infections, blood incompatibility

assess a client's likelihood of falling by using: history, gait, metal status etc

MORSE fall scale

Minimizes risk of harm to patients and providers through both system effectiveness and individual performance

QSEN

Minimizes risk of harm to patients and providers through both system effectiveness and individual performance.

QSEN

Anti-hypertensive drug is going to LOWER the BP and thus increase?

a risk for fall

The nurse is teaching a group of older adults at an assisted-living facility about age-related physiological changes affecting safety. Which question would be most important for the nurse to ask this group? a. "Are you able to hear the tornado sirens in your area?" b. "Are you able to read your favorite book?" c. "Are you able to taste spices like before?" d. "Are you able to open a jar of pickles?"

a. "Are you able to hear the tornado sirens in your area?"

14. The nurse is teaching a group of older adults at an assisted-living facility about age-related physiological changes. Which question would be the most important to ask this group? a. "Are you able to hear the tornado sirens in your area?" b. "Are you able to read your favorite book?" c. "Are you able to remember the name of the person you just met?" d. "Are you able to open a jar of pickles?"

a. "Are you able to hear the tornado sirens in your area?" The ability to hear safety alerts and seek shelter is imperative to life safety. Although age-related changes may cause a decrease in sight that affects reading, and although difficulties in remembering short-term information and opening jars as arthritis sets in are important to patients and to those caring for them, being able to hear safety alerts is the priority.

36. The nurse is instructing the student nurse regarding discharge teaching and medications. Which response by the student would indicate that learning has occurred? a. "I need to be precise when teaching a patient about Zyprexa (olanzapine) and Zyrtec (cetirizine)." b. "The medications can be picked up at the pharmacy on the way out of the hospital." c. "I need to be sure to give the patient leftover medications from the medication drawer." d. "I need to remember to teach the patient to take all medications at the same time of the day."

a. "I need to be precise when teaching a patient about Zyprexa (olanzapine) and Zyrtec (cetirizine)." Zyprexa and Zyrtec are sound-alike, look-alike medications. Zyprexa is an antipsychotic and Zyrtec an antihistamine; these agents treat two different conditions. Bringing the differences and similarities in spelling and sound to the attention of the patient is important for patient safety. Medications are not distributed by the hospital, and medications do not need to be administered at the same time each day.

A nurse teaches the patient about the prescribed buccal medication. Which statement by the patient indicates teaching by the nurse is successful? a. "I should let the medication dissolve completely." b. "I will place the medication in the same location." c. "I can only drink water, not juice, with this medication." d. "I better chew my medication first for faster distribution."

a. "I should let the medication dissolve completely."

The nurse is discussing threats to adult safety with a college group. Which statement by a group member indicates understanding of the topic? a. "Smoking just to control stress is not good for my body." b. "Our campus is safe; we leave our dorms unlocked all the time." c. "As long as I have only two drinks, I can still be the designated driver." d. "I am young, so I can work nights and go to school with 2 hours' sleep."

a. "Smoking just to control stress is not good for my body."

A patient has an order to receive 12.5 mg of hydrochlorothiazide. The nurse has on hand a 25 mg tablet of hydrochlorothiazide. How many tablet(s) will the nurse administer? a. 1/2 tablet b. 1 tablet c. 1 1/2 tablets d. 2 tablets

a. 1/2 tablet

The nurse is planning to administer a tuberculin test with a 27-gauge,3/4-inch needle. At which angle will the nurse insert the needle? a. 15 degree b. 30 degree c. 45 degree d. 90 degree

a. 15 degree

The nurse enters the patient's room and notices a small fire in the headlight above the patient's bed. In which order will the nurse perform the steps, beginning with the first one? 1. Pull the alarm. 2. Remove the patient. 3. Use the fire extinguisher. 4. Close doors and windows. a. 2, 1, 4, 3 b. 1, 2, 4, 3 c. 1, 2, 3, 4 d. 2, 1, 3, 4

a. 2, 1, 4, 3

8. The nurse is caring for a patient with a large abrasion from a motorcycle accident. The nurse recalls that if the wound is kept moist, it can resurface in _____ day(s). a. 4 b. 2 c. 1 d. 7

a. 4 A partial-thickness wound repair has three compartments: the inflammatory response, epithelial proliferation and migration, and re-establishment of the epidermal layers. Epithelial proliferation and migration start at all edges of the wound, allowing for quick resurfacing. Epithelial cells begin to migrate across the wound bed soon after the wound occurs. A wound left open to air resurfaces within 6 to 7 days, whereas a wound that is kept moist can resurface in 4 days. One or 2 days is too soon for this process to occur, moist or dry.

A patient who is being discharged today is going home with an inhaler. The patient is to administer 2 puffs of the inhaler twice daily. The inhaler contains 200 puffs. When should the nurse appropriately advise the patient to refill the medication? a. 6 weeks from the start of using the inhaler. b. As soon as the patient leaves the hospital. c. When the inhaler is half empty. d. 50 days after discharge.

a. 6 weeks from the start of using the inhaler.

3. The patient has been diagnosed with a respiratory illness and complains of shortness of breath. The nurse adjusts the temperature to facilitate the comfort of the patient. What is the usual comfort range for most patients? a. 65° F to 75° F b. 60° F to 75° F c. 15° C to 17° C d. 25° C to 28° C

a. 65° F to 75° F The comfort zone for most individuals is the range between 65° F and 75° F (18.3° C to 23.9° C). The other ranges do not reflect the average person's comfort zone.

As prescribed, the nurse leaves the pressure ulcer open to air and does not apply a dressing. Which stage of ulcer did the nurse appropriately treat? a. A Stage I b. A Stage II c. A Stage III d. A Stage IV

a. A Stage I

A nurse is teaching a community group of school-aged parents about safety. The proper fitting of which safety item is most important for the nurse to include in the teaching session? a. A bicycle helmet b. Soccer shin guards c. Swimming goggles d. Baseball sliding shorts

a. A bicycle helmet

The nurse is starting an exercise program in a local community as a health promotion project. Which information will the nurse include in the teaching session? a. A cool-down period lasts about 5 to 10 minutes. b. The purpose of weight training is to bulk up muscles. c. Resistance training is appropriate for warm-up and cool-down periods. d. Aerobic exercise should be done 3 to 5 times per week for about 20 minutes.

a. A cool-down period lasts about 5 to 10 minutes.

A nurse is providing care to a group of patients. Which patient will the nurse see first? a. A patient with a hip replacement on prolonged bed rest reporting chest pain and dyspnea b. A bedridden patient who has a reddened area on the buttocks who needs to be turned c. A patient on bed rest who has renal calculi and needs to go to the bathroom d. A patient after knee surgery who needs range of motion exercises

a. A patient with a hip replacement on prolonged bed rest reporting chest pain and dyspnea

A nurse is supervising the logrolling of a patient. To which patient is the nurse most likely providing care? a. A patient with neck surgery b. A patient with hypostatic pneumonia c. A patient with a total knee replacement d. A patient with a Stage IV pressure ulcer

a. A patient with neck surgery

The patient weighs 450 lbs (204.5 kg) and reports shortness of breath with any exertion. The health care provider has recommended beginning an exercise program. The patient states that she can hardly get out of bed and just cannot do anything around the house. Which nursing diagnosis will the nurse add to the care plan? a. Activity intolerance related to excessive weight b. Impaired physical mobility related to bed rest c. Imbalanced nutrition: less than body requirements d. Impaired gas exchange related to shortness of breath

a. Activity intolerance related to excessive weight

The nurse is administering medications to several patients. Which action should the nurse take? a. Advise a patient to wait 2 minutes after a corticosteroid inhaler treatment to rinse mouth with water. b. Administer an intravenous medication through tubing that is infusing blood. c. Pinch up the deltoid muscle of an adult patient receiving a vaccination. d. Aspirate before administering a subcutaneous injection in the abdomen.

a. Advise a patient to wait 2 minutes after a corticosteroid inhaler treatment to rinse mouth with water.

The nurse is caring for a patient at risk for skin impairment. Which initial action should the nurse take to decrease this risk? a. After cleansing thoroughly dry the skin. b. Request a therapeutic bed and mattress. c. Pad the bed with absorbent pads. d. Use products that retain moisture.

a. After cleansing thoroughly dry the skin.

The nurse is caring for a patient who suddenly becomes confused and tries to remove an intravenous (IV) infusion. Which priority action will the nurse take to minimize the patient's risk for injury? a. Assess the patient. b. Gather restraint supplies. c. Try alternatives to restraint. d. Call the health care provider for a restraint order.

a. Assess the patient.

The patient is being admitted to the neurological unit with a diagnosis of stroke. When will the nurse begin discharge planning? a. At the time of admission b. The day before the patient is to be discharged c. When outpatient therapy will no longer be needed d. As soon as the patient's discharge destination is known

a. At the time of admission

A nurse is preparing to administer an antibiotic medication at 1000 to a patient but gets busy in another room. When should the nurse give the antibiotic medication? a. By 1030 b. By 1100 c. By 1130 d. By 1200

a. By 1030

A prescription is written for phenytoin 500 mg IM q3-4h prn for pain. The nurse recognizes that treatment of pain is not a standard therapeutic indication for this drug. The nurse believes that the health care provider meant to write hydromorphone. What action should the nurse take? a. Call the health care provider to clarify the order. b. Give the patient hydromorphone, as it was meant to be written. c. Administer the medication and monitor the patient frequently. d. Refuse to give the medication and notify the nurse supervisor.

a. Call the health care provider to clarify the order.

The nurse is caring for a patient who has a wound drain with a collection device. The nurse notices that the collection device has a sudden decrease in drainage. Which action will the nurse take initially? a. Call the health care provider; a blockage is present in the tubing. b. Chart the results on the intake and output flow sheet. c. Do nothing, as long as the evacuator is compressed. d. Remove the drain; a drain is no longer needed.

a. Call the health care provider; a blockage is present in the tubing.

Upon entering the room of a patient with a healing Stage III pressure ulcer, the nurse notices an odor and observes a purulent discharge, along with increased redness at the wound site. What action should the nurse give priority to? a. Completing a head-to-toe assessment, including current treatment, vital signs, and laboratory results b. Notifying the health care provider by utilizing Situation, Background, Assessment, and Recommendation (SBAR) c. Consulting the wound care nurse about the change in status and the potential for infection d. Conferring with the charge nurse about the change in status and the potential for infection

a. Completing a head-to-toe assessment, including current treatment, vital signs, and laboratory results

A nurse participating in a research project associated with pressure ulcers will assess for what predisposing factor that tends to increase the risk for pressure ulcer development? a. Decreased level of consciousness b. Adequate dietary intake c. Shortness of breath d. Muscular pain

a. Decreased level of consciousness

6. The nurse is preparing a patient for surgery. The nurse explains that the reason for writing in indelible ink on the surgical site the word "correct" is to a. Distinguish the correct surgical site. b. Label the correct patient. c. Comply with the surgeon's preference. d. Adhere to the correct regulatory standard.

a. Distinguish the correct surgical site. The purpose of writing on the surgical site as part of the Universal Protocol from the Joint Commission is to distinguish the correct site on the correct patient and match with the correct surgeon for patient safety and prevention of wrong site surgery. All patients who are having an invasive procedure should receive labeling in many different ways, including the record and patient armbands. Writing in indelible ink may comply with the surgeon's preference, but safety is the driving factor. Although labeling of the site helps to meet regulatory standards, this is not the reason to do this activity—the reason is to keep the patient safe.

A nurse is caring for a patient who is receiving pain medication through a saline lock. After flushing the patient's peripheral IV and obtaining a good blood return, the patient reports pain. Upon assessment, the nurse notices a red streak that is warm and tender to the touch. What is the nurse's initial action? a. Do not administer the pain medication. b. Administer the pain medication slowly. c. Apply a warm compress to the site. d. Apply a cool compress to the site.

a. Do not administer the pain medication.

A nurse observes a patient rising from a chair slowly by pushing on the chair arms. Which type of tension and contraction did the nurse observe? a. Eccentric tension and isotonic contraction b. Eccentric tension and isometric contraction c. Concentric tension and isotonic contraction d. Concentric tension and isometric contraction

a. Eccentric tension and isotonic contraction

A nurse is teaching the patient and family about wound care. Which technique will the nurse teach to best prevent transmission of pathogens? a. Effective hand hygiene b. Saline wound irrigation c. Appropriate use of gloves d. When eye protection is needed

a. Effective hand hygiene

A patient needs assistance in eliminating an anesthetic gaseous medication (nitrous oxide). Which action will the nurse take? a. Encourage the patient to cough and deep-breathe. b. Suction the patient's respiratory secretions. c. Suggest voiding every 2 hours. d. Increase fluid intake.

a. Encourage the patient to cough and deep-breathe.

The nurse is caring for an older-adult patient who has been diagnosed with a stroke. Which intervention will the nurse add to the care plan? a. Encourage the patient to perform as many self-care activities as possible. b. Provide a complete bed bath to promote patient comfort. c. Coordinate with occupational therapy for gait training. d. Place the patient on bed rest to prevent fatigue.

a. Encourage the patient to perform as many self-care activities as possible.

A nurse is developing an individualized plan of care for a patient. Which action is important for the nurse to take? a. Establish goals that are measurable and realistic. b. Set goals that are a little beyond the capabilities of the patient. c. Use the nurse's own judgment and not be swayed by family desires. d. Explain that without taking alignment risks, there can be no progress.

a. Establish goals that are measurable and realistic.

The nurse administers a central nervous system stimulant to a patient. Which assessment finding indicates to the nurse that an idiosyncratic event is occurring? a. Falls asleep during daily activities. b. Presents with a pruritic rash. c. Develops restlessness. d. Experiences alertness.

a. Falls asleep during daily activities.

Which nursing observation will indicate the patient is at risk for pressure ulcer formation? a. Fecal incontinence b. Ate two thirds of breakfast c. A raised red rash on the right shin d. Capillary refill is less than 2 seconds

a. Fecal incontinence

A nurse has withdrawn a narcotic from the medication dispenser and must waste a portion of the medication. What should the nurse do? a. Have another nurse witness the wasted medication. b. Return the wasted medication to the medication dispenser. c. Place the wasted portion of the medication in the sharps container. d. Exit the medication room to call the health care provider to request an order that matches the dosages.

a. Have another nurse witness the wasted medication.

The nurse is providing teaching to an immobilized patient with impaired skin integrity about diet. Which diet will the nurse recommend? a. High protein, high calorie b. High carbohydrate, low fat c. High vitamin A, high vitamin E d. Fluid restricted, bland

a. High protein, high calorie

A patient has damage to the cerebellum. Which disorder is most important for the nurse to assess? a. Imbalance b. Hemiplegia c. Muscle sprain d. Lower extremity paralysis

a. Imbalance

2. a nurse is collecting data from a client who is 5 days postoperative following abdominal surgery. the surgeon suspects an incisional wound infection and has prescribed antibiotic therapy for the nurse to initiate after collecting wound and blood specimens for culture and sensitivity. Which of the following ndings should the nurse expect? (select all that apply.) a. Increase in incisional pain B. Fever and chills c. reddened wound edges d. Increase in serosanguineous drainage e. decrease in thirst

a. Increase in incisional pain B. Fever and chills c. reddened wound edges gonna be purulent drainage

A patient presents to the emergency department with a laceration of the right forearm caused by a fall. After determining that the patient is stable, what is the next best step for the nurse to take? a. Inspect the wound for bleeding. b. Irrigate the wound to remove foreign bodies. c. Measure and document the size of the wound. d. Determine when the patient last had a tetanus antitoxin injection.

a. Inspect the wound for bleeding.

A patient at risk for skin impairment is able to sit up in a chair. How long should the nurse schedule the patient to sit in the chair? a. Less than 2 hours at any one time b. For a total of least than 3 hours daily c. No longer than 30 minutes out of every hour d. Until the patient expresses being uncomfortable

a. Less than 2 hours at any one time

The nurse is caring for a patient with inner ear problems. Which goal is the priority? a. Maintain balance. b. Maintain proprioception. c. Maintain muscle strength. d. Maintain body alignment.

a. Maintain balance.

A nurse is working in a facility that follows a comprehensive safe patient-handling program. Which finding will alert the nurse to intervene? a. Mechanical lifts are in a locked closet. b. Algorithms for patient handling are available. c. Ergonomic assessment protocols are being followed. d. A no-lift policy is in place with adherence by all staff.

a. Mechanical lifts are in a locked closet.

The nurse is preparing to move a patient to a wheelchair. Which action indicates the nurse is following recommendations for safe patient handling? a. Mentally reviews the transfer steps before beginning. b. Uses own strength to transfer the patient. c. Focuses solely on body mechanics. d. Bases decisions on intuition.

a. Mentally reviews the transfer steps before beginning.

A nurse delegates a position change to a nursing assistive personnel. The nurse instructs the NAP to place the patient in the lateral position. Which finding by the nurse indicates a correct outcome? a. Patient is lying on side. b. Patient is lying on back. c. Patient is lying semiprone. d. Patient is lying on abdomen.

a. Patient is lying on side.

The nurse is caring for a patient who needs to be placed in the prone position. Which action will the nurse take? a. Place pillow under the patient's lower legs. b. Turn head toward one side with large, soft pillow. c. Position legs flat against bed. d. Raise head of bed to 45 degrees.

a. Place pillow under the patient's lower legs.

A home health nurse is assessing a family's home after the birth of an infant. A toddler also lives in the home. Which finding will cause the nurse to follow up? a. Plastic grocery bags are neatly stored under the counter. b. Electric outlets are covered in all rooms. c. No bumper pads are in the crib. d. Crib slats are 5 cm apart.

a. Plastic grocery bags are neatly stored under the counter.

The nurse is completing an assessment of the patient's skin's integrity. Which assessment is the priority? a. Pressure points b. Breath sounds c. Bowel sounds d. Pulse points

a. Pressure points

The nurse is caring for a patient on the medical-surgical unit with a wound that has a drain and a dressing that needs changing. Which action should the nurse take first? a. Provide analgesic medications as ordered. b. Avoid accidentally removing the drain. c. Don sterile gloves. d. Gather supplies.

a. Provide analgesic medications as ordered.

A 2-year-old child is ordered to have eardrops daily. Which action will the nurse take? a. Pull the auricle down and back to straighten the ear canal. b. Pull the auricle upward and outward to straighten the ear canal. c. Sit the child up for 2 to 3 minutes after instilling drops in ear canal. d. Sit the child up to insert the cotton ball into the innermost ear canal.

a. Pull the auricle down and back to straighten the ear canal.

What is the nurse's priority action to protect a patient from medication error? a. Reading medication labels at least 3 times before administering b. Administering as many of the medications as possible at one time c. Asking anxious family members to leave the room before giving a medication d. Checking the patient's room number against the medication administration record

a. Reading medication labels at least 3 times before administering

The nurse closely monitors an older adult for signs of medication toxicity. Which physiological change is the reason for the nurse's action? a. Reduced kidney functioning b. Reduced esophageal stricture c. Increased gastric motility d. Increased liver mass

a. Reduced kidney functioning

The nurse has prepared a pain injection for a patient but was called to check on another patient. When asked to give the medication what action by the new nurse is best? a. Refuse to give the medication. b. Administer the medication just this once. c. Give the medication if the pain score greater than 8. d. Avoid the issue and pretend to not hear the request.

a. Refuse to give the medication.

A patient has an ankle restraint applied. Upon assessment the nurse finds the toes a light blue color. Which action will the nurse take next? a. Remove the restraint. b. Place a blanket over the feet. c. Immediately do a complete head-to-toe neurological assessment. d. Take the patient's blood pressure, pulse, temperature, and respiratory rate.

a. Remove the restraint.

The nurse is preparing to administer medications to two patients with the same last name. After the administration, the nurse realizes that did not check the identification of the patient before administering medication. Which action should the nurse complete first? a. Return to the room to check and assess the patient. b. Administer the antidote to the patient immediately. c. Alert the charge nurse that a medication error has occurred. d. Complete proper documentation of the medication error in the patient's chart.

a. Return to the room to check and assess the patient.

A confused patient is restless and continues to try to remove the oxygen cannula and urinary catheter. What is the priority nursing diagnosis and intervention to implement for this patient? a. Risk for injury: Check on patient every 15 minutes. b. Risk for suffocation: Place "Oxygen in Use" sign on door. c. Disturbed body image: Encourage patient to express concerns about body. d. Deficient knowledge: Explain the purpose of oxygen therapy and the urinary catheter.

a. Risk for injury: Check on patient every 15 minutes.

28. A confused patient is restless and continues to try to remove his oxygen and urinary catheter. What is the priority nursing diagnosis and intervention to implement for this patient? a. Risk for injury: Prevent harm to patient, use restraints if alternatives fail. b. Deficient knowledge: Explain the purpose of oxygen therapy and the urinary catheter. c. Disturbed body image: Encourage patient to express concerns about body. d. Caregiver role strain: Identify resources to assist with care.

a. Risk for injury: Prevent harm to patient, use restraints if alternatives fail. The priority nursing diagnosis is risk for injury. This patient could cause harm to himself by interrupting the oxygen therapy or by damaging the urethra by pulling the urinary catheter out. Before restraining a patient, it is important to implement and exhaust alternatives to restraint. Alternatives can include distraction and providing companionship or supervision. Patients may be moved to a location closer to the nurses' station; trained sitters or family members may be involved. Nurses need to ensure that patients are provided adequate food, liquid, toileting, and relief from pain. If these and other alternatives fail, this individual may need restraints; in this case, an order would need to be obtained for the restraint. This patient may have deficient knowledge; educating the patient about treatments could be considered as an alternative to restraints; however, the nursing diagnosis of highest priority is risk for injury. This scenario does not indicate that the patient has a disturbed body image or that the patient's caregiver is strained.

A nurse notices that a patient has a structural curvature of the spine associated with vertebral rotation. Which condition will the nurse most likely find documented in the patient's medical record? a. Scoliosis b. Arthritis c. Osteomalacia d. Osteogenesis

a. Scoliosis

19. Equipment-related accidents are risks in the health care agency. The nurse assesses for this risk when using a. Sequential compression devices. b. A measuring device that measures urine. c. Computer-based documentation. d. A manual medication-dispensing device.

a. Sequential compression devices. Sequential compression devices are used on a patient's extremities to assist in prevention of deep vein thrombosis and have the potential to malfunction and harm the patient. Measuring devices used by the nurse to measure urine, computer documentation, and manual dispensing devices can break or malfunction but are not used directly on a patient.

While preparing medications, the nurse knows one of the drug is an acidic medication. In which area does the nurse anticipate the drug will be absorbed? a. Stomach b. Mouth c. Small intestine d. Large intestine

a. Stomach

A nurse is teaching a health promotion class about isotonic exercises. Which types of exercises will the nurse give as examples? a. Swimming, jogging, and bicycling b. Tightening or tensing of muscles without moving body parts c. Quadriceps set exercises and contraction of the gluteal muscles d. Push-ups, hip lifting, pushing feet against a footboard on the bed

a. Swimming, jogging, and bicycling

2. The nurse is caring for an elderly patient admitted with nausea, vomiting, and diarrhea. Upon completing the health history, which priority concern would require collaboration with social services to address the patient's health care needs? a. The electricity was turned off 2 days ago. b. The water comes from the county water supply. c. A son and family recently moved into the home. d. The home is not furnished with a microwave oven.

a. The electricity was turned off 2 days ago. Electricity is needed for refrigeration of food, and lack of electricity could have contributed to the nausea, vomiting, and diarrhea—potential food poisoning. This discussion about the patient's electrical needs can be referred to social services. The water supply, the increased number of individuals in the home, and not having a microwave may or may not be concerns but do not pertain to the current health care needs of this patient.

The nurse is caring for an older-adult patient admitted with nausea, vomiting, and diarrhea due to food poisoning. The nurse completes the health history. Which priority concern will require collaboration with social services to address the patient's health care needs? a. The electricity was turned off 3 days ago. b. The water comes from the county water supply. c. A son and family recently moved into the home. d. This home is not furnished with a microwave oven.

a. The electricity was turned off 3 days ago.

A nurse is assessing body alignment. What is the nurse monitoring? a. The relationship of one body part to another while in different positions b. The coordinated efforts of the musculoskeletal and nervous systems c. The force that occurs in a direction to oppose movement d. The inability to move about freely

a. The relationship of one body part to another while in different positions

The patient has been diagnosed with a spinal cord injury and needs to be repositioned using the logrolling technique. Which technique will the nurse use for logrolling? a. Use at least three people. b. Have the patient reach for the opposite side rail when turning. c. Move the top part of the patient's torso and then the bottom part. d. Do not use pillows after turning.

a. Use at least three people.

Which activity will increase the need for the nurse to monitor for equipment-related accidents? a. Using a patient-controlled analgesic pump b. Making an entry in a computer-based documentation record c. Using a plastic measuring device to accurately measure urine d. Removing medications from a manual medication-dispensing device

a. Using a patient-controlled analgesic pump

A nurse is assessing the body alignment of a standing patient. Which finding will the nurse report as normal? a. When observed laterally, the spinal curves align in a reversed "S" pattern. b. When observed posteriorly, the hips and shoulders form an "S" pattern. c. The arms should be crossed over the chest or in the lap. d. The feet should be close together with toes pointed out.

a. When observed laterally, the spinal curves align in a reversed "S" pattern.

11. A nurse is teaching a community group of school-aged parents about safety. The most important item to prioritize and explain is how to check the proper fit of a. a bicycle helmet. b. swimming goggles. c. soccer shin guards. d. baseball sliding shorts.

a. a bicycle helmet. Bicycle-related injuries are a major cause of death and disability among children. Proper fit of the helmet helps to decrease head injuries resulting from bicycle accidents. Goggles, shin guards, and sliding shorts are important sports safety equipment and should fit properly, but they do not protect from this leading cause of death.

A child is brought to the emergency department after swallowing liquid cleanser. He is awake and alert and able to swallow. Which action should the nurse take first? 1) Administer a dose of syrup of ipecac. 2) Administer activated charcoal immediately. 3) Give water to the child immediately. 4) Call the nearest poison control center.

water to child then call poison control

A nurse is attempting to administer an oral medication to a child, but the child refuses to take the medication. A parent is in the room. Which statement by the nurse to the parent is best? a. "Please hold your child's arms down, so I can give the full dose." b. "I will prepare the medication for you and observe if you would like to try to administer the medication." c. "Let's turn the lights off and give your child a moment to fall asleep before administering the medication." d. "Since your child loves applesauce, let's add the medication to it, so your child doesn't resist."

b. "I will prepare the medication for you and observe if you would like to try to administer the medication."

A nurse is teaching a patient about medications. Which statement from the patient indicates teaching is effective? a. "My parenteral medication must be taken with food." b. "I will rotate the sites in my left leg when I give my insulin." c. "Once I start feeling better, I will stop taking my antibiotic." d. "If I am 30 minutes late taking my medication, I should skip that dose."

b. "I will rotate the sites in my left leg when I give my insulin."

A patient prefers not to take the daily allergy pill this morning because it causes drowsiness throughout the day. Which response by the nurse is best? a. "The physician ordered it; therefore, you must take your medication every morning at the same time whether you're drowsy or not." b. "Let's see if we can change the time you take your pill to 9 PM, so the drowsiness occurs when you would normally be sleeping." c. "You can skip this medication on days when you need to be awake and alert." d. "Try to get as much done as you can before you take your pill, so you can sleep in the afternoon."

b. "Let's see if we can change the time you take your pill to 9 PM, so the drowsiness occurs when you would normally be sleeping."

The patient is to receive phenytoin at 0900. When will be the ideal time for the nurse to schedule a trough level? a. 0800 b. 0830 c. 0900 d. 0930

b. 0830

The nurse performing a moist-to-dry dressing has prepared the supplies, solution, and removed the old dressing. In which order will the nurse implement the following steps, starting with the first one? 1. Apply sterile gloves. 2. Cover and secure topper dressing. 3. Assess wound and surrounding skin. 4. Moisten gauze with prescribed solution. 5. Gently wring out excess solution and unfold. 6. Loosely pack until all wound surfaces are in contact with gauze. a. 4, 3, 1, 5, 6, 2 b. 1, 3, 4, 5, 6, 2 c. 4, 1, 3, 5, 6, 2 d. 1, 4, 3, 5, 6, 2

b. 1, 3, 4, 5, 6, 2

The nurse is teaching a patient how to sit with crutches. In which order will the nurse present the instructions starting with the first step? 1. Place both crutches in one hand. 2. Grasp arm of chair with free hand. 3. Completely lower self into chair. 4. Transfer weight to crutches and unaffected leg. a. 4, 1, 2, 3 b. 1, 4, 2, 3 c. 1, 2, 4, 3 d. 4, 2, 1, 3

b. 1, 4, 2, 3

A health care provider orders lorazepam 1 mg orally 2 times a day. The dose available is 0.5 mg per tablet. How many tablet(s) will the nurse administer for each dose? a. 1 b. 2 c. 3 d. 4

b. 2

When making rounds the nurse observes a purple wristband on a patient's wrist. What information about the patient does this provide the nurse? a. They are allergic to certain medications or foods. b. A "Do not resuscitate" order is in effect c. The patient has a high risk for falls. d. The patient is at risk for seizures.

b. A "Do not resuscitate" order is in effect

31. Which patient using an inhaler would benefit most from using a spacer? a. A 15 year old with a repaired cleft palate who is alert b. A 25 year old with limited coordination of the extremities c. A 50 year old with hearing impairment who uses a hearing aid d. A 72 year old with left-sided hemiparesis using a dry powder inhaler

b. A 25 year old with limited coordination of the extremities

Which patient will the nurse see first? a. A 56-year-old patient with oxygen with a lighter on the bedside table b. A 56-year-old patient with oxygen using an electric razor for grooming c. A 1-month-old infant looking at a shiny, round battery just out of arm's reach d. A 1-month-old infant with a pacifier that has no string around the baby's neck

b. A 56-year-old patient with oxygen using an electric razor for grooming

The nurse is caring for a patient who is immobile and needs to be turned every 2 hours. The patient has poor lower extremity circulation, and the nurse is concerned about irritation of the patient's toes. Which device will the nurse use? a. Hand rolls b. A foot cradle c. A trapeze bar d. A trochanter roll

b. A foot cradle

Which patient will cause the nurse to select a nursing diagnosis of Impaired physical mobility for a care plan? a. A patient who is completely immobile b. A patient who is not completely immobile c. A patient at risk for single-system involvement d. A patient who is at risk for multisystem problems

b. A patient who is not completely immobile

The nurse is monitoring for Never Events. Which finding indicates the nurse will report a Never Event? a. Lack of blood incompatibility with a blood transfusion. b. A surgical sponge is left in the patient's incision. c. Pulmonary embolism after lung surgery d. Stage II pressure ulcer

b. A surgical sponge is left in the patient's incision.

The nurse is assessing a patient for possible lead poisoning. Which patient is the nurse most likely assessing? a. A teenager b. A toddler c. A young adult d. An adolescent

b. A toddler

The nurse is caring for a patient who has a Stage IV pressure ulcer with grafted surgical sites. Which specialty bed will the nurse use for this patient? a. Low-air-loss b. Air-fluidized c. Lateral rotation d. Standard mattress

b. Air-fluidized

When caring for a group of patients, which task can the nurse delegate to the nursing assistive personnel (AP)? a. Assessing a surgical patient for risk of pressure ulcers b. Applying a gauze bandage to secure a nonsterile dressing c. Treating a pressure ulcer on the buttocks of a medical patient d. Implementing negative-pressure wound therapy on a stable patient

b. Applying a gauze bandage to secure a nonsterile dressing

The nurse is caring for a patient who has had a stroke causing total paralysis of the right side. To help maintain joint function and minimize the disability from contractures, passive ROM will be initiated. When should the nurse begin this therapy? a. After the acute phase of the disease has passed b. As soon as the ability to move is lost c. Once the patient enters the rehab unit d. When the patient requests it

b. As soon as the ability to move is lost

7. The nurse identifies that a patient has received Mylanta (simethicone) instead of the prescribed Pepto-Bismol (bismuth subsalicylate) for the problem of indigestion. The nurse's next intervention is to a. Do nothing, no harm has occurred. b. Assess and monitor the patient. c. Notify the physician, treat and document. d. Complete an incident report.

b. Assess and monitor the patient. After providing an incorrect medication, assessing and monitoring the patient to determine the effects of the medication is the first step. Notifying the physician and providing treatment would be the best next step. After the patient has stabilized, completing an incident report would be the last step in the process.

The nurse is preparing to lift a patient. Which action will the nurse take first? a. Position a drawsheet under the patient. b. Assess weight and determine assistance needs. c. Delegate the task to a nursing assistive personnel. d. Attempt to manually lift the patient alone before asking for assistance.

b. Assess weight and determine assistance needs.

The patient is admitted to a skilled care unit for rehabilitation after the surgical procedure of fixation of a fractured left hip. The patient's nursing diagnosis is Impaired physical mobility related to musculoskeletal impairment from surgery and pain with movement. The patient is able to use a walker but needs assistance ambulating and transferring from the bed to the chair. Which nursing intervention is most appropriate for this patient? a. Obtain assistance and physically transfer the patient to the chair. b. Assist with ambulation and measure how far the patient walks. c. Give pain medication after ambulation so the patient will have a clear mind. d. Bring the patient to the cafeteria for group instruction on ambulation.

b. Assist with ambulation and measure how far the patient walks.

The nurse is assessing the patient for respiratory complications of immobility. Which action will the nurse take when assessing the respiratory system? a. Inspect chest wall movements primarily during the expiratory cycle. b. Auscultate the entire lung region to assess lung sounds. c. Focus auscultation on the upper lung fields. d. Assess the patient at least every 4 hours.

b. Auscultate the entire lung region to assess lung sounds.

An older-adult patient is using a wheelchair to attend a physical therapy session. Which action by the nurse indicates safe transport of the patient? a. Positions patient's buttocks close to the front of wheelchair seat. b. Backs wheelchair into elevator, leading with large rear wheels first. c. Places locked wheelchair on same side of bed as patient's weaker side. d. Unlocks wheelchair for easy maneuverability when patient is transferring.

b. Backs wheelchair into elevator, leading with large rear wheels first.

A nurse is performing the three accuracy checks before administering an oral liquid medication to a patient. When will the nurse perform the second accuracy check? a. At the patient's bedside b. Before going to the patient's room c. When checking the medication order d. When selecting medication from the unit-dose drawer

b. Before going to the patient's room

The nurse is evaluating the body alignment of a patient in the sitting position. Which observation by the nurse will indicate a normal finding? a. The edge of the seat is in contact with the popliteal space. b. Both feet are supported on the floor with ankles flexed. c. The body weight is directly on the buttocks only. d. The arms hang comfortably at the sides.

b. Both feet are supported on the floor with ankles flexed.

A nurse is preparing to reposition a patient. Which task can the nurse delegate to the nursing assistive personnel? a. Determining the level of comfort b. Changing the patient's position c. Identifying immobility hazards d. Assessing circulation

b. Changing the patient's position

33. The nurse has been called to a hospital room where a patient is using a hair dryer from home. The patient has received an electrical shock from the dryer. The patient is unconscious and is not breathing. What is the best next step? a. Ask the family to leave the room. b. Check for a pulse. c. Begin compressions. d. Defibrillate the patient.

b. Check for a pulse. In this scenario, the patient is in a hospital setting, and it has been determined that the patient is not conscious and is not breathing. The next step is to check the pulse. An electrical shock can interfere with the heart's normal electrical impulses and can cause arrhythmias. Checking the pulse helps to determine the need for cardiopulmonary resuscitation (CPR) and defibrillation.

A nurse is evaluating care of an immobilized patient. Which action will the nurse take? a. Focus on whether the interdisciplinary team is satisfied with the care. b. Compare the patient's actual outcomes with the outcomes in the care plan. c. Involve primarily the patient's family and health care team to determine goal achievement. d. Use objective data solely in determining whether interventions have been successful.

b. Compare the patient's actual outcomes with the outcomes in the care plan.

The patient has been in bed for several days and needs to be ambulated. Which action will the nurse take first? a. Maintain a narrow base of support. b. Dangle the patient at the bedside. c. Encourage isometric exercises. d. Suggest a high-calcium diet.

b. Dangle the patient at the bedside.

The patient applies sequential compression devices after going to the bathroom. The nurse checks the patient's application of the devices and finds that they have been put on upside down. Which nursing diagnosis will the nurse add to the patient's plan of care? a. Risk for falls b. Deficient knowledge c. Risk for suffocation d. Impaired physical mobility

b. Deficient knowledge

29. The patient applies sequential compression devices after going to the bathroom. The nurse checks the patient's application of the devices and finds that they have been put on upside down. Which of the following nursing diagnoses will the nurse add to the patient's plan of care? a. Risk for poisoning b. Deficient knowledge c. Risk for imbalanced body temperature d. Risk for suffocation.

b. Deficient knowledge The patient needs to understand the purpose of the compression devices and that proper application is needed for them to be effective. The patient has a knowledge need and requires instruction regarding the device and its purpose and procedure. The nurse will intervene by teaching the patient about the sequential compression device and instructing the patient to call for assistance when getting up to go to the bathroom in the future, so that the nurse may assist with removal and proper reapplication. No data support a risk for poisoning, imbalanced body temperature, or suffocation

A home health nurse is teaching a family to prevent electrical shock. Which information will the nurse include in the teaching session? a. Run wires under the carpet. b. Disconnect items before cleaning. c. Grasp the cord when unplugging items. d. Use masking tape to secure cords to the floor.

b. Disconnect items before cleaning.

The nurse is preparing to transfer an uncooperative patient who does not have upper body strength. Which piece of equipment will be best for the nurses to obtain? a. Drawsheet b. Full body sling c. Overhead trapeze d. Friction-reducing slide sheet

b. Full body sling

The nurse is caring for a patient with a Stage IV pressure ulcer. Which type of healing will the nurse consider when planning care for this patient? a. Partial-thickness wound repair b. Full-thickness wound repair c. Primary intention d. Tertiary intention

b. Full-thickness wound repair

A patient is at risk for aspiration. Which nursing action is most appropriate? a. Give the patient a straw to control the flow of liquids. b. Have the patient self-administer the medication. c. Thin out liquids so they are easier to swallow. d. Turn the head toward the stronger side.

b. Have the patient self-administer the medication.

A patient is receiving vancomycin. Which physiological function is the priority for the nurses to assess? a. Vision b. Hearing c. Heart tones d. Bowel sounds

b. Hearing

An adult patient presents to the emergency department and is treated for hypothermia. What risk factor should the patient be assessed for? a. Tobacco use b. Homelessness c. High carbohydrate diet d. History of chronic respiratory disorder

b. Homelessness

A patient is in need of immediate pain relief for a severe headache. Which medication will the nurse administer to be absorbed the quickest? a. Acetaminophen 650 mg PO b. Hydromorphone 4 mg IV c. Ketorolac 8 mg IM d. Morphine 6 mg SQ

b. Hydromorphone 4 mg IV

A nurse is assessing pressure points in a patient placed in the Sims' position. Which areas will the nurse observe? a. Chin, elbow, hips b. Ileum, clavicle, knees c. Shoulder, anterior iliac spine, ankles d. Occipital region of the head, coccyx, heels

b. Ileum, clavicle, knees

The nurse documents the following assessment data: right heel with reddened area that does not blanch. Which nursing diagnosis will the nurse assign to this patient? a. Imbalanced nutrition: less than body requirements b. Ineffective peripheral tissue perfusion c. Risk for infection d. Acute pain

b. Ineffective peripheral tissue perfusion

The nurse is developing a plan of care for a patient diagnosed with activity intolerance. Which strategy will the nurse use to provide the best chance of maintaining patient compliance? a. Performing 20 minutes of aerobic exercise 7 days a week with 10-minute warm-up and cool-down periods b. Instructing the patient to use an exercise log to record day, time, duration, and responses to exercise activity c. Stressing the harm of not exercising by getting the patient to take responsibility for current health status d. Arranging for the patient to join a gym that takes self-pay rather than insurance

b. Instructing the patient to use an exercise log to record day, time, duration, and

25. The patient presents to the clinic with a family member. The family member states that the patient has been wandering around the house and mumbling. What is the first assessment the nurse should do? a. Ask the patient why she has been wandering around the house. b. Introduce self and ask the patient her name. c. Take the patient's blood pressure, pulse, temperature, and respiratory rate. d. Immediately do a complete head-to-toe neurologic assessment.

b. Introduce self and ask the patient her name. Introduce self and engage the patient by asking her name to assess orientation; ask the patient why she is visiting the clinic today. Continue the assessment with vital signs and a complete workup, including a neurologic assessment.

The nurse caring for a patient with a healing Stage III pressure ulcer notes that the wound is clean and granulating. Which health care provider's order will the nurse question? a. Use a low-air-loss therapy unit. b. Irrigate with Dakin's solution. c. Apply a hydrogel dressing. d. Consult a dietitian.

b. Irrigate with Dakin's solution.

The nurse is assessing body alignment for a patient who is immobilized. Which patient position will the nurse use? a. Supine position b. Lateral position c. Lateral position with positioning supports d. Supine position with no pillow under the patient's head

b. Lateral position

An adolescent tells the nurse that a health professional said the fibrous tissue that connects bone and cartilage was strained in a sporting accident. On which structure will the nurse focus an assessment? a. Tendon b. Ligament c. Synergistic muscle d. Antagonistic muscle

b. Ligament

32. An elderly patient presents to the hospital with a history of falls, confusion, and stroke. The nurse determines that the patient is at high risk for falls. Which of the following interventions is most appropriate for the nurse to take? a. Place the patient in restraints. b. Lock beds and wheelchairs when transferring. c. Place a bath mat outside the tub. d. Silence fall alert alarm upon request of family.

b. Lock beds and wheelchairs when transferring. Locking the bed and wheelchairs when transferring will help to prevent these pieces of equipment from moving during transfer and will assist in the prevention of falls. Patients are not automatically placed in restraints. The restraint process consists of many steps, including thorough assessment and exhausting of alternatives. All mats and rugs should be secured to help prevent falls. Silencing alarms upon the request of family is not appropriate and could contribute to an unsafe environment.

A patient diagnosed with diabetes mellitus is starting an exercise program. Which types of exercises will the nurse suggest? a. Low intensity b. Low to moderate intensity c. Moderate to high intensity d. High intensity

b. Low to moderate intensity

The patient is immobilized after undergoing hip replacement surgery. Which finding will alert the nurse to monitor for hemorrhage in this patient? a. Thick, tenacious pulmonary secretions b. Low-molecular-weight heparin doses c. SCDs wrapped around the legs d. Elastic stockings (TED hose)

b. Low-molecular-weight heparin doses Rationale: heparin is used to increase circulation so it increases bleeding risk

The emergency department has been notified of a potential bioterrorism attack. Which action by the nurse is priority? a. Monitor for specific symptoms. b. Manage all patients using standard precautions. c. Transport patients quickly and efficiently through the elevators. d. Prepare for posttraumatic stress associated with this bioterrorism attack.

b. Manage all patients using standard precautions.

The nurse is assessing an immobile patient for deep vein thromboses (DVTs). Which action will the nurse take? a. Remove elastic stockings every 4 hours. b. Measure the calf circumference of both legs. c. Lightly rub the lower leg for redness and tenderness. d. Dorsiflex the foot while assessing for patient discomfort.

b. Measure the calf circumference of both legs.

A nurse is caring for a patient with osteoporosis and lactose intolerance. What will the nurse do? a. Encourage dairy products. b. Monitor intake of vitamin D. c. Increase intake of caffeinated drinks. d. Try to do as much as possible for the patient.

b. Monitor intake of vitamin D.

23. The nurse is caring for a patient who has a stage IV pressure ulcer awaiting plastic surgery consultation. Which of the following specialty beds would be most appropriate? a. Standard mattress b. Nonpowered redistribution air mattress c. Low-air-loss therapy unit d. Lateral rotation

b. Nonpowered redistribution air mattress A low-air-loss therapy unit is utilized for stage IV pressure ulcers and when prevention or treatment of skin breakdown is needed. If the patient has a stage III or stage IV ulcer or a postoperative myocutaneous flap, the low-air-loss therapy unit would be an appropriate selection. A static air mattress or nonpowered redistribution is utilized for the patient at high risk for skin breakdown. A standard mattress is utilized for an individual who does not have actual or potential altered or impair skin integrity. Lateral rotation is used for treatment and prevention of pulmonary complications associated with mobility.

When the nurse discovers a patient on the floor, the patient states, "I fell out of bed". The nurse assesses the patient and then places the patient back in bed. Which action should the nurse take next? a. Do nothing, no harm has occurred. b. Notify the health care provider. c. Complete an incident report. d. Re-assess the patient.

b. Notify the health care provider.

The patient appears anxious as the nurse is preparing to change their wound dressing. Which action should the nurse take? a. Distract the patient with the television. b. Offer to explain what they should expect. c. Suggest that the patient "Close your eyes." d. Wait until family is visiting to support the patient.

b. Offer to explain what they should expect.

A nurse is administering oral medications to patients. Which action will the nurse take? a. Remove the medication from the wrapper and place it in a cup labeled with the patient's information. b. Place all of the patient's medications in the same cup, except medications with assessments. c. Crush enteric-coated medication and place it in a medication cup with water. d. Measure liquid medication by bringing liquid medication cup to eye level.

b. Place all of the patient's medications in the same cup, except medications with assessments.

The nurse is teaching a patient how to use a cane. Which information will the nurse include in the teaching session? a. Place the cane at the top of the hip bone. b. Place the cane on the stronger side of the body. c. Place the cane in front of the body and then move the good leg. d. Place the cane 10 to 15 inches in front of the body when walking.

b. Place the cane on the stronger side of the body.

The nurse caring for an unconscious patient who was involved in an automobile accident 2 weeks ago will give priority to which element when planning care to decrease the development of a decubitus ulcer? a. Resistance b. Pressure c. Weight d. Stress

b. Pressure

The nurse is collaborating with the dietitian about a patient with a Stage III pressure ulcer. Which nutrient will the nurse expect to be increased after collaboration with the dietitian? a. Fat b. Protein c. Vitamin E d. Carbohydrate

b. Protein

A nurse is caring for a patient with a wound. Which assessment data will be most relevant with regard to wound healing? a. Muscular strength assessment b. Pulse oximetry assessment c. Sensation assessment d. Sleep assessment

b. Pulse oximetry assessment

Which health care team member will the nurse consult when a patient has received a nursing diagnosis of Impaired skin integrity? a. Respiratory therapist b. Registered dietitian c. Case manager d. Chaplain

b. Registered dietitian

A nurse reviews an immobilized patient's laboratory results and discovers hypercalcemia. Which condition will the nurse monitor for most closely in this patient? a. Hypostatic pneumonia b. Renal calculi c. Pressure ulcers d. Thrombus formation

b. Renal calculi (stones/kidney stones)

The nurse is caring for two patients with the same last name. In this situation which right of medication administration is the priority to reduce the chance of an error? a. Right medication b. Right patient c. Right dose d. Right route

b. Right patient

The nurse caring for a patient in the burn unit should expect what type of wound healing when planning care for this patient? a. Partial-thickness repair b. Secondary intention c. Tertiary intention d. Primary intention

b. Secondary intention

The nurse admitting an older patient notes a shallow open reddish, pink ulcer without slough on the right heel of the patient. How will the nurse stage this pressure ulcer? a. Stage I b. Stage II c. Stage III d. Stage IV

b. Stage II

The nurse is caring for a patient in the emergency department with an injured shoulder. Which type of joint will the nurse assess? a. Fibrous b. Synovial c. Synergistic d. Cartilaginous

b. Synovial

4. A homeless adult patient presents to the emergency department. The nurse obtains the following vital signs: temperature 94.8° F, blood pressure 100/56, apical pulse 56, respiratory rate 12. Which of the vital signs should be addressed immediately? a. Respiratory rate b. Temperature c. Apical pulse d. Blood pressure

b. Temperature Hypothermia is defined as a core body temperature of 95° F or below. Homeless individuals are more at risk for hypothermia owing to exposure to the elements.

42. The nurse is caring for a patient after an open abdominal aortic aneurysm repair. The nurse requests an abdominal binder and carefully applies the binder. What is the best explanation for the nurse to use when teaching the patient the reason for the binder? a. The binder creates pressure over the abdomen. b. The binder supports the abdomen. c. The binder reduces edema at the surgical site. d. The binder secures the dressing in place.

b. The binder supports the abdomen. The patient has a large abdominal incision. This incision will need support, and an abdominal binder will support this wound, especially during movement, as well as during deep breathing and coughing. A binder can be used to create pressure over a body part, for example, over an artery after it has been punctured. A binder can be used to prevent edema, for example, in an extremity but is not used to reduce edema at a surgical site. A binder can be used to secure dressings such as elastic webbing applied around a leg after vein stripping.

The nurse is caring for a hospitalized patient. Which behavior alerts the nurse to consider the temporary need for a restraint? a. The patient refuses to call for help to go to the bathroom. b. The patient continues to remove the nasogastric tube. c. The patient gets confused regarding the time at night. d. The patient does not sleep and continues to ask for items.

b. The patient continues to remove the nasogastric tube.

15. The nurse is caring for a hospitalized patient. Which of the following behaviors alerts the nurse to consider the need for restraint? a. The patient refuses to call for help to go to the bathroom. b. The patient continues to remove the nasogastric tube. c. The patient gets confused regarding the time at night. d. The patient does not sleep and continues to ask for items.

b. The patient continues to remove the nasogastric tube. Restraints are utilized only when alternatives have been exhausted, the patient continues a behavior that can be harmful to himself or others, and the restraint is clinically justified. In this circumstance, continuing to remove a needed nasogastric tube would meet these criteria. Refusing to call for help, although unsafe, is not a reason for restraint. Getting confused at night regarding the time or not sleeping and bothering the staff to ask for items is not a reason for restraint.

During the admission assessment, the nurse assesses the patient for fall risk. Which finding will alert the nurse to an increased risk for falls? a. The patient is oriented. b. The patient takes a hypnotic. c. The patient walks only 2 miles a day. d. The patient recently became widowed.

b. The patient takes a hypnotic.

The nurse has placed a yellow armband on a 70-year-old patient. Which observation by the nurse will indicate the patient has an understanding of this action? a. The patient removes the armband to bathe. b. The patient wears the red nonslip footwear. c. The patient insists on taking a "water" pill in the evening. d. The patient who is allergic to penicillin asks the name of a new medicine.

b. The patient wears the red nonslip footwear.

35. The nurse has placed a patient on high-risk alert for falls. Which of the following observations by the nurse would indicate that the patient has an understanding of this alert? a. The patient removes the high alert armband to bathe. b. The patient wears the red nonslip footwear. c. The call light is kept on the bedside table. d. The patient insists on taking a "water" pill on home schedule in the evening.

b. The patient wears the red nonslip footwear. Red nonslip footwear helps to grip the floor and decreases the chance of falling. The communication armband should stay in place and should not be removed, so that all members of the interdisciplinary team have the information about the high risk for falls. Call lights should be kept within reach of the patient. Taking diuretics early in the day assists with decreasing the number of bathroom trips at night—the time when falls are most frequent.

Which goal is most appropriate for a patient who has had a total hip replacement? a. The patient will ambulate briskly on the treadmill by the time of discharge. b. The patient will walk 100 feet using a walker by the time of discharge. c. The nurse will assist the patient to ambulate in the hall 2 times a day. d. The patient will ambulate by the time of discharge.

b. The patient will walk 100 feet using a walker by the time of discharge.

The patient is brought to the emergency department with possible injury to the left shoulder. Which area will the nurse assess to best determine joint mobility? a. The patient's gait b. The patient's range of motion c. The patient's ethnic influences d. The patient's fine-motor coordination

b. The patient's range of motion

The patient is eager to begin an exercise program with a 2-mile jog. The nurse instructs the patient to warm up. The patient does not want to waste time with a "warm-up." Which information will the nurse share with the patient? a. The warm-up in this case can be done after the 2-mile jog. b. The warm-up prepares the body and decreases the potential for injury. c. The warm-up allows the body to readjust gradually to baseline functioning. d. The warm-up should be performed with high intensity to prepare for the coming challenge.

b. The warm-up prepares the body and decreases the potential for injury.

21. A patient with an intravenous infusion requests a new gown after bathing. Which of the following actions is most appropriate? a. Disconnect the intravenous tubing, thread the end through the sleeve of the old gown and through the sleeve of the new gown, and reconnect. b. Thread the intravenous bag and tubing through the sleeve of the old gown and through the sleeve of the new gown without disconnecting. c. Inform the patient that a new gown is not an option while receiving an intravenous infusion in the hospital. d. Call the charge nurse for assistance because linen use is monitored and this is not a common procedure.

b. Thread the intravenous bag and tubing through the sleeve of the old gown and through the sleeve of the new gown without disconnecting. Procedure-related accidents such as contamination of sterile items can occur in the health care setting. Keeping the intravenous tubing intact without breaks in the system is imperative to decrease the risk of infection while changing a patient's gown and satisfying the patient's request.

The nurse is caring for a patient who is immobile and is at risk for skin impairment. The plan of care includes turning the patient. Which is the best method for repositioning the patient? a. Place the patient in a 30-degree supine position. b. Utilize a transfer device to lift the patient. c. Elevate the head of the bed 45 degrees. d. Slide the patient into the new position.

b. Utilize a transfer device to lift the patient.

The nurse is giving an intramuscular (IM) injection. Upon aspiration, the nurse notices blood return in the syringe. What should the nurse do? a. Administer the injection at a slower rate. b. Withdraw the needle and prepare the injection again. c. Pull the needle back slightly and inject the medication. d. Give the injection and hold pressure over the site for 3 minutes.

b. Withdraw the needle and prepare the injection again.

The nurse is completing an assessment on a patient who has a Stage IV pressure ulcer. The wound is odorous with a drain is currently in place. Which statement by the patient indicates issues with self-concept? a. "I am so weak and tired. I just want to feel better." b. "I been thinking I will be ready to go home early next week." c. "I really need a bath and linen change right; I feel so awful." d. "I am hoping there will be something good to eat for my dinner tonight."

c. "I really need a bath and linen change right; I feel so awful."

A nurse is preparing to administer an injection to a patient. Which statement made by the patient is an indication for the nurse to use the Z-track method? a. "I am allergic to many medications." b. "I'm really afraid that a big needle will hurt." c. "The last shot really irritated my skin around the site." d. "My legs are too obese for the needle to go through."

c. "The last shot really irritated my skin around the site."

The nurse is completing a skin risk assessment using the Braden Scale. The patient has slight sensory impairment, has skin that is rarely moist, walks occasionally, and has slightly limited mobility, along with excellent intake of meals and no apparent problem with friction and shear. Which score will the nurse document for this patient? a. 15 b. 17 c. 20 d. 23

c. 20

The nurse is preparing to administer an injection into the deltoid muscle of an adult patient weighing approximately 160 lb. Which needle size and length will the nurse choose? a. 18 gauge x 1 1/2 inch b. 23 gauge x 1/2 inch c. 25 gauge x 1 inch d. 27 gauge x 5/8 inch

c. 25 gauge x 1 inch

The nurse needs to move a patient up in bed using a drawsheet. The nurse has another nurse helping. In which order will the nurses perform the steps, beginning with the first one? 1. Grasp the drawsheet firmly near the patient. 2. Move the patient and drawsheet to the desired position. 3. Position one nurse at each side of the bed. 4. Place the drawsheet under the patient from shoulder to thigh. 5. Place your feet apart with a forward-backward stance. 6. Flex knees and hips and on count of three shift weight from the front to back leg. a. 1, 4, 5, 6, 3, 2 b. 4, 1, 3, 5, 6, 2 c. 3, 4, 1, 5, 6, 2 d. 5, 6, 3, 1, 4, 2

c. 3, 4, 1, 5, 6, 2

A nurse is preparing to administer a medication from a vial. In which order will the nurse perform the steps, starting with the first step? 1. Invert the vial. 2. Fill the syringe with medication. 3. Inject air into the airspace of the vial. 4. Clean with alcohol swab and allow to dry. 5. Pull back on the plunger the amount to be drawn up. 6. Tap the side of the syringe barrel to remove air bubbles. a. 4, 1, 5, 3, 6, 2 b. 1, 4, 5, 3, 2, 6 c. 4, 5, 3, 1, 2, 6 d. 1, 4, 5, 3, 6, 2

c. 4, 5, 3, 1, 2, 6

The supervising nurse is observing several different nurses. Which action will cause the supervising nurse to intervene? a. A nurse administers a vaccine without aspirating. b. A nurse gives an IV medication through a 22-gauge IV needle without blood return. c. A nurse draws up the NPH insulin first when mixing a short-acting and intermediate-acting insulin. d. A nurse calls the health care provider for a patient with nasogastric suction and orders for oral meds.

c. A nurse draws up the NPH insulin first when mixing a short-acting and intermediate-acting insulin.

The nurse is caring for a group of patients. Which patient will the nurse see first? a. A patient with chronic obstructive pulmonary disease doing stretching exercises b. A patient with diabetes mellitus carrying hard candy while doing exercises c. A patient with a heart attack doing isometric exercises d. A patient with hypertension doing Tai Chi exercises

c. A patient with a heart attack doing isometric exercises

The nurse is caring for a group of patients. Which patient will the nurse see first? a. A patient with a Stage IV pressure ulcer b. A patient with a Braden Scale score of 18 c. A patient with appendicitis using a heating pad d. A patient with an incision that is approximated

c. A patient with appendicitis using a heating pad

The nurse is caring for a patient with a spinal cord injury and notices that the patient's hips have a tendency to rotate externally when the patient is supine. Which device will the nurse use to help prevent injury secondary to this rotation? a. Hand rolls b. A trapeze bar c. A trochanter roll d. Hand-wrist splints

c. A trochanter roll

The patient reports being tired and weak and lacks energy. Upon assessment, the nurse finds that patient has gained weight, and blood pressure and pulse are elevated after climbing stairs. Which nursing diagnosis will the nurse add to the care plan? a. Fatigue b. Ineffective coping c. Activity intolerance d. Decreased cardiac output

c. Activity intolerance

12. The nurse is presenting an educational session on safety for parents of adolescents. The nurse should include which of the following teaching points? a. Adolescents need unsupervised time with friends two to three times a week. b. Parents and friends should teach adolescents how to drive. c. Adolescents need information about the effects of beer on the liver. d. Adolescents need to be reminded to use seatbelts on long trips.

c. Adolescents need information about the effects of beer on the liver. Providing information about drugs and alcohol is important because adolescents may choose to participate in risk-taking behaviors. Adolescents need to socialize but need supervision. Parents can encourage and support learning processes associated with driving, but organized classes can help to decrease motor vehicle accidents. Seatbelts should be used all the time.

34. A nurse is in the hallway assisting a patient to ambulate and hears an alarm sound. What is the best next step for the nurse to take? a. Seek out the source of the alarm. b. Wait to see if the alarm discontinues. c. Ask another nurse to check on the alarm. d. Continue ambulating the patient.

c. Ask another nurse to check on the alarm. The nurse who heard the alarm has a duty to address it even though she is busy with another patient. Ask someone to check on the alarm. The nurse cannot leave the patient in the hallway to look for the source of the alarm and cause a potentially unsafe situation for this patient, but a patient on the unit may have an urgent need. Someone needs to seek out the source of the alarm and address it. Never ignore an alarm. Alarms are in place to maximize the safety of the patient. Waiting to see if an alarm stops may cause a delay in a possible emergency situation.

The nurse is admitting an older adult to the surgical unit. What intervention is necessary when determining the safe use of side rails for this patient? a. Explain to the patient the need to call for assistance when side rails are up. b. Discuss whether the patient is accepting of having the side rails up. c. Assess the patient's ability to effectively follow instructions. d. Always keeping the bed in its lowest position to the floor.

c. Assess the patient's ability to effectively follow instructions.

17. The nurse is caring for a patient who suddenly becomes confused and tries to remove an intravenous infusion. The nurse begins to develop a plan to care for the patient. Which nursing intervention should take priority? a. Gather restraint supplies. b. Try alternatives to restraint. c. Assess the patient. d. Call the physician for a restraint order.

c. Assess the patient. When a patient becomes suddenly confused, the priority is to assess the patient, including checking laboratory test and oxygen status and treating and eliminating the cause of the change in mental status. If interventions and alternatives are exhausted, the nurse working with the physician may determine the need for restraints.

The nurse is working on an orthopedic rehabilitation unit that requires lifting and positioning of patients. Which personal injury will the nurse most likely try to prevent? a. Arm b. Hip c. Back d. Ankle

c. Back

A nurse is preparing to assess a patient for orthostatic hypotension. Which piece of equipment will the nurse obtain to assess for this condition? a. Thermometer b. Elastic stockings c. Blood pressure cuff d. Sequential compression devices

c. Blood pressure cuff

A nurse is assessing a patient with activity intolerance for possible orthostatic hypotension. Which finding will help confirm orthostatic hypotension? a. Blood pressure sitting 120/64; blood pressure 140/70 standing b. Blood pressure sitting 126/64; blood pressure 120/58 standing c. Blood pressure sitting 130/60; blood pressure 110/60 standing d. Blood pressure sitting 140/60; blood pressure 130/54 standing

c. Blood pressure sitting 130/60; blood pressure 110/60 standing

Which intervention should be included as the nurse cleanses a wound? a. Allow the solution to flow from the most contaminated to the least contaminated. b. Scrub vigorously when applying noncytotoxic solution to the skin. c. Cleanse in a direction from the least contaminated area. d. Utilize clean gauze and clean gloves to cleanse a site.

c. Cleanse in a direction from the least contaminated area.

41. The nurse is cleansing a wound site. As the nurse administers the procedure, what intervention should be included? a. Allowing the solution to flow from the most contaminated to the least contaminated b. Scrubbing vigorously when applying solutions to the skin c. Cleansing in a direction from the least contaminated area d. Utilizing clean gauge and clean gloves to cleanse a site

c. Cleansing in a direction from the least contaminated area Cleanse surgical or traumatic wounds by applying noncytotoxic solution with sterile gauze or irrigations. Cleanse in a direction from the least contaminated area. Use gentle friction when applying solutions to the skin, and allow irrigation to flow from the least to the most contaminated area.

Which behavior indicates the nurse is using a team approach when caring for a patient who is experiencing alterations in mobility? a. Delegates assessment of lung sounds to nursing assistive personnel b. Becomes solely responsible for modifying activities of daily living c. Consults physical therapy for strengthening exercises in the extremities d. Involves respiratory therapy for altered breathing from severe anxiety levels

c. Consults physical therapy for strengthening exercises in the extremities

26. The emergency department has been notified of a potential bioterrorist attack. The nurse assigned to the department realizes that the most important task for safety in this situation is to a. Carry out the role and responsibilities of the nurse quickly and efficiently. b. Cluster all patients with the same symptoms to a specific part of the department. c. Determine the biologic agent and manage all patients using Standard Precautions. d. Prepare for post-traumatic stress associated with this bioterrorist attack.

c. Determine the biologic agent and manage all patients using Standard Precautions. It is essential to determine the agent and manage all patients who are symptomatic with the suspected or confirmed bioterrorism-related illness using Standard Precautions. For certain diseases, additional precautions may be necessary. Clustering patients may be helpful with staffing and, depending on the illness, may decrease the spread. All nurses every day should carry out their roles quickly and efficiently. Psychosocial concerns are important but are not the first priority at this moment.

The nurse caring for an immobile patient wants to decrease the risk of the formation of pressure ulcers. Which action will the nurse take first? a. Offer favorite fluids. b. Turn the patient every 2 hours. c. Determine the patient's risk factors. d. Encourage increased quantities of carbohydrates and fats.

c. Determine the patient's risk factors.

8. The nurse preceptor recognizes the new nurse's ability to determine patient safety risks when which behavior is observed? a. Checking patient identification once every shift b. Multitasking by gathering two patients' medications c. Disposing of used needles in a red needle container d. Raising all four side rails per family request

c. Disposing of used needles in a red needle container Needles, syringes, and other single-use injection devices should be used once and disposed of in safety red needle containers that will be disposed of properly. Patient identification should be checked multiple times a day, including before each medication, treatment, procedure, blood administration, and transfer, and at the beginning of each shift. Gathering more than one patient's medication increases the likelihood of error. Raising all four side rails is considered a restraint and requires special orders, assessment, and monitoring of the patient.

24. The nurse is caring for a patient with a pressure ulcer on the left hip. The ulcer is black. The nurse recognizes that the next step in caring for this patient includes a. Monitoring of the wound. b. Irrigation of the wound. c. Débridement of the wound. d. Management of drainage.

c. Débridement of the wound. Débridement is the removal of nonviable necrotic tissue. Removal of necrotic tissue is necessary to rid the ulcer of a source of infection, to enable visualization of the wound bed, and to provide a clean base for healing. A wound will not move through the phases of healing if the wound is infected. Irrigating the wound with noncytotoxic cleaners will not damage or kill fibroblasts and healing tissue and will help to keep the wound clean once débrided. When treating a pressure ulcer, it is important to monitor and reassess the wound at least every 8 hours. Management of drainage will help keep the wound clean.

31. The nurse is caring for a patient with a stage III pressure ulcer. The nurse has assigned a nursing diagnosis of Risk for infection. Which intervention would be most important for this patient? a. Teach the family how to manage the odor associated with the wound. b. Discuss with the family how to prepare for care of the patient in the home. c. Encourage thorough handwashing of all individuals caring for the patient. d. Encourage increased quantities of carbohydrates and fats.

c. Encourage thorough handwashing of all individuals caring for the patient The number one way to decrease the risk of infection by breaking the chain of infection is to wash hands. Encouraging fluid and food intake helps with overall wellness and wound healing, especially protein, but an increase in carbohydrates and fats does not relate to the risk of infection. If the patient will be discharged before the wound is healed, the family will certainly need education on how to care for the patient. Teaching the family how to manage the odor associated with a wound is certainly important, but these interventions do not directly relate to the risk of infection and breaking the chain of the infectious process.

When the nurse administers an intramuscular (IM) corticosteroid injection, the nurse aspirates. What is the rationale for the nurse aspirating? a. Prevent the patient from choking. b. Increase the force of the injection. c. Ensure proper placement of the needle. d. Reduce the discomfort of the injection.

c. Ensure proper placement of the needle.

The nurse is presenting an educational session on safety for parents of adolescents. Which information will the nurse include in the teaching session? a. Frequent injuries related to poor psychomotor coordination b. Recognizing common signs and symptoms of the schizophrenia c. Failing grades and changes in dress may indicate substance abuse d. The importance of the use seat belts whenever riding in the backseat of a car

c. Failing grades and changes in dress may indicate substance abuse

The nurse is caring for a patient who is experiencing a full-thickness wound repair. Which type of tissue will the nurse expect to observe when the wound is healing? a. Eschar b. Slough c. Granulation d. Purulent drainage

c. Granulation

9. The nurse is caring for a patient who is experiencing a full-thickness repair. The nurse would expect to see which of the following in this type of repair? a. Eschar b. Slough c. Granulation d. Purulent drainage

c. Granulation Granulation tissue is red, moist tissue composed of new blood vessels, the presence of which indicates progression toward healing. Soft yellow or white tissue is characteristic of slough—a substance that needs to be removed for the wound to heal. Black or brown necrotic tissue is called eschar, which also needs to be removed for a wound to heal. Purulent drainage is indicative of an infection and will need to be resolved for the wound to heal.

The wound care nurse is monitoring a patient with a Stage III pressure ulcer whose wound presents with healthy tissue. How should the nurse document this ulcer in the patient's medical record? a. Stage I pressure ulcer b. Healing Stage II pressure ulcer c. Healing Stage III pressure ulcer d. Stage III pressure ulcer

c. Healing Stage III pressure ulcer

A patient is to receive a proton pump inhibitor through a nasogastric (NG) feeding tube. Which nursing action is vital to ensuring effective absorption? a. Thoroughly shake the medication before administering. b. Position patient in the supine position for 30 minutes to 1 hour. c. Hold feeding for at least 30 minutes after medication administration. d. Flush tube with 10 to 15 mL of water, after all medications are administered.

c. Hold feeding for at least 30 minutes after medication administration.

The nurse is caring for a patient who cannot bear weight but needs to be transferred from the bed to a chair. The nurse decides to use a transportable hydraulic lift. What action indicates the nurse is aware of appropriate hydraulic life use? a. Places a horseshoe-shaped base on the opposite side from the chair. b. Removes straps before lowering the patient to the chair. c. Hooks longer straps to the bottom of the sling. d. Attaches short straps to the bottom of the sling.

c. Hooks longer straps to the bottom of the sling.

The nurse is caring for a patient with a Stage IV pressure ulcer. Which nursing diagnosis should the nurse add to the care plan? a. Readiness for enhanced nutrition b. Impaired physical mobility c. Impaired skin integrity d. Chronic pain

c. Impaired skin integrity

A nurse is following the no-lift policy when working to prevent personal injury from twisting. Which type of personal back injury is the nurse most likely trying to prevent? a. Thoracic b. Cervical c. Lumbar d. Sacral

c. Lumbar

Which laboratory data will be important for the nurse to monitor when a patient develops a pressure ulcer? a. Vitamin E b. Potassium c. Prealbumin d. Sodium

c. Prealbumin

The nurse providing care to a bedridden patient raises the height of the bed. What is the rationale for the nurse's action? a. Narrows the nurse's base of support. b. Allows the nurse to bring feet closer together. c. Prevents a shift in the nurse's base of support. d. Shifts the nurse's center of gravity farther away from the base of support.

c. Prevents a shift in the nurse's base of support.

30. The nurse enters the patient's room and notices a small fire in the headlight above the patient's bed. Immediately, the nurse assigns a nursing diagnosis of risk for injury with a goal for the patient to be safe. Which of the following actions should the nurse take first? a. Activate the alarm. b. Extinguish the fire. c. Remove the patient. d. Confine the fire.

c. Remove the patient. Nurses use the mnemonic RACE to set priorities in case of fire. All of these interventions are necessary, but this patient is in immediate danger with the fire being over his head and should be rescued and removed from the situation.

Which finding will alert the nurse to a potential wound dehiscence? a. Protrusion of visceral organs through a wound opening b. Chronic drainage of fluid through the incision site c. Report by patient that something has given way d. Drainage that is odorous and purulent

c. Report by patient that something has given way

The nurse gives instructions to a nursing assistive personnel (NAP) regarding exercise for a patient. Which action by the NAP indicates a correct understanding of the directions? a. Determines the patient's ability to exercise. b. Teaches the patient how to do the exercises. c. Reports the patient got dizzy after exercising. d. Advises the patient to work through the pain.

c. Reports the patient got dizzy after exercising.

A nurse is assisting the patient to perform isometric exercises. Which action will the nurse take? a. Encourage wearing tight shoes. b. Set the pace for the exercise session. c. Stop the exercise if pain is experienced. d. Force muscles or joints to go just beyond resistance.

c. Stop the exercise if pain is experienced.

The nurse notes that a patient has a black pressure ulcer on the left hip. Which event will the nurse anticipate when planning care for this patient? a. Increased monitoring of the wound condition b. Documenting the wound's status daily c. Surgical debridement of the wound d. Increased drainage from wound

c. Surgical debridement of the wound

16. The nurse is discussing with a patient's physician the need for restraint. The nurse indicates that alternatives have been utilized. What behaviors would indicate that the alternatives are working? a. The patient continues to get up from the chair at the nurses' station. b. The patient apologizes for being "such a bother." c. The patient folds three washcloths over and over. d. The sitter leaves the patient alone to go to lunch.

c. The patient folds three washcloths over and over. Offering diversionary activities such as something to hold is a way to keep the hands busy and provides an alternative to restraints. Assigning a room near the nurses' station or a chair at the desk can be an alternative for continuous monitoring. Getting up constantly can be cause for concern. Apologizing is not an alternative to restraints. Having a sitter sit with the patient to keep him occupied can be an alternative to restraints, but the sitter needs to be continuous.

9. The nurse is completing discharge education for the patient regarding home medications. Which patient behavior is an indication that the patient understands the directions regarding the antibiotic medication? a. The patient nods throughout the educational session. b. The patient reads the medication prescription out loud. c. The patient states, "I will finish the antibiotic in ten days." d. The patient asks where to get the prescription filled.

c. The patient states, "I will finish the antibiotic in ten days." The patient stating the time frame for when the medication will be complete is the best answer. Nodding, reading the prescription out loud, or knowing where to get the prescription filled does not indicate understanding regarding directions for taking the antibiotic.

23. During the admission assessment, the nurse assesses the patient for fall risk. Which of the following has the greatest potential to increase the patient's risk for falls? a. The patient is 59 years of age. b. The patient walks 2 miles a day. c. The patient takes Benadryl (diphenhydramine) for allergies. d. The patient recently became widowed.

c. The patient takes Benadryl (diphenhydramine) for allergies. Benadryl (diphenhydramine) has the potential to cause drowsiness and dizziness as a side effect, thereby increasing the risk for falls. Over 60 is the age typically found on fall assessments that increase the risk for falls. Walking has many benefits, including increasing strength, which would be beneficial in decreasing risk. Becoming widowed would increase stress and may affect concentration but is not the greatest risk.

The nurse is evaluating care of a patient for crutches. Which finding indicates a successful outcome? a. The top of the crutch is three to four finger widths from the armpit. b. The elbows are slightly flexed at 30 to 35 degrees when the patient is standing. c. The tip of the crutch is 4 to 6 inches anterior to the front of the patient's shoes. d. The position of the handgrips allows the axilla to support the patient's body weight.

c. The tip of the crutch is 4 to 6 inches anterior to the front of the patient's shoes.

A nurse is assessing the skin of an immobilized patient. What will the nurse do? a. Assess the skin every 4 hours. b. Limit the amount of fluid intake. c. Use a standardized tool such as the Braden Scale. d. Have special times for inspection so as to not interrupt routine care.

c. Use a standardized tool such as the Braden Scale.

A nurse is providing care to a patient. Which action indicates the nurse is following the National Patient Safety Goals? a. Identifies patient with one identifier before transporting to x-ray department. b. Initiates an intravenous (IV) catheter using clean technique on the first try. c. Uses medication bar coding when administering medications. d. Obtains vital signs to place on a surgical patient's chart.

c. Uses medication bar coding when administering medications.

An older-adult patient needs an intramuscular (IM) injection of antibiotic. Which site is best for the nurse to use? a. Deltoid b. Dorsal gluteal c. Ventrogluteal d. Vastus lateralis

c. Ventrogluteal

18. The nurse knows that four categories of risk have been identified in the health care environment. Which of the following provides the best examples of those risks? a. Tile floors, cold food, scratchy linen, and noisy alarms b. Carpeted floors, ice machine empty, unlocked supply cabinet, and call light in reach c. Wet floors, pinching fingers in door, failure to use lift for patient, and alarms not functioning properly d. Dirty floors, hallways blocked, medication room locked, and alarms set

c. Wet floors, pinching fingers in door, failure to use lift for patient, and alarms not functioning properly The four categories are falls, patient-inherent accidents, procedure-related accidents, and equipment-related accidents. Wet floors contribute to falls, pinching finger in door is patient inherent, failure to use the lift is procedure related, and an alarm not functioning properly is equipment related. Tile floors and carpeted or dirty floors do not necessarily contribute to falls. Cold food, ice machine empty, and hallways blocked are not patient-inherent issues in the hospital setting but are more of patient satisfaction or infection control issues or fire safety issues. Scratchy linen, unlocked supply cabinet, and medication room locked are not procedure-related accidents. These are patient satisfaction issues and control of supply issues, and are examples of actually following a procedure correctly. Noisy alarms, call light within reach, and alarms set are not equipment-related accidents but are patient satisfaction issues and examples of following a procedure correctly.

A nurse is attempting to minimize the risk of future infection for a post-surgical patient about to be discharged. Which technique will the nurse teach the patient to best achieve this goal? a. Sanitizing of eating utensils b. Medical asepsis handwashing c. Wound care using surgical asepsis d. Limiting visitors during flu season

c. Wound care using surgical asepsis

13. The nurse discussed threats to adult safety with a college group. Which of the following statements would indicate understanding of the topic? a. "Our campus is safe; we leave our dorms unlocked all the time." b. "As long as I have only two drinks, I can still be the designated driver." c. "I am young, so I can work nights and go to school with 2 hours' sleep." d. "I guess smoking even at parties is not good for my body."

d. "I guess smoking even at parties is not good for my body." Lifestyle choices frequently affect adult safety. Smoking conveys great risk for pulmonary and cardiovascular disease. It is prudent to secure belongings. When an individual has been determined to be the designated driver, that individual does not consume alcohol, beer, or wine. Sleep is important no matter the age of the individual and is important for rest and integration of learning. The average young adult needs 6 1/2 to 8 hours of sleep each night.

24. The older patient presents to the emergency department after stepping in front of a car at a crosswalk. After the patient has been triaged, the nurse interviews the patient. Which of the following comments would require follow-up by the nurse? a. "I try to exercise, so I walk that block almost every day." b. "I waited and stepped out when the traffic sign said go." c. "The car was going too fast, the speed limit is 20." d. "I was so surprised; I didn't see or hear the car coming."

d. "I was so surprised; I didn't see or hear the car coming." The patient did not see or hear the car coming. As patients age, sensory impairment can increase the risk for injury. This statement by the patient would require follow-up by the nurse. The patient needs hearing and eye examinations. Exercise is important at every stage of development. The patient seemed to comprehend how to cross an intersection correctly and was able to determine the speed of the car.

The nurse is providing information regarding safety and accidental poisoning to a grandparent who will be taking custody of a 1-year-old grandchild. Which comment by the grandparent will cause the nurse to intervene? a. "The number for poison control is 800-222-1222." b. "Never induce vomiting if my grandchild drinks bleach." c. "I should call 911 if my grandchild loses consciousness." d. "If my grandchild eats a plant, I should provide syrup of ipecac."

d. "If my grandchild eats a plant, I should provide syrup of ipecac."

31. The nurse is providing information regarding safety and accidental poisoning to a grandmother who will be taking custody of a 1-year-old grandchild. Which of the following comments would indicate that the grandmother needs further instruction? a. "The number for poison control is 800-222-1222." b. "Never induce vomiting if my grandchild drinks bleach." c. "I should call 911 if my grandchild loses consciousness." d. "If my grandchild eats a plant, I should provide syrup of ipecac."

d. "If my grandchild eats a plant, I should provide syrup of ipecac." Syrup of ipecac to induce vomiting after ingestion of a poison has not been proven effective in preventing poisoning. This medication should not be administered to the child. The poison control number is 800-222-1222. After a caustic substance such as bleach has been drunk, do not induce vomiting. This can cause further burning and injury as the medication is eliminated. Loss of consciousness associated with poisoning requires calling 911.

1. A home health nurse is performing a home assessment for safety. Which of the following comments by the patient would indicate a need for further education? a. "I will schedule an appointment with a chimney inspector next week." b. "Daylight savings is the time to change batteries on the carbon monoxide detector." c. "If I feel dizzy when using the heater, I need to have it inspected." d. "When it is cold outside in the winter, I can warm my car up in the garage."

d. "When it is cold outside in the winter, I can warm my car up in the garage." Allowing a car to run in the garage introduces carbon monoxide into the environment and decreases the available oxygen for human consumption. Garages should be opened and not just cracked to allow fresh air into the space and allay this concern. Checking the chimney and heater, changing the batteries on the detector, and following up on symptoms such as dizziness, nausea, and fatigue are all statements that would indicate that the individual has understood the education.

A home health nurse is performing a home assessment for safety. Which comment by the patient will cause the nurse to follow up? a. "Every December is the time to change batteries on the carbon monoxide detector." b. "I will schedule an appointment with a chimney inspector next week." c. "If I feel dizzy when using the heater, I need to have it inspected." d. "When it is cold outside in the winter, I will use a non-vented heater."

d. "When it is cold outside in the winter, I will use a non-vented heater."

What statement by the nurse demonstrates an understanding of food safety to be provided for a patient living alone? a. "It's acceptable to eat unwashed fruits and vegetables if they are organically grown." b. "It's best to allow cooked foods to thoroughly cool off before putting them into the refrigerator." c. "You can use the same cutting board for meats and for vegetables if it is washed between uses." d. "Your perishable left-over food should be stored in a refrigerator at below 45° F"

d. "Your perishable left-over food should be stored in a refrigerator at below 45° F"

A nurse is preparing an intravenous (IV) piggyback infusion. In which order will the nurse perform the steps, starting with the first one? 1. Compare the label of the medication with the medication administration record at the patient's bedside. 2. Connect the tubing of the piggyback infusion to the appropriate connector on the upper Y-port. 3. Hang the piggyback medication bag above the level of the primary fluid bag. 4. Clean the main IV line port with an antiseptic swab. 5. Connect the infusion tubing to the medication bag. 6. Regulate flow. a. 5, 2, 1, 4, 3, 6 b. 5, 2, 1, 3, 4, 6 c. 1, 5, 4, 3, 2, 6 d. 1, 5, 3, 4, 2, 6

d. 1, 5, 3, 4, 2, 6

4. A patient has an order to receive 10 units of U-50 insulin. The nurse is using a U-100 syringe. How many units should the nurse draw up in the syringe and administer? a. 0.2 units b. 2 units c. 5 units d. 20 units

d. 20 units

The patient has a risk for skin impairment and has a 15 on the Braden Scale upon admission. The nurse has implemented interventions. Upon reassessment, which Braden score will be the best sign that the risk for skin breakdown is removed? a. 12 b. 13 c. 20 d. 23

d. 23

A nurse is developing an exercise plan for a middle-aged patient. In which order will the nurse instruct the patient to execute the plan, beginning with the first step? 1. Design the fitness program. 2. Assemble equipment. 3. Assess fitness level. 4. Monitor progress. 5. Get started. a. 5, 1, 3, 2, 4 b. 1, 2, 3, 5, 4 c. 2, 5, 3, 1, 4 d. 3, 1, 2, 5, 4

d. 3, 1, 2, 5, 4

A health care provider prescribes aspirin 650 mg every 4 hours PO when febrile. For which patient will this order be appropriate? a. A 7 year old with a bleeding disorder b. A 21 year old with a sprained ankle c. A 35 year old with a severe headache from hypertension d. A 62 year old with a high fever from an infection

d. A 62 year old with a high fever from an infection

The nurse is caring for a postoperative patient recovering from a medial meniscus repair of the right knee. Which action should the nurse take to assist with pain management? a. Monitor vital signs every 15 minutes. b. Check pulses in the right foot. c. Keep the leg dependent. d. Apply ice.

d. Apply ice.

A patient may need restraints. Which task can the nurse delegate to a nursing assistive personnel? a. Determining the need for restraints b. Assessing the patient's orientation c. Obtaining an order for a restraint d. Applying the restraint

d. Applying the restraint

The patient has the nursing diagnosis of Impaired physical mobility related to pain in the left shoulder. Which priority action will the nurse take? a. Encourage the patient to do self-care. b. Keep the patient as mobile as possible. c. Encourage the patient to perform ROM. d. Assist the patient with comfort measures.

d. Assist the patient with comfort measures. Rationale: Pain must be controlled so the patient will not be reluctant to initiate movement.

A nurse is providing passive range of motion (ROM) for a patient with impaired mobility. Which technique will the nurse use for each movement? a. Each movement is repeated 5 times by the patient. b. Each movement is performed until the patient experiences pain. c. Each movement is completed quickly and smoothly by the nurse. d. Each movement is moved just to the point of resistance by the nurse.

d. Each movement is moved just to the point of resistance by the nurse.

A patient refuses medication. Which is the nurse's first action? a. Educate the patient about the importance of the medication. b. Discreetly hide the medication in the patient's favorite gelatin. c. Agree with the patient's decision and document it in the chart. d. Explore with the patient reasons for not wanting to take the medication.

d. Explore with the patient reasons for not wanting to take the medication.

The nurse is admitting a patient who has been diagnosed as having had a stroke. The health care provider writes orders for "ROM as needed." What should the nurse do next? a. Restrict patient's mobility as much as possible. b. Realize the patient is unable to move extremities. c. Move all the patient's extremities. d. Further assess the patient.

d. Further assess the patient.

The nurse is observing the way a patient walks. Which aspect is the nurse assessing? a. Activity tolerance b. Body alignment c. Range of motion d. Gait

d. Gait

10. The nurse knows that children in late infancy and toddlerhood are at risk for injury owing to a. Learning to walk. b. Trying to pull up on furniture. c. Being dropped by a caregiver. d. Growing ability to explore and oral activity.

d. Growing ability to explore and oral activity. Injury is a leading cause of death in children over age 1, which is closely related to normal growth and development because of the child's increased oral activity and growing ability to explore the environment.

Which is the best explanation for the nurse to provide when teaching the patient the reason for the binder after an open abdominal aortic aneurysm repair? a. It reduces edema at the surgical site. b. It secures the dressing in place. c. It immobilizes the abdomen. d. It supports the abdomen.

d. It supports the abdomen.

A patient is admitted after having experienced a stroke. The outcome of this disorder is uncertain, but the patient is unable to move the right arm and leg. The nurse starts passive range-of-motion (ROM) exercises. Which finding indicates successful goal achievement? a. Heart rate decreased. b. Contractures developed. c. Muscle strength improved. d. Joint mobility maintained.

d. Joint mobility maintained.

A patient is admitted and is placed on fall precautions. The nurse teaches the patient and family about fall precautions. Which action will the nurse take in accordance with hospital policy? a. Check on the patient once a shift. b. Encourage visitors in the early evening. c. Place all four side rails in the "up" position. d. Keep the patient on fall risk until discharge.

d. Keep the patient on fall risk until discharge.

A nurse is providing range of motion to the shoulder and must perform external rotation. Which action will the nurse take? a. Moves patient's arm in a full circle b. Moves patient's arm cross the body as far as possible c. Moves patient's arm behind body, keeping elbow straight d. Moves patient's arm until thumb is upward and lateral to head with elbow flexed

d. Moves patient's arm until thumb is upward and lateral to head with elbow flexed

Which item should the nurse use first to assist in staging an ulcer on the heel of a darkly pigmented skin patient? a. Disposable measuring tape b. Cotton-tipped applicator c. Sterile gloves d. Natural light

d. Natural light

The prescriber wrote for a 40-kg child to receive 25 mg of medication 4 times a day. The therapeutic range is 5 to 10 mg/kg/day. What is the nurse's priority? a. Change the dose to one that is within range. b. Administer the medication because it is within the therapeutic range. c. Notify the health care provider that the prescribed dose is in the toxic range. d. Notify the health care provider that the prescribed dose is below the therapeutic range.

d. Notify the health care provider that the prescribed dose is below the therapeutic range.

A registered nurse interprets that a scribbled medication prescription reads 25 mg. The nurse administers 25 mg of the medication to a patient and then discovers that the dose was incorrectly interpreted and should have been 15 mg. Who is ultimately responsible for the error? a. Health care provider b. Pharmacist c. Hospital d. Nurse

d. Nurse

22. The nurse is precepting a student nurse and is careful to check with the student all components of the medication process. The nurse explains to the student that most errors occur in a. Ordering and transcribing. b. Dispensing and administering. c. Dispensing and transcribing. d. Ordering and administering.

d. Ordering and administering. Most medication errors occur in the ordering and administering stages of the medication process.

A nurse reviews the history of a newly admitted patient. Which finding will alert the nurse that the patient is at risk for falls? a. 55 years old b. 20/20 vision c. Urinary continence d. Orthostatic hypotension

d. Orthostatic hypotension

20. A patient has been admitted and placed on fall precautions. The nurse explains to the patient that interventions for the precautions include a. Encouraging visitors in the early evening. b. Placing all four side rails in the "up" position. c. Checking on the patient once a shift. d. Placing a high risk for falls armband on the patient.

d. Placing a high risk for falls armband on the patient. Placing a high risk for falls armband on the patient encourages communication among the whole interdisciplinary team. Anyone who interacts with the patient should see this armband, understand its meaning, and assist the patient as necessary. The timing of visitors would not affect falls. All four side rails are considered a restraint and can contribute to falling. Individuals on high risk for fall alerts should be checked frequently, at least every hour.

The supervising nurse is watching nurses prepare medications. Which action by one of the nurses will result in the supervising nurse to intervene immediately? a. Rolls insulin vial between hands. b. Administers a dose of correction insulin. c. Draws up glargine (Lantus) in a syringe by itself. d. Prepares NPH insulin to be given intravenously (IV).

d. Prepares NPH insulin to be given intravenously (IV).

A nurse is performing passive range of motion (ROM) and splinting on an at-risk patient. Which finding will indicate goal achievement for the nurse's action? a. Prevention of atelectasis b. Prevention of renal calculi c. Prevention of pressure ulcers d. Prevention of joint contractures

d. Prevention of joint contractures

The nurse is caring for a patient who has experienced a laparoscopic appendectomy. For which type of healing will the nurse focus the care plan? a. Partial-thickness repair b. Secondary intention c. Tertiary intention d. Primary intention

d. Primary intention

The nurse is caring for a surgical patient. Which intervention is most important for the nurse to complete to decrease the risk of pressure ulcers and encourage the patient's willingness and ability to increase mobility? a. Explain the risks of immobility to the patient. b. Turn the patient every 3 hours while in bed. c. Encourage the patient to sit up in the chair. d. Provide analgesic medication as ordered.

d. Provide analgesic medication as ordered.

5. The nurse is caring for a patient with a urinary catheter. After the nurse empties the collection bag and disposes of the urine, the next step is to a. Use alcohol-based gel on hands. b. Wash hands with soap and water. c. Remove eye protection and dispose of in garbage. d. Remove gloves and dispose of in garbage.

d. Remove gloves and dispose of in garbage. After disposing of the urine, the first step in removing personal protective equipment is removing gloves and disposing of them properly. In this scenario, the next step would be to remove eye protection followed by hand hygiene. Wash hands if the hands are visibly soiled; otherwise the use of alcohol-based gel is indicated for routine decontamination of hands.

The patient is confused, is trying to get out of bed, and is pulling at the intravenous infusion tubing. Which nursing diagnosis will the nurse add to the care plan? a. Impaired home maintenance b. Deficient knowledge c. Risk for poisoning d. Risk for injury

d. Risk for injury

27. The patient is confused, is trying to get out of bed, and is pulling at the intravenous infusion tubing. These data would help to support a nursing diagnosis of a. Risk for poisoning. b. Knowledge deficit. c. Impaired home maintenance. d. Risk for injury.

d. Risk for injury The patient's behaviors support the nursing diagnosis of risk for injury. The patient is confused, is pulling at the intravenous line, and is trying to climb out of bed. Injury could result if the patient falls out of bed or begins to bleed from a pulled line. Nothing in the scenario indicates that this patient lacks knowledge or is at risk for poisoning. Nothing in the scenario refers to the patient's home maintenance.

Which nursing observation will indicate the patient's wound healed by the process of secondary intention? a. Minimal loss of tissue function b. Permanent dark redness at site c. Minimal scar tissue d. Scarring that may be severe

d. Scarring that may be severe

Which patient does the nurse most closely monitor for an unintended synergistic effect? a. The 4 year old who has mistakenly taken a half bottle of vitamins. b. The 35 year old who has ingested meth mixed with several household chemicals. c. The 50 year old who is prescribed a second blood pressure medication. d. The 72 year old who is seeing four different specialists.

d. The 72 year old who is seeing four different specialists.

The nurse is caring for a patient who has experienced a total abdominal hysterectomy. Which nursing observation related to the incision will indicate the patient is experiencing a complication of wound healing? a. Patient reporting, "My incision is hurting." b. Approximation of the incision edges has occurred. c. Patient asks, "Why has my incision started to itch?" d. The incision appears both swollen and bluish in color.

d. The incision appears both swollen and bluish in color.

When a comatose patient develops a Stage II pressure ulcer, the nurse includes the nursing diagnosis of Risk for infection to the care plan. Which is the best goal for this patient? a. The patient will state what to look for with regard to an infection. b. The patient's family will demonstrate specific care of the wound site. c. The patient's family members will wash their hands when visiting the patient. d. The patient will remain free of odorous or purulent drainage from the wound.

d. The patient will remain free of odorous or purulent drainage from the wound.

The nurse is caring for an older-adult patient with a diagnosis of urinary tract infection (UTI). Upon assessment the nurse finds the patient confused and agitated. How will the nurse interpret these assessment findings? a. These are normal signs of aging. b. These are early signs of dementia. c. These are purely psychological in origin. d. These are common manifestation with UTIs.

d. These are common manifestation with UTIs.

A nurse is caring for a patient who is experiencing some symptoms related to arthritis. The nurse is teaching the patient about this process. Which information will the nurse include in the teaching session? a. This will affect synovial fluid. b. This will affect the body systemically. c. This involves mostly non-weight-bearing joints. d. This involves an increased risk for impaired weight bearing.

d. This involves an increased risk for impaired weight bearing.

A patient has been prescribed to receive 0.3 mL of U-500 insulin. Which syringe will the nurse use to administer the medication? a. 3-mL syringe b. U-100 syringe c. Needleless syringe d. Tuberculin syringe

d. Tuberculin syringe

The patient is unable to move self and needs to be pulled up in bed. What will the nurse do to make this procedure safe? a. Place the pillow under the patient's head and shoulders. b. Do by self if the bed is in the flat position. c. Place the side rails in the up position. d. Use a friction-reducing device.

d. Use a friction-reducing device.

The nurse is monitoring for risks for injury identified in the health care environment. Which finding will alert the nurse that these safety risks are occurring? a. Tile floors, cold food, scratchy linen, and noisy alarms b. Dirty floors, hallways blocked, medication room locked, and alarms set c. Carpeted floors, ice machine empty, unlocked supply cabinet, and nurse call system in reach d. Wet floors unmarked, failure to use lift for patient, and alarms not functioning properly

d. Wet floors unmarked, failure to use lift for patient, and alarms not functioning properly

4. a nurse is performing skin assessments on a group of clients. Which of the following lesions should the nurse identify as vesicles? (select all that apply.) a. acne B. Warts c. Psoriasis d. herpes simplex e. Varicella

d. herpes simplex e. Varicella

SPEAKUP from the JC states what?

encourages patients to be active, involved, and informed participants on the healthcare team to protect from medical errors.

identify those who are at risk for falls and those who need more evaluation- stand up without using your arms for support

get up and go test

single most important thing to do to prevent falls?

identify those who are at risk

A patient safety event that reaches a patient and results in death, severe temporary harm, permanent harm:

sentinel event which Joint Commission must know about (falls, suicide, delay in treatment, medication error, etc)


Ensembles d'études connexes

Chapter 8: T cell mediated immunity

View Set

MedSurg Chapter 15-Oncology: Nursing Management in Cancer Care

View Set

SPAN 2320: Capitulos 6 y 7 Actividades

View Set